Download as pdf or txt
Download as pdf or txt
You are on page 1of 150

PASS PRO - Copyright 2004 Inmarkets Ltd - Licensed to Raymond chen

1/150

Print

Close

PASS PRO Questions


1.
If a cash flow of $10,000 in two years' time has a PV of $8,455, the annual percentage rate,
assuming continuous compounding is CLOSEST to:
A. 8.13%.
B. 8.39%.
C. 8.75%.
D. 8.95%.
2.
The present value of $700 received in 9 month's time,, using a continuous discount rate of 9%,
is CLOSEST to:
A. $653.
B. $654.
C. $655.
D. $656.
3.
If a cash flow of $100 in 2 years' time has a PV of $75, the annual percentage rate, assuming
continuous compounding is CLOSEST to:
A. 11.1%.
B. 12.2%.
C. 13.5%.
D. 14.4%.
4.
If a cash flow of $100 in 2 years' time has a PV of $75, the annual percentage rate, assuming
continuous compounding is CLOSEST to:
A. 11.1%.
B. 12.2%.
C. 13.5%.
D. 14.4%.
5.
The price of a stock has risen from $60 to $70 over the past 4 years. If the stock did not pay
any dividends during this period, its continuously compounded return is CLOSEST to:
A. 2.18%.

http://64.106.152.240/inmarkets/passpro/index.cfm?event=questions....

2004-08-02

PASS PRO - Copyright 2004 Inmarkets Ltd - Licensed to Raymond chen


2/150

B. 2.25%.
C. 3.35%.
D. 3.85%.
6.
The price of a stock has risen from $13 to $35 over the past five years. If the stock did not pay
any dividends during this period, its continuously compounded return is CLOSEST to:
A. 20%.
B. 22%.
C. 24%.
D. 26%.
7.
The present value of $800 received in 12 month's time,, using a continuous discount rate of
20%, is CLOSEST to:
A. $621.
B. $638.
C. $655.
D. $667.
8.
The price of a stock has risen from $60 to $70 over the past 4 years. If the stock did not pay
any dividends during this period, its continuously compounded return is CLOSEST to:
A. 2.18%.
B. 2.25%.
C. 3.35%.
D. 3.85%.
9.
The price of a stock has risen from $13 to $35 over the past five years. If the stock did not pay
any dividends during this period, its continuously compounded return is CLOSEST to:
A. 20%.
B. 22%.
C. 24%.
D. 26%.
10.
The price of a stock has risen from $30 to $45 over the past 5 years. If the stock did not pay
any dividends during this period, its continuously compounded return is CLOSEST to:

http://64.106.152.240/inmarkets/passpro/index.cfm?event=questions....

2004-08-02

PASS PRO - Copyright 2004 Inmarkets Ltd - Licensed to Raymond chen


3/150

A. 7.2%.
B. 8.1%.
C. 8.3%.
D. 8.6%.
11.
The present value of $4,500 received in 6 month's time, using a continuous discount rate of
9%, is CLOSEST to:
A. $3,290
B. $4,302
C. $4,306
D. $4,310
12.
If a cash flow of $700 in 3 years' time has a PV of $400, the annual percentage rate, assuming
continuous compounding is CLOSEST to:
A. 8.52%.
B. 12.16%.
C. 18.65%.
D. 25.44%.
13.
The present value of $800 received in 12 month's time,, using a continuous discount rate of
20%, is CLOSEST to:
A. $621.
B. $638.
C. $655.
D. $667.
14.
The price of a stock has risen from $15 to $40 over the past 2 years. If the stock did not pay
any dividends during this period, its continuously compounded return is CLOSEST to:
A. 9.15%
B. 14.94%
C. 29.24%
D. 49.04%
15.
If a cash flow of $800 in 6 years' time has a PV of $500, the annual percentage rate, assuming

http://64.106.152.240/inmarkets/passpro/index.cfm?event=questions....

2004-08-02

PASS PRO - Copyright 2004 Inmarkets Ltd - Licensed to Raymond chen


4/150

continuous compounding is CLOSEST to:


A. 4.48%.
B. 5.23%.
C. 7.83%.
D. 9.12%.
16.
The present value of $700 received in 9 month's time,, using a continuous discount rate of 9%,
is CLOSEST to:
A. $653.
B. $654.
C. $655.
D. $656.
17.
Given that a stock price rises on two days out of five and falls on three days out of five, what is
the probability that it will rise on exactly seven out of the next eight days?
A. 0.8%.
B. 4.5%.
C. 9.0%.
D. 12.0%.
18.
A portfolio has a mean value of $75 million and a daily standard deviation of $4.27 million.
Assuming that the portfolio values are normally distributed, the probability of the portfolio value
falling below $40 million within the next seven days is CLOSEST to:
A. 0.10%.
B. 1.00%.
C. 5.00%.
D. 15.87%.
19.
A standard normal distribution has:
A. no tails.
B. fat tails.
C. infinite tails.
D. asymmetric tails.
20.

http://64.106.152.240/inmarkets/passpro/index.cfm?event=questions....

2004-08-02

PASS PRO - Copyright 2004 Inmarkets Ltd - Licensed to Raymond chen


5/150

The covariance between the return from two securities is 4 and the correlation between them is
0.5. If the variance of the first return is 16, the variance of the second return will be CLOSEST
to:
A. 0.25.
B. 0.50.
C. 2.00.
D. 4.00.
21.
Which of the following are characteristics of a normal distribution?
I. Skewness equal to zero.
II. Mean less than median.
III. Kurtosis greater than zero.
IV. Continuous and unbounded.
A. I and IV.
B. II and III.
C. I, II and III.
D. I, III and IV.
22.
What is the quantile corresponding to the 95% confidence interval?
A. -3.715.
B. -2.326.
C. -1.645.
D. -1.282.
23.
Consider a portfolio whose expected return is normally distributed with a mean of 20 percent
and a standard deviation of 10 percent. The probability that the return will lie between 0
percent and 10 percent is CLOSEST to:
A. 2%.
B. 6%.
C. 14%.
D. 19%.
24.
Assume a variable with normal distribution. The mean is 100. The variance is 100. The
probability of observing an outcome of 77 and below is approximately:
A. 1.00%.
B. 10.35%.

http://64.106.152.240/inmarkets/passpro/index.cfm?event=questions....

2004-08-02

PASS PRO - Copyright 2004 Inmarkets Ltd - Licensed to Raymond chen


6/150

C. 12.35%.
D. 50.35%.
25.
An analyst determines that the S&P500 falls every 3 out of 8 days but without any serial
dependency, i.e. probability of a fall on a certain date is not affected by its movement on the
previous days. Given that there are 285 trading days in a year, the standard deviation in the
number of days that the S&P500 will fall is CLOSEST to:
A. 4
B. 6
C. 8
D. 14
26.
A portfolio has a mean value of $90 million and a daily standard deviation of $9 million.
Assuming that the portfolio values are normally distributed, the lowest value that the portfolio
will fall to over the next ten days and within 99.9% probability is:
A. $1.8 million.
B. $23.8 million.
C. $43.2 million.
D. $61.5 million.
27.
The covariance between the return from two securities is 4.2 and the correlation between them
is 0.6. If the variance of the first return is 5, the variance of the second return will be CLOSEST
to:
A. 1.40.
B. 1.96.
C. 3.13.
D. 9.80.
28.
What is the quantile corresponding to the 99% confidence interval?
A. -3.715.
B. -2.326.
C. -1.645.
D. -1.282.
29.
Which of the following are characteristic of a normal distribution?

http://64.106.152.240/inmarkets/passpro/index.cfm?event=questions....

2004-08-02

PASS PRO - Copyright 2004 Inmarkets Ltd - Licensed to Raymond chen


7/150

I. It is bell shaped.
II. It is a continuous distribution.
III. It is symmetrical about the mean.
IV. It peaks at the mean expected value.
A. I and II.
B. I and IV.
C. I, II and III.
D. I, II, III and IV.
30.
The mean age of the 80 employees in a company is 35 and the standard deviation is 15.
Assuming that the ages are normally distributed and using 95 percent confidence, we can say
that the employees within the firm fall between:
A. 20.0 and 50.0 years.
B. 31.7 and 38.3 years.
C. 33.8 and 36.2 years.
D. 34.6 and 35.4 years.
31.
Which of the following is the most appropriate for modeling stock prices?
A. Normal distribution.
B. Poisson distribution.
C. Random distribution.
D. Lognormal distribution.
32.
Which of the following are characteristics of a normal distribution?
I. Mean less than median.
II. Skewness equal to three.
III. Kurtosis greater than zero.
IV. Continuous and unbounded.
A. I and II.
B. III and IV.
C. I, III and IV.
D. II, III and IV.
33.
Given that a stock price rises on two days out of five and falls on three days out of five, what is
the probability that it will rise on exactly seven out of the next eight days?
A. 0.8%.

http://64.106.152.240/inmarkets/passpro/index.cfm?event=questions....

2004-08-02

PASS PRO - Copyright 2004 Inmarkets Ltd - Licensed to Raymond chen


8/150

B. 4.5%.
C. 9.0%.
D. 12.0%.
34.
The covariance between the return from two securities is 4 and the correlation between them is
0.5. If the variance of the first return is 16, the variance of the second return will be CLOSEST
to:
A. 0.25.
B. 0.50.
C. 2.00.
D. 4.00.
35.
The covariance between the return from two securities is 15 and the correlation between them
is 0.5. If the variance of the first return is 25, the variance of the second return will be
CLOSEST to:
A. 1.20.
B. 1.44.
C. 6
D. 36
36.
An analyst determines that in the current market environment the S&P500 goes up five out of
every seven days but without any serial dependency, i.e. probability of going up on a certain
date is not affected by its movement on the previous days. Given that 280 trading days in a
year the standard deviation in the number of days that the S&P500 will go up is CLOSEST to:
A. 7.56.
B. 11.95.
C. 57.18.
D. 142.74.
37.
A standard normal distribution has:
A. no tails.
B. fat tails.
C. infinite tails.
D. asymmetric tails.
38.

http://64.106.152.240/inmarkets/passpro/index.cfm?event=questions....

2004-08-02

PASS PRO - Copyright 2004 Inmarkets Ltd - Licensed to Raymond chen


9/150

A portfolio has a mean value of $140 million and a daily standard deviation of $1.5 million.
Assuming that the portfolio values are normally distributed, the lowest value that the portfolio
will fall to over the next 250 days and within 95% probability is:
A. $66.5 million.
B. $84.9 million.
C. $101 million.
D. $116.3 million.
39.
Which of the following is the most appropriate for modeling stock prices?
A. Normal distribution.
B. Poisson distribution.
C. Random distribution.
D. Lognormal distribution.
40.
A portfolio has a mean value of $100 million and a daily standard deviation of $19 million.
Assuming that the portfolio values are normally distributed, the lowest value that the portfolio
will fall to over the next five days and within 95% probability is:
A. -$31.7 million.
B. $1.2 million.
C. $30.1 million.
D. $57.5 million.
41.
The covariance between the return from two securities is 22 and the correlation between them
is 0.8. If the variance of the first return is 25, the variance of the second return will be
CLOSEST to:
A. 1.10.
B. 1.21.
C. 5.50.
D. 30.25.
42.
Pat Fineman is due to receive $6,000 one year from now, which he can invest at an interest
rate of 4 percent for a further four years. The value of this payment at the end of this period
will be CLOSEST to:
A. $6,960
B. $7,019
C. $7,200

http://64.106.152.240/inmarkets/passpro/index.cfm?event=questions....

2004-08-02

PASS PRO - Copyright 2004 Inmarkets Ltd - Licensed to Raymond chen

10/150

D. $7,300
43.
Assuming continuous compounding and an annual percentage rate of 8%, the present value of
$8,400 received in six months' time is CLOSEST to:
A. $6,203.
B. $8,071.
C. $8,077.
D. $8,083.
44.
If a cash flow of $21,000 in two years' time has a PV of $16,000, the annual percentage rate,
assuming continuous compounding is CLOSEST to:
A. 13.60%.
B. 13.80%.
C. 14.24%.
D. 14.56%.
45.
An investor is being promised an annuity of $4,000 starting in 8 years from now and continuing
for 10 years. Assuming a constant interest rate of 5.25 percent per annum, the value of this
annuity today is CLOSEST to:
A. $21,328.
B. $26,563.
C. $27,958.
D. $30,515.
46.
What is the standard deviation of the following set of numbers?
{2, 4, 6, 8, 10}.
A. 2.
B. 2.8.
C. 3.2.
D. 3.6.
47.
A bank is offering loans with monthly interest payments based on a stated annual interest rate
of 9 percent. The effective annual rate of these loans is CLOSEST to:
A. 9.20%.

http://64.106.152.240/inmarkets/passpro/index.cfm?event=questions....

2004-08-02

PASS PRO - Copyright 2004 Inmarkets Ltd - Licensed to Raymond chen

11/150

B. 9.31%.
C. 9.34%.
D. 9.38%.
48.
If a cash flow of $10,000 in five years' time has a PV of $7,500, the annual percentage rate,
assuming continuous compounding, is CLOSEST to:
A. 5.65%.
B. 5.75%.
C. 5.85%.
D. 5.92%.
49.
If all the cash flows in an investment are positive what is its IRR?
A. Positive.
B. Zero.
C. Negative.
D. Cannot be determined.
50.
Peter Roche is due to receive $4,000 one year from now, which he can invest at an interest rate
of 7 percent for a further ten years. The value of this payment at the end of this period will be
CLOSEST to:
A. $6,800.
B. $7,080.
C. $7,869.
D. $8,419.
51.
An investor is being promised an annuity of $900 starting in two years from now and continuing
for 25 years. Assuming a constant interest rate of 8 percent per annum, the value of this
annuity today is CLOSEST to:
A. $8,895.
B. $9,607.
C. $19,290.
D. $20,833.
52.
An investor is being promised an annuity of $1,500 starting in 15 years from now and
continuing for ten years. Assuming a constant interest rate of 8 percent per annum, the value

http://64.106.152.240/inmarkets/passpro/index.cfm?event=questions....

2004-08-02

PASS PRO - Copyright 2004 Inmarkets Ltd - Licensed to Raymond chen

12/150

of this annuity today is CLOSEST to:


A. $3,427.
B. $4,729.
C. $5,107.
D. $10,065.
53.
Sam Walsh is due to receive $7,000 one year from now, which he can invest at an interest rate
of 6 percent for a further eight years. The value of this payment at the end of this period will be
CLOSEST to:
A. $10,360.
B. $10,780.
C. $11,157.
D. $11,826.
54.
If a cash flow of $8,000 in four year's time has a PV of $7,000, the annual percentage rate,
assuming continuous compounding is CLOSEST to:
A. 3.24%.
B. 3.29%.
C. 3.34%.
D. 3.39%.
55.
What is the standard deviation of the following set of numbers?
{2, 4, 6, 8, 10}.
A. 2.
B. 2.8.
C. 3.2.
D. 3.6.
56.
If a cash flow of $21,000 in two years' time has a PV of $16,000, the annual percentage rate,
assuming continuous compounding is CLOSEST to:
A. 13.60%.
B. 13.80%.
C. 14.24%.
D. 14.56%.

http://64.106.152.240/inmarkets/passpro/index.cfm?event=questions....

2004-08-02

PASS PRO - Copyright 2004 Inmarkets Ltd - Licensed to Raymond chen

13/150

57.
An investor is being promised an annuity of $2,000 starting in two years from now and
continuing for eight years. Assuming a constant interest rate of 4 percent per annum, the value
of this annuity today is CLOSEST to:
A. $12,947.
B. $13,465.
C. $14,793.
D. $15,385.
58.
A bank is offering loans with monthly interest payments based on a stated annual interest rate
of 7.79 percent. The effective annual rate of these loans is CLOSEST to:
A. 8.02%.
B. 8.05%.
C. 8.07%.
D. 8.09%.
59.
If a cash flow of $8,000 in four year's time has a PV of $7,000, the annual percentage rate,
assuming continuous compounding is CLOSEST to:
A. 3.24%.
B. 3.29%.
C. 3.34%.
D. 3.39%.
60.
Suppose that Gene owns a perpetuity, issued by an insurance company that pays $1,250 at the
end of each year. The insurance company now wishes to replace it with a decreasing perpetuity
of $1,500 decreasing at 1% p.a. without any change in the payment dates. At what rate of
interest (assuming a flat yield curve) would Gene be indifferent between the choices?
A. 4%.
B. 5%.
C. 6%.
D. 9%.
61.
Sam Walsh is due to receive $7,000 one year from now, which he can invest at an interest rate
of 6 percent for a further eight years. The value of this payment at the end of this period will be
CLOSEST to:
A. $10,360.

http://64.106.152.240/inmarkets/passpro/index.cfm?event=questions....

2004-08-02

PASS PRO - Copyright 2004 Inmarkets Ltd - Licensed to Raymond chen

14/150

B. $10,780.
C. $11,157.
D. $11,826.
62.
Pat Fineman is due to receive $6,000 one year from now, which he can invest at an interest
rate of 4 percent for a further four years. The value of this payment at the end of this period
will be CLOSEST to:
A. $6,960
B. $7,019
C. $7,200
D. $7,300
63.
The inflation rate in an economy where the prices are doubling ever eight years is CLOSEST to:
A. 6%.
B. 9%.
C. 13%.
D. 25%.
64.
What is the approximate price of a zero coupon bond whose par value is 100 and term is two
years if its yields is 5%?
A. 86.
B. 91.
C. 96.
D. 101.
65.
Consider two cash flows, the first of $10,000 receivable after one year and the second one of
$11,000 receivable after three years. At what rate of semi-annual interest would you be
indifferent between the two?
A. 3.8%.
B. 4.8%.
C. 5.8%.
D. 6.0%.
66.
The semi-annual yield in the market for one year is 5%. An options trader is pricing 1-year
options under Black Scholes. What is the rate of interest he should input in his model?

http://64.106.152.240/inmarkets/passpro/index.cfm?event=questions....

2004-08-02

PASS PRO - Copyright 2004 Inmarkets Ltd - Licensed to Raymond chen

15/150

A. 4.8500%.
B. 4.9385%.
C. 5.0000%.
D. 5.0625%.
67.
Which of the following statements are TRUE?
I. Correlation coefficient falls between -1 and +1.
II. Independent stochastic variables have a correlation coefficient of -1.
III. Correlation coefficient measures the non-linear relationship between two variables.
IV. Correlation coefficient can be calculated by scaling the covariance between two variables.
A. I and IV.
B. II and III.
C. I, II and III.
D. I, III and IV.
68.
A trader in your firm is convinced that the stock index in country X is perfectly negatively
correlated to the S&P 500. In order to profit from this analysis, he has taken a long position on
index X and shorted S&P 500 futures. Which of the following is TRUE?
A. This is a riskless trade because the stocks have negative correlation.
B. Apart from the currency risk and cash flow risks on margin calls, this is almost a riskless
strategy.
C. This trading strategy has the same risk as shorting the S&P 500. This is not a hedged
position.
D. There will be some small residual risk due to the currency conversion. Otherwise, it is a
virtually riskless strategy.
69.
The covariance between the return from two securities is 5 and the correlation between them is
0.5. If the variance of the first return is 8, the variance of the second return will be CLOSEST
to:
A. 1.25.
B. 1.56.
C. 3.54.
D. 12.50.
70.
What is the mean for the following probability data?
Probability Value
60% -2

http://64.106.152.240/inmarkets/passpro/index.cfm?event=questions....

2004-08-02

PASS PRO - Copyright 2004 Inmarkets Ltd - Licensed to Raymond chen

16/150

20% 10
20% 20
A. 1.60.
B. 2.80.
C. 3.60.
D. 4.80.
71.
Given the following data for a market variable what is the best estimate of its variance?
Probability Value
24% -12
40% 4
36% 14
A. 6
B. 10
C. 32
D. 97
72.
Which of the following statements about the correlation coefficient are FALSE?
I. Correlation coefficient ranges between 0 and +1.
II. If two random variables are independent they will have a correlation coefficient of zero.
III. Correlation coefficient can be calculated by scaling the covariance between two variables.
IV. Correlation coefficient is a measure of the linear relationship between a dependent and an
independent variable.
A. I and II.
B. I and IV.
C. II and III.
D. III and IV.
73.
The covariance between the return from two securities is 5 and the correlation between them is
0.5. If the variance of the first return is 8, the variance of the second return will be CLOSEST
to:
A. 1.25.
B. 1.56.
C. 3.54.
D. 12.50.
74.

http://64.106.152.240/inmarkets/passpro/index.cfm?event=questions....

2004-08-02

PASS PRO - Copyright 2004 Inmarkets Ltd - Licensed to Raymond chen

17/150

Consider a stock whose 5-day volatility has been estimated as 0.89%. Assuming that the
returns from the stock are uncorrelated over time, the volatility over 22 days will be CLOSEST
to:
A. 0.42%.
B. 1.87%.
C. 3.92%.
D. 4.17%.
75.
What is the mean for the following probability data?
Probability Value
50% -2
25% 8
25% 15
A. 1.58.
B. 2.25.
C. 4.75.
D. 5.38.
76.
What is the standard deviation of the following data?
Probability Value
60% -2
20% 10
20% 20
A. 5.14.
B. 8.91.
C. 26.45.
D. 79.36.
77.
What is the mean for the following probability data?
Probability Value
30% -10
40% 5
30% 25
A. 1.15.
B. 2.17.
C. 3.34.
D. 6.50.

http://64.106.152.240/inmarkets/passpro/index.cfm?event=questions....

2004-08-02

PASS PRO - Copyright 2004 Inmarkets Ltd - Licensed to Raymond chen

18/150

78.
Consider a stock whose 10-day volatility has been estimated as 1.4%. Assuming that the
returns from the stock are uncorrelated over time, the volatility over 23 days will be CLOSEST
to:
A. 0.92%.
B. 2.12%.
C. 3.22%.
D. 6.71%.
79.
Consider a stock whose 5-day volatility has been estimated as 1.2%. Assuming that the returns
from the stock are uncorrelated over time, the volatility over 22 days will be CLOSEST to:
A. 0.57%.
B. 2.52%.
C. 5.28%.
D. 5.63%.
80.
What is the standard deviation of the following data?
Probability Value
30% -15
40% 5
30% 25
A. 8.94.
B. 15.49.
C. 80.00.
D. 240.00.
81.
Asset 1 has correlation of 0.5 with asset 2. A portfolio with equal weights of these two assets
has a standard deviation of 13. The standard deviation of asset 2 is 19.50. What is the
approximate standard deviation of asset 1?
A. 5.
B. 10.
C. 20.
D. Insufficient Information.
82.
Asset 1 has correlation of 0.5 with asset 2. A portfolio with equal weights of these two assets
has a standard deviation of 13. The standard deviation of asset 2 is 19.50. What is the

http://64.106.152.240/inmarkets/passpro/index.cfm?event=questions....

2004-08-02

PASS PRO - Copyright 2004 Inmarkets Ltd - Licensed to Raymond chen

19/150

approximate standard deviation of asset 1?


A. 5.
B. 10.
C. 20.
D. Insufficient Information.
83.
Which of the following statements about the correlation coefficient are FALSE?
I. Correlation coefficient ranges between -1 and +1.
II. Correlation coefficient is a measure of non-linear relationship between two random variables.
III. Correlation coefficient can be calculated by scaling the covariance between two variables.
IV. If the correlation coefficient between two random variables is zero they will be independent.
A. I and II.
B. I and III.
C. II and IV.
D. I, III and IV.
84.
What is the mean for the following probability data?
Probability Value
20% -4
40% 6
40% 12
A. 2.13.
B. 6.40.
C. 12.22.
D. 16.17.
85.
Which of the following statements are TRUE?
I. Correlation coefficient ranges between -1 and +1.
II. Correlation coefficient is a measure of linear relationship between two random variables.
III. Correlation coefficient can be calculated by scaling the covariance between two variables.
IV. If the correlation coefficient between two random variables is zero they will be independent.
A. I and II.
B. III and IV.
C. I, II and III.
D. I, II, III and IV.
86.

http://64.106.152.240/inmarkets/passpro/index.cfm?event=questions....

2004-08-02

PASS PRO - Copyright 2004 Inmarkets Ltd - Licensed to Raymond chen

20/150

Consider a stock whose 5-day volatility has been estimated as 0.89%. Assuming that the
returns from the stock are uncorrelated over time, the volatility over 22 days will be CLOSEST
to:
A. 0.42%.
B. 1.87%.
C. 3.92%.
D. 4.17%.
87.
The covariance between the return from two securities is 5 and the correlation between them is
0.5. If the variance of the first return is 8, the variance of the second return will be CLOSEST
to:
A. 1.25.
B. 1.56.
C. 3.54.
D. 12.50.
88.
Which of the following statements are TRUE?
I. Correlation coefficient ranges between -1 and +1.
II. If two random variables are independent they will have a correlation coefficient of zero.
III. Correlation coefficient is a measure of the linear relationship between a dependent and an
independent variable.
IV. Covariance between two variables can be calculated from the variances of the variables and
their correlation coefficient.
A. I and II.
B. I and III.
C. II and IV.
D. I, II, III and IV.
89.
Consider a stock whose 5-day volatility has been estimated as 1.5%. Assuming that the returns
from the stock are uncorrelated over time, the volatility over 250 days will be CLOSEST to:
A. 0.21%.
B. 10.61%.
C. 23.72%.
D. 75.00%.
90.
What is the mean for the following probability data?

http://64.106.152.240/inmarkets/passpro/index.cfm?event=questions....

2004-08-02

PASS PRO - Copyright 2004 Inmarkets Ltd - Licensed to Raymond chen

21/150

Probability Value
20% -4
40% 6
40% 12
A. 2.13.
B. 6.40.
C. 12.22.
D. 16.17.
91.
What is the mean for the following probability data?
Probability Value
50% -2
25% 8
25% 15
A. 1.58.
B. 2.25.
C. 4.75.
D. 5.38.
92.
An analyst is studying a stock that is currently trading at $35. The analyst estimates that there
is 33 percent probability that the stock will trade at $50 after one year, a 20 percent probability
that the stock will trade at $42, and a 47 percent chance that the stock will trade at $20. What
is the implied volatility of this stock price?
A. 13%.
B. 24%.
C. 31%.
D. 39%.
93.
Which of the following investments has the highest co-efficient of variation?
A. An investment with an expected return of 15% and a variance of return of 0.008.
B. An investment with an expected return of 12% and a variance of return of 0.005.
C. An investment with an expected return of 8% and standard deviation of return of 3%.
D. An investment with an expected return of 3% and standard deviation of return of 2%.
94.
An analyst regresses the returns of 60 stocks in a stock market and finds that the best fitting
line is:

http://64.106.152.240/inmarkets/passpro/index.cfm?event=questions....

2004-08-02

PASS PRO - Copyright 2004 Inmarkets Ltd - Licensed to Raymond chen

22/150

Return = 8% + 9% x Beta
If the standard error of the estimate is 6% and the standard error of the coefficient of Beta is
4%, the test statistic for the coefficient is CLOSEST to:
A. 1.33.
B. 1.43.
C. 1.50.
D. 2.25.
95.
Which of the following investment has the highest co-efficient of variation?
A. An investment with an expected return of 19% and a variance of return of 0.005.
B. An investment with an expected return of 15% and a variance of return of 0.002.
C. An investment with an expected return of 10% and standard deviation of return of 3%.
D. An investment with an expected return of 3% and standard deviation of return of 1%.
96.
Which of the following investments has the highest co-efficient of variation?
A. An investment with an expected return of 15% and a variance of of return of 0.008.
B. An investment with an expected return of 12% and a variance of of return of 0.005.
C. An investment with an expected return of 8% and standard deviation of return of 3%.
D. An investment with an expected return of 3% and standard deviation of return of 2%.
97.
An analyst collects the data for interest rate expectations. The mean expected rate is 2.5
percent, the lowest expectation is 1 percent and the highest expectation is 5 percent. This
distribution is:
A. sparse.
B. normal.
C. skewed.
D. abnormal.
98.
Using the returns from 64 stocks, an analyst determines that the best fitting line for the capital
market relationship is:
Return = 2% + 12% x Beta
If the standard error of the estimate is 3% and the standard error of the coefficient of Beta is
4%, the test statistic for the coefficient is CLOSEST to:
A. 2

http://64.106.152.240/inmarkets/passpro/index.cfm?event=questions....

2004-08-02

PASS PRO - Copyright 2004 Inmarkets Ltd - Licensed to Raymond chen

23/150

B. 3
C. 4
D. 6
99.
An analyst regresses the returns of 60 stocks in a stock market and finds that the best fitting
line is:
Return = 8% + 9% x Beta
If the standard error of the estimate is 6% and the standard error of the coefficient of Beta is
4%, the test statistic for the coefficient is CLOSEST to:
A. 1.33.
B. 1.43.
C. 1.50.
D. 2.25.
100.
An analyst regresses the returns of 16 stocks in a stock market and finds that the best fitting
line is:
Return = 6.5% + 11.8% x Beta
If the standard error of the estimate is 4% and the standard error of the coefficient of Beta is
3%, the test statistic for the coefficient is CLOSEST to:
A. 1.63.
B. 2.95.
C. 2.80.
D. 3.93.
101.
If the mean P/E of 30 stocks in a certain industrial sector is 18 and the sample standard
deviation is 3.5, standard error of the mean is CLOSEST to:
A. 0.12.
B. 0.34.
C. 0.64.
D. 1.56.
102.
An analyst regresses the returns of 50 stocks in a stock market and finds that the best fitting
line is:
Return = 8% + 12.25% x Beta

http://64.106.152.240/inmarkets/passpro/index.cfm?event=questions....

2004-08-02

PASS PRO - Copyright 2004 Inmarkets Ltd - Licensed to Raymond chen

24/150

If the standard error of the estimate is 10% and the standard error of the coefficient of Beta is
8%, the test statistic for the coefficient is CLOSEST to:
A. 0.80.
B. 1.16.
C. 1.23.
D. 1.53.
103.
Which of the following investments has the highest co-efficient of variation?
A. An investment with an expected return of 15% and a variance of return of 0.008.
B. An investment with an expected return of 12% and a variance of return of 0.005.
C. An investment with an expected return of 8% and standard deviation of return of 3%.
D. An investment with an expected return of 3% and standard deviation of return of 2%.
104.
Using the returns from 64 stocks, an analyst determines that the best fitting line for the capital
market relationship is:
Return = 2% + 12% x Beta
If the standard error of the estimate is 3% and the standard error of the coefficient of Beta is
4%, the test statistic for the coefficient is CLOSEST to:
A. 2
B. 3
C. 4
D. 6
105.
Which of the following investments has the highest co-efficient of variation?
A. An investment with an expected return of 12% and a variance of of return of 0.005.
B. An investment with an expected return of 10% and a variance of of return of 0.002.
C. An investment with an expected return of 15% and standard deviation of return of 3%.
D. An investment with an expected return of 6% and standard deviation of return of 5%.
106.
Which of the following investments has the highest co-efficient of variation?
A. An investment with an expected return of 10% and a variance of of return of 0.002.
B. An investment with an expected return of 15% and a variance of of return of 0.003.
C. An investment with an expected return of 8% and standard deviation of return of 3%.
D. An investment with an expected return of 5% and standard deviation of return of 1%.

http://64.106.152.240/inmarkets/passpro/index.cfm?event=questions....

2004-08-02

PASS PRO - Copyright 2004 Inmarkets Ltd - Licensed to Raymond chen

25/150

107.
If the mean P/E of 40 stocks in a certain industrial sector is 12 and the sample standard
deviation is 4, standard error of the mean is CLOSEST to:
A. 0.10.
B. 0.32.
C. 0.63.
D. 1.58.
108.
Which of the following investments has the highest co-efficient of variation?
A. An investment with an expected return of 15% and a variance of of return of 0.008.
B. An investment with an expected return of 12% and a variance of of return of 0.005.
C. An investment with an expected return of 8% and standard deviation of return of 3%.
D. An investment with an expected return of 3% and standard deviation of return of 2%.
109.
If the mean P/E of 90 stocks in a certain industrial sector is 12 and the sample standard
deviation is 6, standard error of the mean is CLOSEST to:
A. 0.07.
B. 0.26.
C. 0.63.
D. 1.58.
110.
If the mean P/E of 50 stocks in a certain industrial sector is ten and the sample standard
deviation is six, standard error of the mean is CLOSEST to:
A. 0.12.
B. 0.35.
C. 0.85.
D. 1.18.
111.
If the mean P/E of 60 stocks in a certain industrial sector is 18 and the sample standard
deviation is 7, standard error of the mean is CLOSEST to:
A. 0.12.
B. 0.34.
C. 0.90.
D. 1.11.

http://64.106.152.240/inmarkets/passpro/index.cfm?event=questions....

2004-08-02

PASS PRO - Copyright 2004 Inmarkets Ltd - Licensed to Raymond chen

26/150

112.
If the mean P/E of 90 stocks in a certain industrial sector is 12 and the sample standard
deviation is 6, standard error of the mean is CLOSEST to:
A. 0.07.
B. 0.26.
C. 0.63.
D. 1.58.
113.
An analyst regresses the returns of 50 stocks in a stock market and finds that the best fitting
line is:
Return = 8% + 9% x Beta
If the standard error of the estimate is 5.5% and the standard error of the coefficient of Beta is
3.25%, the test statistic for the coefficient is CLOSEST to:
A. 1.45.
B. 1.64.
C. 1.55.
D. 2.77.
114.
An analyst wants to test whether the variance of return from pharmaceutical stocks is different
from that of the overall market. For this purpose, he obtains the following data from a sample
of 21 pharmaceutical stocks and a sample of 41 stocks that are representative of the market.
Mean return from pharmaceutical stocks = 8% Standard deviation of return from
pharmaceutical stocks = 9.2% Mean return from market stocks = 12% Standard deviation of
return from market stocks = 13% Based on this information and a 0.05 significance level:
A. there is sufficient evidence for a difference between the variance of pharmaceutical stocks
and the variance of the market stocks.
B. there is insufficient evidence for a difference between the variance of pharmaceutical stocks
and the variance of the market stocks.
C. there is sufficient evidence that there is no difference between the variance of
pharmaceutical stocks and the variance of the market stocks.
D. there is insufficient evidence that there is no difference between the variance of
pharmaceutical stocks and the variance of the market stocks.
115.
An analyst wants to test whether the mean spending by tourists coming to a holiday resort is
equal to or less than $2,000 with a 1 percent level of significance. He finds that the average
spending by 16 tourists is $2,200 and the standard deviation of the population is $400. The
critical value of the Z statistic for this study is:
A. 1.65.
B. -1.96.

http://64.106.152.240/inmarkets/passpro/index.cfm?event=questions....

2004-08-02

PASS PRO - Copyright 2004 Inmarkets Ltd - Licensed to Raymond chen

27/150

C. 2.33.
D. 2.58.
116.
A t-test is used instead of a z-test when:
A. the sample size is small.
B. greater accuracy is required.
C. the variance of the population is known.
D. the standard deviation is larger than the mean.
117.
An analyst wants to test whether the standard deviation of return from pharmaceutical stocks is
lower than 0.2. For this purpose, he obtains the following data from a sample of 30
pharmaceutical stocks. Mean return from pharmaceutical stocks = 8%. Standard deviation of
return from pharmaceutical stocks = 12%. Mean return from the market = 12%. Standard
deviation of return from the market = 16%. What is the appropriate test statistic for this test?
A. t-statistic.
B. z-statistic.
C. F-statistic.
D. Chi-squared statistic.
118.
Using a sample size of 61 observations, an analyst determines that the standard deviation of
the returns from a stock is 21 percent. Using a 0.05 significance level, the analyst:
A. can conclude that the standard deviation of returns is higher than 14%.
B. cannot conclude that the standard deviation of returns is higher than 14%.
C. can conclude that the standard deviation of returns is not higher than 14%.
D. none of the above.
119.
An analyst wants to test whether the return from utility stocks are less volatile than that of the
overall market. For this purpose, he obtains the following data from a sample of 21 utility
stocks and a sample of 41 stocks that are representative of the market. Mean return from utility
stocks = 9%. Standard deviation of return from utility stocks = 11%. Mean return from the
market stocks = 12%. Standard deviation of return from the market stocks = 15%. Based on
this information and a 0.05 significance level:
A. there is sufficient evidence to say that the standard deviation of utility stocks lower than that
of the market.
B. there is sufficient evidence to say that the standard deviation of utility stocks higher than
that of the market.
C. there is insufficient evidence to say that the standard deviation of utility stocks lower than
that of the market.

http://64.106.152.240/inmarkets/passpro/index.cfm?event=questions....

2004-08-02

PASS PRO - Copyright 2004 Inmarkets Ltd - Licensed to Raymond chen

28/150

D. none of the above.


120.
Using a sample size of 20 observations, an analyst determines that the standard deviation of
the returns from a stock is 12 percent. Using a 0.05 significance level, the analyst:
A. can conclude that the standard deviation of returns is same as 18%.
B. can conclude that the standard deviation of returns is different from 18%.
C. cannot conclude that the standard deviation of returns is different from 18%.
D. none of the above.
121.
An analyst wants to test whether the return from transportation sector stocks is different from
that of the utility stocks. For this purpose, he obtains the following data from a sample of 21
transportation stocks and a sample of 41 utility stocks. Mean return from transportation stocks
= 15%. Standard deviation of return from transportation stocks = 12%. Mean return from
utility stocks = 12%. Standard deviation of return from utility stocks = 13%. Based on this
information and using a 0.05 significance level:
A. there is no evidence for a difference between the means.
B. there is sufficient evidence for a difference between the means.
C. there is insufficient evidence for a difference between the means.
D. none of the above.
122.
Using a sample size of 15 observations, an analyst determines that the standard deviation of
the returns from a stock is 10 percent. Using a 0.05 significance level, the analyst:
A. can conclude that the standard deviation of returns is same as 15%.
B. can conclude that the standard deviation of returns is different from 15%.
C. cannot conclude that the standard deviation of returns is different from 15%.
D. none of the above.
123.
An analyst wants to test whether the return from transportation sector stocks is different from
that of the utility stocks. For this purpose, he obtains the following data from a sample of 21
transportation stocks and a sample of 41 utility stocks. Mean return from transportation stocks
= 9%. Standard deviation of return from transportation stocks = 13%. Mean return from utility
stocks = 12%. Standard deviation of return from utility stocks = 15%. What is the value of the
test statistic for this test?
A. 0.78.
B. 0.96.
C. 1.13.
D. 1.34.

http://64.106.152.240/inmarkets/passpro/index.cfm?event=questions....

2004-08-02

PASS PRO - Copyright 2004 Inmarkets Ltd - Licensed to Raymond chen

29/150

124.
An analyst wants to test whether the standard deviation of return from utility stocks is lower
than 0.14. For this purpose, he obtains the following data from a sample of 25 utility stocks.
Mean return from utility stocks = 8%. Standard deviation of return from utility stocks = 12%.
Mean return from the market = 10%. Standard deviation of return from the market = 15%.
What is the critical value of the statistic for this test at a 0.05 level of significance?
A. 13.85.
B. 23.77.
C. 36.42.
D. 39.36.
125.
An analyst wants to test whether the standard deviation of return from food and drinks stocks is
lower than 0.18. For this purpose, he obtains the following data from a sample of 15 food and
drinks stocks. Mean return from food and drinks stocks = 8%. Standard deviation of return
from food and drinks stocks = 13%. Mean return from the market = 12%. Standard deviation
of return from the market = 17% What is the value of the test statistic for this test?
A. 6.83.
B. 6.11.
C. 7.30.
D. 7.82.
126.
An analyst is given the task of determining whether a group of 16 active portfolio managers
have achieved a significantly higher performance (using a significance level of 0.05) than the
average for all portfolio managers over a certain period. Over the period of the study, the active
portfolio managers achieved a mean return of 15 percent. Over the same period the mean
return for all portfolio managers was 12 percent and their standard deviation was 8 percent.
The correct conclusion from this study is that:
A. the performance of active portfolio managers is significantly higher than the average for all
portfolio managers.
B. the performance of active portfolio managers is not significantly higher than the average for
all portfolio managers.
C. the performance of active portfolio managers is significantly lower than the average for all
portfolio managers.
D. the performance of active portfolio managers is the same as the average for all portfolio
managers.
127.
The z-statistic cannot be used to test the mean of a population when the population variance is:
A. known and the sample size is large.
B. known and the sample size is small.
C. unknown and the sample size is large.

http://64.106.152.240/inmarkets/passpro/index.cfm?event=questions....

2004-08-02

PASS PRO - Copyright 2004 Inmarkets Ltd - Licensed to Raymond chen

30/150

D. unknown and the sample size is small.


128.
Which of the following is NOT a step in the hypothesis testing process?
A. State a hypothesis.
B. Identify the test statistic and its probability distribution.
C. Obtain the significance level from the data.
D. Make the statistical decision.
129.
Rutherford Enterprises has a sales force of 1,000 sales people. The average turnover generated
by each sales person is $800,000 and the standard deviation of this population is $100,000. If
Peter, a salesman at Rutherford, has generated a turnover of $1,000,000, we can conclude,
using a significance level of 0.05, that Peter's performance:
A. is significantly higher than average.
B. is not significantly higher than average.
C. is lower or not significantly higher than average.
D. cannot be determined on the basis of this data alone.
130.
An analyst wants to test whether the return from utility stocks are less volatile than that of the
overall market. For this purpose, he obtains the following data from a sample of 21 utility
stocks and a sample of 41 stocks that are representative of the market. Mean return from utility
stocks = 9%. Standard deviation of return from utility stocks = 11%. Mean return from the
market stocks = 12%. Standard deviation of return from the market stocks = 15%. What is the
critical value of the statistic for this test at a 0.05 level of significance?
A. 1.84.
B. 1.99.
C. 2.07.
D. 2.29.
131.
An analyst wants to test whether the variance of return from oil and gas stocks is higher than
0.045. For this purpose, he obtains the following data from a sample of 60 oil and gas stocks.
Mean return from oil and gas stocks = 16%. Standard deviation of return from oil and gas
stocks = 22% Mean return from the market = 12%. Standard deviation of return from the
market = 14% Based on this information, we can say that at a 0.05 significance level the
variance of oil and gas firms is:
A. lower than 0.045.
B. higher than 0.045.
C. not higher than 0.045.
D. none of the above.

http://64.106.152.240/inmarkets/passpro/index.cfm?event=questions....

2004-08-02

PASS PRO - Copyright 2004 Inmarkets Ltd - Licensed to Raymond chen

31/150

132.
Using a sample size of 20 observations, an analyst determines that the standard deviation of
the returns from a stock is 9 percent. Using a 0.05 significance level, the analyst:
A. can conclude that the standard deviation of returns is same as 16%.
B. can conclude that the standard deviation of returns is different from 16%.
C. cannot conclude that the standard deviation of returns is different from 16%.
D. none of the above.
133.
You want to test at the 0.05 level of significance that the mean price of luxury cars is greater
than $80,000. A random sample of 50 cars has a mean price of $88,000. The population
standard deviation is $15,000. What is the alternative hypothesis?
A. The population mean is greater than or equal to $80,000.
B. The population mean is less than $80,000.
C. The population mean is not equal to $80,000.
D. The population mean is greater than is $80,000.
134.
An analyst is studying the impact of the dot-com bust on the beta of business services sector
stocks. From price data for 10 stocks in this sector, she has obtained the following results:
Mean beta of the stocks before dot-com bust = 1.3 Mean beta of the stocks after dot-com bust
= 1.4 Standard deviation before and after dot-com bust = 0.25. Mean difference in betas before
and after dot-com bust = 0.18 The standard deviation of differences in betas before and after
dot-com bust = 0.2. Based on these results and using a 0.05 significance level, the analyst:
A. can conclude that the dot-com bust has affected the beta of these stocks.
B. cannot conclude that the dot-com bust has affected the beta of these stocks.
C. can conclude that the dot-com bust has not affected the beta of these stocks.
D. none of the above.
135.
An analyst wants to test whether the variance of return from telecom stocks is higher than
0.04. For this purpose, he obtains the following data from a sample of 31 telecom stocks. Mean
return from telecom stocks = 15% Standard deviation of return from telecom stocks = 24%
Mean return from market = 12% Standard deviation of return from market = 13% What is the
critical value of the statistic for this test at a 0.05 level of significance?
A. 16.79.
B. 18.49.
C. 43.73.
D. 46.98.
136.

http://64.106.152.240/inmarkets/passpro/index.cfm?event=questions....

2004-08-02

PASS PRO - Copyright 2004 Inmarkets Ltd - Licensed to Raymond chen

32/150

An analyst wants to test whether the variance of return from oil and gas stocks is higher than
0.045. For this purpose, he obtains the following data from a sample of 60 oil and gas stocks.
Mean return from oil and gas stocks = 16%. Standard deviation of return from oil and gas
stocks = 22% Mean return from the market = 12%. Standard deviation of return from the
market = 14% What is the value of the test statistic for this test?
A. 61.19.
B. 63.46.
C. 64.53.
D. 93.33.
137.
An analyst is studying the impact of the 1990 deregulation on the beta of financial services
sector stocks. From price data for 12 stocks in this sector she has obtained the following
results: Mean beta of the stocks before 1990 deregulation = 1.2 Mean beta of the stocks after
1990 deregulation = 1.4 The standard deviation before and after 1990 deregulation = 0.5.
Mean difference in betas before and after 1990 deregulation = 0.25 Standard deviation of
differences in betas before and after 1990 deregulation = 0.3. Based on these results and using
a 0.01 significance level, the analyst:
A. can conclude that the 1990 deregulation has affected the beta of these stocks.
B. cannot conclude that the 1990 deregulation has affected the beta of these stocks.
C. can conclude that the 1990 deregulation has not affected the beta of these stocks.
D. none of the above.
138.
Which of the following statements are NOT true? I. Type I error occurs when the null hypothesis
is not rejected when it is actually false. II. Type II error occurs when the null hypothesis is
rejected when it is actually true. III. Type I error occurs when the alternate hypothesis is
wrongly accepted. IV. Minimizing the probability of Type II error maximizes the power of the
test.
A. I and II.
B. I and III.
C. II and IV.
D. I, II and IV.
139.
An analyst is using a statistical package to perform a
linear regression between the risk and return from
securities in an emerging market country. The original
data and intermediate statistics are shown on the
right. The value of coefficient of determination for this
regression is CLOSEST to:

http://64.106.152.240/inmarkets/passpro/index.cfm?event=questions....

2004-08-02

PASS PRO - Copyright 2004 Inmarkets Ltd - Licensed to Raymond chen

33/150

A. 0.043.
B. 0.084.
C. 0.916.
D. 0.957.

140.
An analyst has constructed the following t-test for a portfolio of financial securities whose
returns are normally distributed:
Number of securities = 40.
H0: Mean return >= 18 percent.
Significance level = 0.1
What is the rejection point for this test?
A. 1.304.
B. 1.684.
C. 2.021.
D. 2.023.
141.
A fund manager is using a statistical package to
perform a linear regression between the number of
securities that she uses to replicate an index and
the 95 percent shortfall risk. The original data and
intermediate statistics are shown on the right. The
correlation between the price and volume is
CLOSEST to:

A. 0.120.
B.

http://64.106.152.240/inmarkets/passpro/index.cfm?event=questions....

2004-08-02

PASS PRO - Copyright 2004 Inmarkets Ltd - Licensed to Raymond chen

34/150

0.062.
C. 0.880.
D. 0.938.

142.
An analyst is using a statistical package to perform a
linear regression between the risk and return from
securities in an emerging market country. The original
data and intermediate statistics are shown on the
right. The value of the intercept for this regression is
CLOSEST to:

A. -2.97.
B. -1.09.
C. 3.07.
D. 5.62.

143.
A fund manager is using a statistical package to
perform a linear regression between the number of
securities that she uses to replicate an index and
the 95 percent shortfall risk. The original data and
intermediate statistics are shown on the right. The
value of the intercept for this regression is
CLOSEST to:

http://64.106.152.240/inmarkets/passpro/index.cfm?event=questions....

2004-08-02

PASS PRO - Copyright 2004 Inmarkets Ltd - Licensed to Raymond chen

35/150

A. -51.85.
B. 0.
C. 20.51.
D. 38.22.

144.
An analyst is using a statistical package to perform a
linear regression between price of a new semiconductor cleaning product and its price. The original
data and intermediate statistics are shown on the
right. The correlation between the price and volume
is CLOSEST to:

A. 0.038.
B. 0.074.
C. 0.926.
D. 0.962.

145.
An analyst is using a statistical package to perform
a linear regression between the features offered in
a product and its sales volume at a constant price.
The original data and intermediate statistics are
shown on the right. The standard error of the
estimate from this regression is CLOSEST to:

http://64.106.152.240/inmarkets/passpro/index.cfm?event=questions....

2004-08-02

PASS PRO - Copyright 2004 Inmarkets Ltd - Licensed to Raymond chen

36/150

A. 53.
B. 59.
C. 2,852.
D. 3,423.

146.
A fund manager is using a statistical package to
perform a linear regression between the number of
securities that she uses to replicate an index and
the 95 percent shortfall risk. The original data and
intermediate statistics are shown on the right. The
value of the intercept for this regression is
CLOSEST to:

A. -51.85.
B. 0.
C. 20.51.
D. 38.22.

147.
An analyst is using a statistical package to perform a
linear regression between the risk and return from
securities in an emerging market country. The original
data and intermediate statistics are shown on the
right. The correlation between the price and volume is
CLOSEST to:

http://64.106.152.240/inmarkets/passpro/index.cfm?event=questions....

2004-08-02

PASS PRO - Copyright 2004 Inmarkets Ltd - Licensed to Raymond chen

37/150

A. 0.043.
B. 0.084.
C. 0.916.
D. 0.957.

148.
If the correlation coefficient of a linear regression is 0.6, the percentage of variation of the
dependent variable that is not explained by the independent variable is CLOSEST to:
A. 36%.
B. 40%.
C. 60%.
D. 64%.
149.
A fund manager is using a statistical package to
perform a linear regression between the number of
securities that she uses to replicate an index and
the 95 percent shortfall risk. The original data and
intermediate statistics are shown on the right. The
correlation between the price and volume is
CLOSEST to:

A. 0.120.
B. 0.062.
C. 0.880.
D. 0.938.

http://64.106.152.240/inmarkets/passpro/index.cfm?event=questions....

2004-08-02

PASS PRO - Copyright 2004 Inmarkets Ltd - Licensed to Raymond chen

38/150

150.
If the correlation coefficient of a linear regression is 0.75, the percentage of variation of the
dependent variable that is not explained by the independent variable is CLOSEST to:
A. 25%.
B. 44%.
C. 56%.
D. 75%.
151.
If the correlation coefficient of a linear regression is 0.85, the percentage of variation of the
dependent variable that is not explained by the independent variable is CLOSEST to:
A. 15%.
B. 28%.
C. 72%.
D. 85%.
152.
If the correlation coefficient of a linear regression is 0.25, the percentage of variation of the
dependent variable that is not explained by the independent variable is CLOSEST to:
A. 6%.
B. 25%.
C. 75%.

http://64.106.152.240/inmarkets/passpro/index.cfm?event=questions....

2004-08-02

PASS PRO - Copyright 2004 Inmarkets Ltd - Licensed to Raymond chen

39/150

D. 94%.
153.
An analyst is using a statistical package to perform a
linear regression between the risk and return from
securities in an emerging market country. The original
data and intermediate statistics are shown on the
right. The value of the intercept for this regression is
CLOSEST to:

A. -2.97.
B. -1.09.
C. 3.07.
D. 5.62.

154.
If the correlation coefficient of a linear regression is 0.75, the percentage of variation of the
dependent variable that is not explained by the independent variable is CLOSEST to:
A. 25%.
B. 44%.
C. 56%.
D. 75%.
155.
Which of the following test statistics is most appropriate for conducting the hypothesis test
given below?
H0: variance A = variance B; where the test is based on two random independent samples from
two normally distributed populations.
A. t-statistic.
B. z-statistic.
C. F-statistic.
D. chi-square.
156.
An analyst is using a statistical package to perform a
linear regression between price of a new semi-

http://64.106.152.240/inmarkets/passpro/index.cfm?event=questions....

2004-08-02

PASS PRO - Copyright 2004 Inmarkets Ltd - Licensed to Raymond chen

40/150

conductor cleaning product and its price. The original


data and intermediate statistics are shown on the
right. The value of coefficient of determination for
this regression is CLOSEST to:

A. 0.038.
B. 0.074.
C. 0.926.
D. 0.962.

157.
An analyst is using a statistical package to perform a
linear regression between the risk and return from
securities in an emerging market country. The original
data and intermediate statistics are shown on the
right. The standard error of the estimate from this
regression is CLOSEST to:

A. 0.225.
B. 0.247.
C. 0.051.
D. 0.061.

158.
An analyst is using a statistical package to perform a
linear regression between price of a new semiconductor cleaning product and its price. The original
data and intermediate statistics are shown on the
right. The standard error of the estimate from this
regression is CLOSEST to:

http://64.106.152.240/inmarkets/passpro/index.cfm?event=questions....

2004-08-02

PASS PRO - Copyright 2004 Inmarkets Ltd - Licensed to Raymond chen

41/150

A. 55.
B. 60.
C. 3,009.
D. 3,611.

159.
An analyst is using a statistical package to perform a
linear regression between the risk and return from
securities in an emerging market country. The original
data and intermediate statistics are shown on the
right. The value of the slope coefficient for this
regression is CLOSEST to:

A. 0.37.
B. 1.24.
C. 2.09.
D. 7.11.

160.
If the correlation coefficient of a linear regression is 0.25, the percentage of variation of the
dependent variable that is not explained by the independent variable is CLOSEST to:
A. 6%.
B. 25%.
C. 75%.

http://64.106.152.240/inmarkets/passpro/index.cfm?event=questions....

2004-08-02

PASS PRO - Copyright 2004 Inmarkets Ltd - Licensed to Raymond chen

42/150

D. 94%.
161.
An analyst is using a statistical package to perform
a linear regression between the features offered in
a product and its sales volume at a constant price.
The original data and intermediate statistics are
shown on the right. The value of the slope
coefficient for this regression is CLOSEST to:

A. 7.0.
B. 3.6.
C. 58.6.
D. 113.9.

162.
An analyst is using a statistical package to perform a
linear regression between price of a new semiconductor cleaning product and its price. The original
data and intermediate statistics are shown on the
right. The correlation between the price and volume
is CLOSEST to:

A. 0.038.
B. 0.074.
C. 0.926.
D. 0.962.

163.
An analyst is using a statistical package to perform
a linear regression between the features offered in
a product and its sales volume at a constant price.
The original data and intermediate statistics are
shown on the right. The correlation between the
price and volume is CLOSEST to:

http://64.106.152.240/inmarkets/passpro/index.cfm?event=questions....

2004-08-02

PASS PRO - Copyright 2004 Inmarkets Ltd - Licensed to Raymond chen

43/150

A. 0.079.
B. 0.151.
C. 0.849.
D. 0.921.

164.
A portfolio contains two perfectly negatively correlated investments with volatilities of 5 percent
and 7 percent. The proportion of these two securities that would lead to the lowest risk are:
A. 58% and 42%.
B. 42% and 58%.
C. 34% and 66%.
D. 66% and 34%.
165.
If Security A and Security B are positively correlated, and the price of Security A increases, the
price of Security B:
A. will increase.
B. will decrease.
C. is most likely to increase than to decrease.
D. may decrease or remain unchanged, but will not increase.
166.
A portfolio contains two perfectly negatively correlated investments with volatilities of 5 percent
and 7 percent. The proportion of these two securities that would lead to the lowest risk are:
A. 34% and 66%.
B. 66% and 34%.
C. 58% and 42%.
D. 42% and 58%.

http://64.106.152.240/inmarkets/passpro/index.cfm?event=questions....

2004-08-02

PASS PRO - Copyright 2004 Inmarkets Ltd - Licensed to Raymond chen

44/150

167.
A portfolio contains two perfectly negatively correlated investments with volatilities of 5 percent
and 7 percent. The proportion of these two securities that would lead to the lowest risk are:
A. 34% and 66%.
B. 66% and 34%.
C. 58% and 42%.
D. 42% and 58%.
168.
A portfolio contains two perfectly negatively correlated investments with volatilities of 5 percent
and 7 percent. The proportion of these two securities that would lead to the lowest risk are:
A. 58% and 42%.
B. 42% and 58%.
C. 34% and 66%.
D. 66% and 34%.
169.
If Security A and Security B are positively correlated, and the price of Security A increases, the
price of Security B:
A. will increase.
B. will decrease.
C. may increase or decrease, but is most likely to increase.
D. may decrease or remain unchanged, but will not increase.
170.
The lowest level of risk, measured by volatility, of a portfolio containing two perfectly negatively
correlated investments with volatilities of 3 percent and 10 percent respectively is:
A. 0%
B. 3%
C. 10%
D. 13%
171.
The lowest level of risk, measured by volatility, of a portfolio containing two perfectly negatively
correlated investments with volatilities of 5 percent and 7 percent is:
A. 0%.
B. 5%.
C. 7%.
D. 12%.

http://64.106.152.240/inmarkets/passpro/index.cfm?event=questions....

2004-08-02

PASS PRO - Copyright 2004 Inmarkets Ltd - Licensed to Raymond chen

45/150

172.
If Security A and Security B are positively correlated, and the price of Security A increases, the
price of Security B:
A. will increase.
B. will decrease.
C. may increase or decrease, but is most likely to increase.
D. may decrease or remain unchanged, but will not increase.
173.
The variance of the returns from stock A is 0.018 and that of the market is 0.025. If the
covariance between the stock and the index is -0.002, their correlation coefficient is CLOSEST
to:
A. -0.23.
B. -0.11.
C. -0.09.
D. -0.08.
174.
A positive covariance means that:
A. there is no relationship between asset returns.
B. the correlation coefficient is zero.
C. asset returns move in the opposite direction.
D. asset returns move in the same direction.
175.
Which of the following model incorporates both mean reversion and no-arbitrage?
A. Vasicek.
B. Black Scholes.
C. Cox, Ingersoll and Ross.
D. Heath, Jarrow and Morton.
176.
Given two variables X and Y that follow geometric Brownian motion, which of the following
variable will also follow geometric Brownian motion?
A. X * Y.
B. X + Y.
C. log(X) + log(Y).
D. exp(X) + exp(Y).

http://64.106.152.240/inmarkets/passpro/index.cfm?event=questions....

2004-08-02

PASS PRO - Copyright 2004 Inmarkets Ltd - Licensed to Raymond chen

46/150

177.
The standard deviation of a Wiener process over a short period of time would be proportional
to:
A. the time period.
B. square of the time period.
C. square-root of the time period.
D. none of the above.
178.
For which of the following process can the mean change be not equal to zero?
I. Ito process.
II. Wiener process.
III. Generalized Wiener process.
A. I and II.
B. I and III.
C. II and III.
D. I, II and III.
179.
Which of the following model incorporates both mean reversion and no-arbitrage?
A. Vasicek.
B. Ho and Lee.
C. Black Scholes.
D. Hull and White.
180.
The standard deviation of a Wiener process over a short period of time would be proportional
to:
A. the time period.
B. square of the time period.
C. square-root of the time period.
D. none of the above.
181.
Given two variables X and Y that are lognormally distributed, what is the distribution of X * Y?
A. Normal.
B. Lognormal.
C. Exponential.

http://64.106.152.240/inmarkets/passpro/index.cfm?event=questions....

2004-08-02

PASS PRO - Copyright 2004 Inmarkets Ltd - Licensed to Raymond chen

47/150

D. None of the above.


182.
Which of the following are no-arbitrage models for fixed income valuation?
I. Ho and Lee.
II. Hull and White.
III. Cox, Ingersoll and Ross.
IV. Heath, Jarrow and Morton.
A. I and II.
B. III and IV.
C. I, II and IV.
D. II, III and IV.
183.
Which of the following models for interest rates would allow for mean reversion?
A. dr = a * dt + s * dz
B. dr = a * dt - b * dt + s * dz
C. dr = a * (b - r) * dt + s * dz
D. dr = a * (r - b) * dt + s * dz
184.
Vasicek proposed the following model for interest rates:
dr = a * (b - r) * dt + s * dz
What is the long term mean of interest rates in this model?
A. a
B. b
C. r
D. s
185.
Given two variables X and Y that are normally distributed, what of the following will also be
normally distributed?
A. X * Y.
B. X + Y.
C. log(X) + log(Y).
D. exp(X) + exp(Y).
186.
Assuming continuous compounding and an annual percentage rate of 6.5%, the present value

http://64.106.152.240/inmarkets/passpro/index.cfm?event=questions....

2004-08-02

PASS PRO - Copyright 2004 Inmarkets Ltd - Licensed to Raymond chen

48/150

of a cash amount of $8,500 that we will receive after 5 years is CLOSEST to:
A. $6,372.
B. $6,141.
C. $8,022.
D. $8,030.
187.
Assuming continuous compounding and an annual percentage rate of 9%, the present value of
a cash amount of $600 that we will receive after 8 years is CLOSEST to:
A. $292.
B. $301.
C. $323.
D. $349.
188.
Assuming continuous compounding and an annual percentage rate of 9%, the present value of
a cash amount of $1,500 that we will receive after 4 years is CLOSEST to:
A. $1,047.
B. $1,063.
C. $1,103.
D. $1,141.
189.
If a cash flow of $700 in 3 years' time has a PV of $400, the annual percentage rate, assuming
continuous compounding is CLOSEST to:
A. 8.52%.
B. 12.16%.
C. 18.65%.
D. 25.44%.
190.
The price of a stock has risen from $15 to $40 over the past 2 years. If the stock did not pay
any dividends during this period, its continuously compounded return is CLOSEST to:
A. 9.15%
B. 14.94%
C. 29.24%
D. 49.04%
191.

http://64.106.152.240/inmarkets/passpro/index.cfm?event=questions....

2004-08-02

PASS PRO - Copyright 2004 Inmarkets Ltd - Licensed to Raymond chen

49/150

The present value of $4,500 received in 6 month's time, using a continuous discount rate of
9%, is CLOSEST to:
A. $3,290
B. $4,302
C. $4,306
D. $4,310
192.
The price of a stock has risen from $15 to $40 over the past 2 years. If the stock did not pay
any dividends during this period, its continuously compounded return is CLOSEST to:
A. 9.15%
B. 14.94%
C. 29.24%
D. 49.04%
193.
The present value of $500 received in 9 month's time, using a continuous discount rate of 9%,
is CLOSEST to:
A. $467.
B. $468.
C. $469.
D. $470.
194.
If a cash flow of $10,000 in two years' time has a PV of $8,455, the annual percentage rate,
assuming continuous compounding is CLOSEST to:
A. 8.13%.
B. 8.39%.
C. 8.75%.
D. 8.95%.
195.
If a cash flow of $10,000 in two years' time has a PV of $8,455, the annual percentage rate,
assuming continuous compounding is CLOSEST to:
A. 8.13%.
B. 8.39%.
C. 8.75%.
D. 8.95%.

http://64.106.152.240/inmarkets/passpro/index.cfm?event=questions....

2004-08-02

PASS PRO - Copyright 2004 Inmarkets Ltd - Licensed to Raymond chen

50/150

196.
The present value of $4,500 received in 6 month's time, using a continuous discount rate of
9%, is CLOSEST to:
A. $3,290
B. $4,302
C. $4,306
D. $4,310
197.
If a cash flow of $100 in 2 years' time has a PV of $75, the annual percentage rate, assuming
continuous compounding is CLOSEST to:
A. 11.1%.
B. 12.2%.
C. 13.5%.
D. 14.4%.
198.
If a cash flow of $800 in 6 years' time has a PV of $500, the annual percentage rate, assuming
continuous compounding is CLOSEST to:
A. 4.48%.
B. 5.23%.
C. 7.83%.
D. 9.12%.
199.
Assuming continuous compounding and an annual percentage rate of 9%, the present value of
a cash amount of $400 that we will receive after 8 years is CLOSEST to:
A. $195.
B. $201.
C. $233.
D. $252.
200.
Assuming continuous compounding and an annual percentage rate of 6.5%, the present value
of a cash amount of $8,500 that we will receive after 5 years is CLOSEST to:
A. $6,372.
B. $6,141.
C. $8,022.
D. $8,030.

http://64.106.152.240/inmarkets/passpro/index.cfm?event=questions....

2004-08-02

PASS PRO - Copyright 2004 Inmarkets Ltd - Licensed to Raymond chen

51/150

201.
Assuming continuous compounding and an annual percentage rate of 6.5%, the present value
of a cash amount of $8,500 that we will receive after 5 years is CLOSEST to:
A. $6,372.
B. $6,141.
C. $8,022.
D. $8,030.
202.
Given that a stock price rises on six days out of ten and falls on four days out of ten, what is
the probability that it will rise on exactly eight out of the next twelve days?
A. 4.2%.
B. 5.8%.
C. 20.2%.
D. 21.3%.
203.
A sports firm produces two different sets of golf clubs and five types of high-tech golf balls. If
the firm bundles each set of golf club with three different balls, how may unique packages can it
make?
A. 20.
B. 40.
C. 120.
D. 250.
204.
An analyst determines that in the current market environment the S&P500 goes up 7 out of
every 15 days but without any serial dependency, i.e. probability of going up on a certain date
is not affected by its movement on the previous days. Given that 275 trading days in a year the
standard deviation in the number of days that the S&P500 will go up is CLOSEST to:
A. 7.74.
B. 8.27.
C. 59.97.
D. 68.45.
205.
Which of the following is the most appropriate for modeling stock prices?
A. Normal distribution.
B. Random distribution.

http://64.106.152.240/inmarkets/passpro/index.cfm?event=questions....

2004-08-02

PASS PRO - Copyright 2004 Inmarkets Ltd - Licensed to Raymond chen

52/150

C. Bernoulli distribution.
D. Lognormal distribution.
206.
The lognormal distribution is:
A. not skewed.
B. positively skewed.
C. negatively skewed.
D. skewed, but the degree of skew is dependent on the a relative mean of the distribution.
207.
The lognormal distribution is:
A. not skewed.
B. positively skewed.
C. negatively skewed.
D. skewed, but the degree of skew is dependent on the a relative mean of the distribution.
208.
The mean age of the 200 employees in a company is 32 and the standard deviation is 12.
Assuming that the ages are normally distributed and using 95 percent confidence we can say
that the employees within the firm fall between:
A. 8.5 and 55.5 years.
B. 29.7 and 34.3 years.
C. 30.3 and 33.7 years.
D. 31.9 and 32.1 years.
209.
A portfolio has a mean value of $60 million and a daily standard deviation of $8 million.
Assuming that the portfolio values are normally distributed, the lowest value that the portfolio
will fall to over the next five days and within 99% probability is:
A. $4.5 million.
B. $18.4 million.
C. $30.6 million.
D. $42.1 million.
210.
Which of the following are characteristic of a normal distribution?
I. It is bell shaped.
II. It is a continuous distribution.

http://64.106.152.240/inmarkets/passpro/index.cfm?event=questions....

2004-08-02

PASS PRO - Copyright 2004 Inmarkets Ltd - Licensed to Raymond chen

53/150

III. It is symmetrical about the mean.


IV. It peaks at the mean expected value.
A. I and II.
B. I and IV.
C. I, II and III.
D. I, II, III and IV.
211.
What is the quantile corresponding to the 95% confidence interval?
A. -3.715.
B. -2.326.
C. -1.645.
D. -1.282.
212.
The covariance between the return from two securities is 16 and the correlation between them
is 0.5. If the variance of the first return is 8, the variance of the second return will be CLOSEST
to:
A. 4.00.
B. 11.31.
C. 16.00.
D. 128.00.
213.
The mean age of the 80 employees in a company is 35 and the standard deviation is 15.
Assuming that the ages are normally distributed and using 95 percent confidence, we can say
that the employees within the firm fall between:
A. 20.0 and 50.0 years.
B. 31.7 and 38.3 years.
C. 33.8 and 36.2 years.
D. 34.6 and 35.4 years.
214.
The covariance between the return from two securities is 16 and the correlation between them
is 0.5. If the variance of the first return is 8, the variance of the second return will be CLOSEST
to:
A. 4.00.
B. 11.31.
C. 16.00.
D. 128.00.

http://64.106.152.240/inmarkets/passpro/index.cfm?event=questions....

2004-08-02

PASS PRO - Copyright 2004 Inmarkets Ltd - Licensed to Raymond chen

54/150

215.
What is the quantile corresponding to the 99.99% confidence interval?
A. -3.715.
B. -2.326.
C. -1.645.
D. -1.282.
216.
Which of the following are characteristic of a normal distribution?
I. Its kurtosis is equal to zero.
II. Its skewness is less than zero.
III. It is symmetrical about the mean.
IV. It has the same mean, median and mode.
A. I and III.
B. III and IV.
C. I, III and IV.
D. I, II, III and IV.
217.
Consider a portfolio whose expected return is normally distributed with a mean of 20 percent
and a standard deviation of 10 percent. The probability that the return will lie between 0
percent and 10 percent is CLOSEST to:
A. 2%.
B. 6%.
C. 14%.
D. 19%.
218.
A portfolio has a mean value of $70 million and a daily standard deviation of $9.62 million.
Assuming that the portfolio values are normally distributed, the probability of the portfolio value
falling below $20 million within the next five days is CLOSEST to:
A. 0.10%.
B. 1.00%.
C. 5.00%.
D. 15.87%.
219.
The covariance between the return from two securities is 5 and the correlation between them is
0.5. If the variance of the first return is 8, the variance of the second return will be CLOSEST
to:

http://64.106.152.240/inmarkets/passpro/index.cfm?event=questions....

2004-08-02

PASS PRO - Copyright 2004 Inmarkets Ltd - Licensed to Raymond chen

55/150

A. 1.25.
B. 1.56.
C. 3.54.
D. 12.50.
220.
Which of the following is FALSE in relation to the distribution of prices of an asset that is found
to be lognormal?
A. The skewness of the returns distribution will be zero.
B. The mean and median of the price distribution will not be equal.
C. The mean, median and mode of the returns distribution will be equal.
D. If volatility increases but the mean stays constant, the median of price distribution will shift
to the right.
221.
The covariance between the return from two securities is 4.2 and the correlation between them
is 0.6. If the variance of the first return is 5, the variance of the second return will be CLOSEST
to:
A. 1.40.
B. 1.96.
C. 3.13.
D. 9.80.
222.
A portfolio has a mean value of $100 million and a daily standard deviation of $2.5 million.
Assuming that the portfolio values are normally distributed, the lowest value that the portfolio
will fall to over the next 14 days and within 95% probability is:
A. $18.6 million.
B. $42.4 million.
C. $78.3 million.
D. $84.6 million.
223.
A sports firm produces two different sets of golf clubs and five types of high-tech golf balls. If
the firm bundles each set of golf club with three different balls, how may unique packages can it
make?
A. 20.
B. 40.
C. 120.
D. 250.

http://64.106.152.240/inmarkets/passpro/index.cfm?event=questions....

2004-08-02

PASS PRO - Copyright 2004 Inmarkets Ltd - Licensed to Raymond chen

56/150

224.
A sports firm produces three different sets of golf clubs and three types of high-tech golf balls.
If the firm bundles each set of golf club with two different balls, how may unique packages can
it make?
A. 9.
B. 18.
C. 27.
D. 81.
225.
An analyst observes that the closing price of a stock during a week as $33, $43, $45, $48, $46.
On the corresponding days the S&P 500 closed at 1150, 1125, 1140, 1160, 1170. Based on this
data the covariance of the stock with the market is CLOSEST to:
A. 15.8
B. 18.0
C. 37.4
D. 54.4
226.
A sports firm produces two different sets of golf clubs and five types of high-tech golf balls. If
the firm bundles each set of golf club with three different balls, how may unique packages can it
make?
A. 20.
B. 40.
C. 120.
D. 250.
227.
If a loan is being offered at a rate of interest of 14.49 percent compounded monthly, the
borrower will pay:
I. a nominal rate of 13.61%.
II. a periodic rate of 14.49%.
III. a stated rate of 14.49%.
IV. an effective rate of 15.49%.
A. I and II.
B. II and III.
C. III and IV.
D. I, III and IV.
228.

http://64.106.152.240/inmarkets/passpro/index.cfm?event=questions....

2004-08-02

PASS PRO - Copyright 2004 Inmarkets Ltd - Licensed to Raymond chen

57/150

As the discount rate rises, the NPV of a set of cash flows:


A. increases.
B. decreases.
C. stays constant.
D. cannot be determined without further information.
229.
An investor is being promised an annuity of $6,000 starting in five years from now and
continuing for 15 years. Assuming a constant interest rate of 6 percent per annum, the value of
this annuity today is CLOSEST to:
A. $46,158.
B. $58,273.
C. $67,253.
D. $71,288.
230.
An investor is being promised an annuity of $900 starting in two years from now and continuing
for 25 years. Assuming a constant interest rate of 8 percent per annum, the value of this
annuity today is CLOSEST to:
A. $8,895.
B. $9,607.
C. $19,290.
D. $20,833.
231.
Assuming continuous compounding and an annual percentage rate of 7%, the present value of
$4,700 received in nine month's time is CLOSEST to:
A. $3,506.
B. $4,460.
C. $4,466.
D. $4,467.
232.
What is the approximate price of a zero coupon bond whose par value is 100 and term is two
years if its yields is 5%?
A. 86.
B. 91.
C. 96.
D. 101.

http://64.106.152.240/inmarkets/passpro/index.cfm?event=questions....

2004-08-02

PASS PRO - Copyright 2004 Inmarkets Ltd - Licensed to Raymond chen

58/150

233.
Assuming continuous compounding and an annual percentage rate of 8%, the present value of
$7,000 received in eight months' time is CLOSEST to:
A. $6,636.
B. $6,646.
C. $6,650.
D. $6,665.
234.
An investor is being promised an annuity of $2,000 starting in two years from now and
continuing for eight years. Assuming a constant interest rate of 4 percent per annum, the value
of this annuity today is CLOSEST to:
A. $12,947.
B. $13,465.
C. $14,793.
D. $15,385.
235.
The inflation rate in an economy where the prices are doubling ever 15 years is CLOSEST to:
A. 3%.
B. 5%.
C. 7%.
D. 13%.
236.
If all the cash flows in an investment are positive what is its IRR?
A. Positive.
B. Zero.
C. Negative.
D. Cannot be determined.
237.
An investor is being promised an annuity of $6,000 starting in five years from now and
continuing for 15 years. Assuming a constant interest rate of 6 percent per annum, the value of
this annuity today is CLOSEST to:
A. $46,158.
B. $58,273.
C. $67,253.
D. $71,288.

http://64.106.152.240/inmarkets/passpro/index.cfm?event=questions....

2004-08-02

PASS PRO - Copyright 2004 Inmarkets Ltd - Licensed to Raymond chen

59/150

238.
If a cash flow of $15,000 in two years' time has a PV of $12,000, the annual percentage rate,
assuming continuous compounding is CLOSEST to:
A. 11.16%.
B. 11.44%.
C. 11.69%.
D. 11.80%.
239.
Suppose that you need to borrow $1 million for 24 months. Two large US-based international
banks with equal credit ratings offer deposit rates of 2%. To choose between the two banks,
you would need all of the following except:
A. day count basis.
B. compounding basis.
C. currency of deposit.
D. balance sheets of the banks.
240.
An investor is being promised an annuity of $1,500 starting in 15 years from now and
continuing for ten years. Assuming a constant interest rate of 8 percent per annum, the value
of this annuity today is CLOSEST to:
A. $3,427.
B. $4,729.
C. $5,107.
D. $10,065.
241.
Suppose that you need to borrow $1 million for 24 months. Two large US-based international
banks with equal credit ratings offer deposit rates of 2%. To choose between the two banks,
you would need all of the following except:
A. day count basis.
B. compounding basis.
C. currency of deposit.
D. balance sheets of the banks.
242.
A bank is offering loans with monthly interest payments based on a stated annual interest rate
of 7.75 percent. The effective annual rate of these loans is CLOSEST to:
A. 7.90%.

http://64.106.152.240/inmarkets/passpro/index.cfm?event=questions....

2004-08-02

PASS PRO - Copyright 2004 Inmarkets Ltd - Licensed to Raymond chen

60/150

B. 7.98%.
C. 8.00%.
D. 8.03%.
243.
If a cash flow of $10,000 in five years' time has a PV of $7,500, the annual percentage rate,
assuming continuous compounding, is CLOSEST to:
A. 5.65%.
B. 5.75%.
C. 5.85%.
D. 5.92%.
244.
Consider a 4.25 percent semi-annual coupon bond with a par value of $100 and three
remaining coupons, which is trading at a yield of 3.975 percent. There are 45 days remaining in
the current period that has a total of 180 days. The clean price of this bond is CLOSEST to:
A. 99.70.
B. 100.30.
C. 101.89.
D. 102.32.
245.
As the discount rate rises, the NPV of a set of cash flows:
A. increases.
B. decreases.
C. stays constant.
D. cannot be determined without further information.
246.
Assuming continuous compounding and an annual percentage rate of 8%, the present value of
$8,400 received in six months' time is CLOSEST to:
A. $6,203.
B. $8,071.
C. $8,077.
D. $8,083.
247.
A bank is offering loans with monthly interest payments based on a stated annual interest rate
of 7.75 percent. The effective annual rate of these loans is CLOSEST to:

http://64.106.152.240/inmarkets/passpro/index.cfm?event=questions....

2004-08-02

PASS PRO - Copyright 2004 Inmarkets Ltd - Licensed to Raymond chen

61/150

A. 7.90%.
B. 7.98%.
C. 8.00%.
D. 8.03%.
248.
Peter Roche is due to receive $4,000 one year from now, which he can invest at an interest rate
of 7 percent for a further ten years. The value of this payment at the end of this period will be
CLOSEST to:
A. $6,800.
B. $7,080.
C. $7,869.
D. $8,419.
249.
A bank is offering loans with monthly interest payments based on a stated annual interest rate
of 9 percent. The effective annual rate of these loans is CLOSEST to:
A. 9.20%.
B. 9.31%.
C. 9.34%.
D. 9.38%.
250.
If a cash flow of $15,000 in two years' time has a PV of $12,000, the annual percentage rate,
assuming continuous compounding is CLOSEST to:
A. 11.16%.
B. 11.44%.
C. 11.69%.
D. 11.80%.
251.
The inflation rate in an economy where the prices are doubling ever eight years is CLOSEST to:
A. 6%.
B. 9%.
C. 13%.
D. 25%.
252.
What is the standard deviation of the following data?

http://64.106.152.240/inmarkets/passpro/index.cfm?event=questions....

2004-08-02

PASS PRO - Copyright 2004 Inmarkets Ltd - Licensed to Raymond chen

62/150

Probability Value
40% -4
25% 8
35% 15
A. 4.80.
B. 8.32.
C. 23.08.
D. 69.23.
253.
Consider a stock whose 10-day volatility has been estimated as 1.8%. Assuming that the
returns from the stock are uncorrelated over time, the volatility over 250 days will be CLOSEST
to:
A. 0.36%.
B. 9.00%.
C. 28.46%.
D. 45.00%.
254.
Assets A and B have a correlation of +1. The volatility of asset A is 15%. The volatility of asset
B is 25%. The daily return on asset A is observed at 1.50%. What is the most likely return on
asset B?
A. -1.50%.
B. 1.50%.
C. 2.50%.
D. Cannot the predicted.
255.
Which of the following statements about the correlation coefficient are FALSE?
I. Correlation coefficient ranges between -1 and +1.
II. Correlation coefficient is a measure of non-linear relationship between two random variables.
III. Correlation coefficient can be calculated by scaling the covariance between two variables.
IV. If the correlation coefficient between two random variables is zero they will be independent.
A. I and II.
B. I and III.
C. II and IV.
D. I, III and IV.
256.
What is the standard deviation of the following data?

http://64.106.152.240/inmarkets/passpro/index.cfm?event=questions....

2004-08-02

PASS PRO - Copyright 2004 Inmarkets Ltd - Licensed to Raymond chen

63/150

Probability Value
15% -6
40% 6
45% 12
A. 3.51.
B. 6.08.
C. 12.33.
D. 36.98.
257.
What is the mean for the following probability data?
Probability Value
30% -10
40% 5
30% 25
A. 1.15.
B. 2.17.
C. 3.34.
D. 6.50.
258.
Which of the following statements are TRUE?
I. Correlation coefficient ranges between 0 and +1.
II. Correlation coefficient is a measure of non-linear relationship between two random variables.
III. Correlation coefficient can be calculated by scaling the covariance between two variables.
IV. If two random variables are independent they will have a correlation coefficient of zero.
A. I and II.
B. I and IV.
C. II and III.
D. III and IV.
259.
What is the mean for the following probability data?
Probability Value
25% -8
40% 12
35% 24
A. 3.73.
B. 5.73.
C. 11.20.

http://64.106.152.240/inmarkets/passpro/index.cfm?event=questions....

2004-08-02

PASS PRO - Copyright 2004 Inmarkets Ltd - Licensed to Raymond chen

64/150

D. 20.10.
260.
The covariance between the return from two securities is 5 and the correlation between them is
0.5. If the variance of the first return is 8, the variance of the second return will be CLOSEST
to:
A. 1.25.
B. 1.56.
C. 3.54.
D. 12.50.
261.
What is the standard deviation of the following data?
Probability Value
25% -8
40% 12
35% 24
A. 7.07.
B. 12.24.
C. 49.92.
D. 149.76.
262.
Which of the following statements about the correlation coefficient are FALSE?
I. Correlation coefficient ranges between -1 and +1.
II. Correlation coefficient can be calculated by scaling the covariance between two variables.
III. If the correlation coefficient between two random variables is zero they will be independent.
IV. Correlation coefficient is a measure of the non-linear relationship between a dependent and
an independent variable.
A. I and II.
B. I and IV.
C. II and III.
D. III and IV.
263.
What is the standard deviation of the following data?
Probability Value
45% -4
40% 8
15% 25
A. 5.77.

http://64.106.152.240/inmarkets/passpro/index.cfm?event=questions....

2004-08-02

PASS PRO - Copyright 2004 Inmarkets Ltd - Licensed to Raymond chen

65/150

B. 10.00.
C. 33.34.
D. 100.03.
264.
Which of the following statements are TRUE?
I. Correlation coefficient ranges between -1 and +1.
II. Correlation coefficient is a measure of linear relationship between two random variables.
III. Correlation coefficient can be calculated by scaling the covariance between two variables.
IV. If the correlation coefficient between two random variables is zero they will be independent.
A. I and II.
B. III and IV.
C. I, II and III.
D. I, II, III and IV.
265.
What is the mean for the following probability data?
Probability Value
25% -8
40% 12
35% 24
A. 3.73.
B. 5.73.
C. 11.20.
D. 20.10.
266.
What is the standard deviation of the following data?
Probability Value
15% -6
40% 6
45% 12
A. 3.51.
B. 6.08.
C. 12.33.
D. 36.98.
267.
Which of the following statements about the correlation coefficient are FALSE?
I. Correlation coefficient ranges between 0 and +1.

http://64.106.152.240/inmarkets/passpro/index.cfm?event=questions....

2004-08-02

PASS PRO - Copyright 2004 Inmarkets Ltd - Licensed to Raymond chen

66/150

II. If two random variables have a correlation coefficient of zero they are independent.
III. Correlation coefficient can be calculated by scaling the covariance between two variables.
IV. Correlation coefficient is a measure of the linear relationship between a dependent and an
independent variable.
A. I and II.
B. I and IV.
C. II and III.
D. III and IV.
268.
Consider a stock whose 10-day volatility has been estimated as 1.4%. Assuming that the
returns from the stock are uncorrelated over time, the volatility over 23 days will be CLOSEST
to:
A. 0.92%.
B. 2.12%.
C. 3.22%.
D. 6.71%.
269.
Which of the following statements about the correlation coefficient are FALSE?
I. Correlation coefficient ranges between 0 and +1.
II. If two random variables have a correlation coefficient of zero they are independent.
III. Correlation coefficient can be calculated by scaling the covariance between two variables.
IV. Correlation coefficient is a measure of the linear relationship between a dependent and an
independent variable.
A. I and II.
B. I and IV.
C. II and III.
D. III and IV.
270.
Assets A and B have a correlation of +1. The volatility of asset A is 15%. The volatility of asset
B is 25%. The daily return on asset A is observed at 1.50%. What is the most likely return on
asset B?
A. -1.50%.
B. 1.50%.
C. 2.50%.
D. Cannot the predicted.
271.
Consider a stock whose 7-day volatility has been estimated as 1.05%. Assuming that the

http://64.106.152.240/inmarkets/passpro/index.cfm?event=questions....

2004-08-02

PASS PRO - Copyright 2004 Inmarkets Ltd - Licensed to Raymond chen

67/150

returns from the stock are uncorrelated over time, the volatility over 30 days will be CLOSEST
to:
A. 0.51%.
B. 2.17%.
C. 4.50%.
D. 5.75%.
272.
What is the mean for the following probability data?
Probability Value
60% -2
20% 10
20% 20
A. 1.60.
B. 2.80.
C. 3.60.
D. 4.80.
273.
What is the standard deviation of the following data?
Probability Value
40% -4
25% 8
35% 15
A. 4.80.
B. 8.32.
C. 23.08.
D. 69.23.
274.
What is the standard deviation of the following data?
Probability Value
60% -2
20% 10
20% 20
A. 5.14.
B. 8.91.
C. 26.45.
D. 79.36.
275.

http://64.106.152.240/inmarkets/passpro/index.cfm?event=questions....

2004-08-02

PASS PRO - Copyright 2004 Inmarkets Ltd - Licensed to Raymond chen

68/150

Consider a stock whose 7-day volatility has been estimated as 1.05%. Assuming that the
returns from the stock are uncorrelated over time, the volatility over 30 days will be CLOSEST
to:
A. 0.51%.
B. 2.17%.
C. 4.50%.
D. 5.75%.
276.
What is the standard deviation of the following data?
Probability Value
30% -15
40% 5
30% 25
A. 8.94.
B. 15.49.
C. 80.00.
D. 240.00.
277.
Which of the following investments has the highest co-efficient of variation?
A. Expected return = 18%; variance of return = 0.004.
B. Expected return = 25%; variance of return = 0.005.
C. Expected return = 12%; standard deviation of return = 4%.
D. Expected return = 8%; standard deviation of return = 2.5%.
278.
Which of the following investments has the highest co-efficient of variation?
A. An investment with an expected return of 12% and a variance of of return of 0.005.
B. An investment with an expected return of 10% and a variance of of return of 0.002.
C. An investment with an expected return of 15% and standard deviation of return of 3%.
D. An investment with an expected return of 6% and standard deviation of return of 5%.
279.
An analyst regresses the returns of 18 stocks in a stock market and finds that the best fitting
line is:
Return = 3.75% + 6% x Beta
If the standard error of the estimate is 4% and the standard error of the coefficient of Beta is
3%, the test statistic for the coefficient is CLOSEST to:

http://64.106.152.240/inmarkets/passpro/index.cfm?event=questions....

2004-08-02

PASS PRO - Copyright 2004 Inmarkets Ltd - Licensed to Raymond chen

69/150

A. 0.94.
B. 1.43.
C. 1.50.
D. 2.00.
280.
Which of the following investments has the highest co-efficient of variation?
A. An investment with an expected return of 24% and a variance of of return of 0.005.
B. An investment with an expected return of 18% and a variance of of return of 0.002.
C. An investment with an expected return of 15% and standard deviation of return of 5%.
D. An investment with an expected return of 25% and standard deviation of return of 8%.
281.
If the mean P/E of 40 stocks in a certain industrial sector is 12 and the sample standard
deviation is 4, standard error of the mean is CLOSEST to:
A. 0.10.
B. 0.32.
C. 0.63.
D. 1.58.
282.
An analyst regresses the returns of 15 stocks in a stock market and finds that the best fitting
line is:
Return = 13% + 15% x Beta
If the standard error of the estimate is 6% and the standard error of the coefficient of Beta is
9%, the test statistic for the coefficient is CLOSEST to:
A. 1.67.
B. 2.17.
C. 2.38.
D. 2.50.
283.
Which of the following investments has the highest co-efficient of variation?
A. Expected return = 18%; variance of return = 0.004.
B. Expected return = 25%; variance of return = 0.005.
C. Expected return = 12%; standard deviation of return = 4%.
D. Expected return = 8%; standard deviation of return = 2.5%.

http://64.106.152.240/inmarkets/passpro/index.cfm?event=questions....

2004-08-02

PASS PRO - Copyright 2004 Inmarkets Ltd - Licensed to Raymond chen

70/150

284.
An analyst regresses the returns of 50 stocks in a stock market and finds that the best fitting
line is:
Return = 8% + 12.25% x Beta
If the standard error of the estimate is 10% and the standard error of the coefficient of Beta is
8%, the test statistic for the coefficient is CLOSEST to:
A. 0.80.
B. 1.16.
C. 1.23.
D. 1.53.
285.
Which of the following investments has the highest co-efficient of variation?
A. An investment with an expected return of 24% and a variance of of return of 0.005.
B. An investment with an expected return of 18% and a variance of of return of 0.002.
C. An investment with an expected return of 15% and standard deviation of return of 5%.
D. An investment with an expected return of 25% and standard deviation of return of 8%.
286.
If the mean P/E of 30 stocks in a certain industrial sector is 18 and the sample standard
deviation is 3.5, standard error of the mean is CLOSEST to:
A. 0.12.
B. 0.34.
C. 0.64.
D. 1.56.
287.
An analyst regresses the returns of 15 stocks in a stock market and finds that the best fitting
line is:
Return = 13% + 15% x Beta
If the standard error of the estimate is 6% and the standard error of the coefficient of Beta is
9%, the test statistic for the coefficient is CLOSEST to:
A. 1.67.
B. 2.17.
C. 2.38.
D. 2.50.
288.

http://64.106.152.240/inmarkets/passpro/index.cfm?event=questions....

2004-08-02

PASS PRO - Copyright 2004 Inmarkets Ltd - Licensed to Raymond chen

71/150

An analyst regresses the returns of 50 stocks in a stock market and finds that the best fitting
line is:
Return = 8% + 9% x Beta
If the standard error of the estimate is 5.5% and the standard error of the coefficient of Beta is
3.25%, the test statistic for the coefficient is CLOSEST to:
A. 1.45.
B. 1.64.
C. 1.55.
D. 2.77.
289.
An analyst collects the data for interest rate expectations. The mean expected rate is 2.5
percent, the lowest expectation is 1 percent and the highest expectation is 5 percent. This
distribution is:
A. sparse.
B. normal.
C. skewed.
D. abnormal.
290.
An analyst is studying a stock that is currently trading at $35. The analyst estimates that there
is 33 percent probability that the stock will trade at $50 after one year, a 20 percent probability
that the stock will trade at $42, and a 47 percent chance that the stock will trade at $20. What
is the implied volatility of this stock price?
A. 13%.
B. 24%.
C. 31%.
D. 39%.
291.
An analyst regresses the returns of 16 stocks in a stock market and finds that the best fitting
line is:
Return = 6.5% + 11.8% x Beta
If the standard error of the estimate is 4% and the standard error of the coefficient of Beta is
3%, the test statistic for the coefficient is CLOSEST to:
A. 1.63.
B. 2.95.
C. 2.80.
D. 3.93.

http://64.106.152.240/inmarkets/passpro/index.cfm?event=questions....

2004-08-02

PASS PRO - Copyright 2004 Inmarkets Ltd - Licensed to Raymond chen

72/150

292.
If the mean P/E of 60 stocks in a certain industrial sector is 18 and the sample standard
deviation is 7, standard error of the mean is CLOSEST to:
A. 0.12.
B. 0.34.
C. 0.90.
D. 1.11.
293.
If the mean P/E of 50 stocks in a certain industrial sector is ten and the sample standard
deviation is six, standard error of the mean is CLOSEST to:
A. 0.12.
B. 0.35.
C. 0.85.
D. 1.18.
294.
An analyst regresses the returns of 18 stocks in a stock market and finds that the best fitting
line is:
Return = 3.75% + 6% x Beta
If the standard error of the estimate is 4% and the standard error of the coefficient of Beta is
3%, the test statistic for the coefficient is CLOSEST to:
A. 0.94.
B. 1.43.
C. 1.50.
D. 2.00.
295.
Which of the following investment has the highest co-efficient of variation?
A. An investment with an expected return of 19% and a variance of return of 0.005.
B. An investment with an expected return of 15% and a variance of return of 0.002.
C. An investment with an expected return of 10% and standard deviation of return of 3%.
D. An investment with an expected return of 3% and standard deviation of return of 1%.
296.
An analyst regresses the returns of 40 stocks in a stock market and finds that the best fitting
line is:
Return = 6% + 13.6% x beta

http://64.106.152.240/inmarkets/passpro/index.cfm?event=questions....

2004-08-02

PASS PRO - Copyright 2004 Inmarkets Ltd - Licensed to Raymond chen

73/150

If the standard error of the estimate is 3% and the standard error of the coefficient of beta is
5%, the test statistic for the coefficient is CLOSEST to:
A. 2.00.
B. 2.72.
C. 4.31.
D. 4.53.
297.
Which of the following investments has the highest co-efficient of variation?
A. An investment with an expected return of 10% and a variance of of return of 0.002.
B. An investment with an expected return of 15% and a variance of of return of 0.003.
C. An investment with an expected return of 8% and standard deviation of return of 3%.
D. An investment with an expected return of 5% and standard deviation of return of 1%.
298.
An analyst regresses the returns of 40 stocks in a stock market and finds that the best fitting
line is:
Return = 6% + 13.6% x beta
If the standard error of the estimate is 3% and the standard error of the coefficient of beta is
5%, the test statistic for the coefficient is CLOSEST to:
A. 2.00.
B. 2.72.
C. 4.31.
D. 4.53.
299.
Which of the following statements are TRUE? I. Lowering the level of significance reduces the
probability of Type I error. II. Lowering the level of significance increases the probability of
Type I error. III. Minimizing the probability of Type II error minimizes the power of the test. IV.
Minimizing the probability of Type II error maximizes the power of the test.
A. I and III.
B. I and IV.
C. II and III.
D. II and IV.
300.
Which of the following statements is TRUE?
A. Statistical significance does not suggest economic significance.

http://64.106.152.240/inmarkets/passpro/index.cfm?event=questions....

2004-08-02

PASS PRO - Copyright 2004 Inmarkets Ltd - Licensed to Raymond chen

74/150

B. Economic significance and statistical significance are not related.


C. A result that is statistically significant may not be economically significant.
D. Economic significance statistical significance refer to the same evidence.
301.
An analyst wants to test whether the standard deviation of return from pharmaceutical stocks is
lower than 0.2. For this purpose, he obtains the following data from a sample of 30
pharmaceutical stocks. Mean return from pharmaceutical stocks = 8%. Standard deviation of
return from pharmaceutical stocks = 12%. Mean return from the market = 12%. Standard
deviation of return from the market = 16%. Based on this information, we can say that at a
0.05 significance level the standard deviation of pharmaceutical stocks is:
A. lower than 20%.
B. higher than 20%.
C. not lower than 20%.
D. none of the above.
302.
Which of the following statements are TRUE? I. Lowering the level of significance reduces the
probability of Type I error. II. Type I error occurs when the null hypothesis is rejected when it is
actually true. III. Type I error occurs when the null hypothesis is accepted when it is actually
false. IV. Type I error occurs when the alternate hypothesis is wrongly accepted.
A. I and II.
B. I and IV.
C. II and III.
D. I, II and IV.
303.
An analyst is testing a hypothesis that firms with a P/B ratio 25 percent below average tend to
outperform the market. If the test statistic calculated from the market data is more than the
critical value of the statistic, the analyst should:
A. reject the null hypothesis.
B. accept the null hypothesis.
C. reject the results of the hypothesis test.
D. none of the above.
304.
An analyst wants to test whether the return from pharmaceutical stocks is less volatile than
that of the overall market. For this purpose, he obtains the following data from a sample of 21
pharmaceutical stocks and a sample of 41 stocks that are representative of the market. Mean
return from pharmaceutical stocks = 8% Standard deviation of return from pharmaceutical
stocks = 10% Mean return from market stocks = 12% Standard deviation of return from
market stocks = 13% What is the value of the test statistic for this test?
A. 1.30.

http://64.106.152.240/inmarkets/passpro/index.cfm?event=questions....

2004-08-02

PASS PRO - Copyright 2004 Inmarkets Ltd - Licensed to Raymond chen

75/150

B. 1.50.
C. 1.69.
D. 2.25.
305.
An analyst wants to test whether the variance of return from telecom stocks is higher than
0.04. For this purpose, he obtains the following data from a sample of 51 telecom stocks. Mean
return from telecom stocks = 15% Standard deviation of return from telecom stocks = 24%
Mean return from market = 12% Standard deviation of return from market = 13% Based on
this information and a 0.05 significance level:
A. we can say that the variance of telecom firms is lower than 0.04.
B. we can say that the variance of telecom firms is higher than 0.04.
C. we cannot say that the variance of telecom firms is lower than 0.04.
D. none of the above.
306.
Which of the following is NOT a step in the hypothesis testing process?
A. State a hypothesis.
B. Identify the population mean and probability distribution.
C. Specify the significance level.
D. Make the investment/economic decision based on the results of the test.
307.
An analyst wants to test whether the standard deviation of return from pharmaceutical stocks is
lower than 0.2. For this purpose, he obtains the following data from a sample of 30
pharmaceutical stocks. Mean return from pharmaceutical stocks = 8%. Standard deviation of
return from pharmaceutical stocks = 12%. Mean return from the market = 12%. Standard
deviation of return from the market = 16%. What is the critical value of the statistic for this
test at a 0.05 level of significance?
A. 17.71.
B. 28.81.
C. 42.56.
D. 45.72.
308.
An analyst wants to test whether the return from transportation sector stocks is different from
that of the utility stocks. For this purpose, he obtains the following data from a sample of 21
transportation stocks and a sample of 41 utility stocks. Mean return from transportation stocks
= 15%. Standard deviation of return from transportation stocks = 12%. Mean return from
utility stocks = 12%. Standard deviation of return from utility stocks = 13%. Based on this
information and using a 0.05 significance level:
A. there is no evidence for a difference between the means.
B. there is sufficient evidence for a difference between the means.

http://64.106.152.240/inmarkets/passpro/index.cfm?event=questions....

2004-08-02

PASS PRO - Copyright 2004 Inmarkets Ltd - Licensed to Raymond chen

76/150

C. there is insufficient evidence for a difference between the means.


D. none of the above.
309.
An analyst wants to test whether the standard deviation of return from pharmaceutical stocks is
lower than 0.2. For this purpose, he obtains the following data from a sample of 40
pharmaceutical stocks. Mean return from pharmaceutical stocks = 14% Standard deviation of
return from pharmaceutical stocks = 11% Mean return from market = 12% Standard deviation
of return from market = 13% What is the value of the test statistic for this test?
A. 11.80.
B. 12.10.
C. 21.45.
D. 22.00.
310.
Using a sample size of 15 observations, an analyst determines that the standard deviation of
the returns from a stock is 10 percent. Using a 0.05 significance level, the analyst:
A. can conclude that the standard deviation of returns is same as 15%.
B. can conclude that the standard deviation of returns is different from 15%.
C. cannot conclude that the standard deviation of returns is different from 15%.
D. none of the above.
311.
An analyst wants to test whether the variance of return from oil and gas stocks is higher than
0.045. For this purpose, he obtains the following data from a sample of 61 oil and gas stocks.
Mean return from oil and gas stocks = 16%. Standard deviation of return from oil and gas
stocks = 22% Mean return from the market = 12%. Standard deviation of return from the
market = 14% Based on this information, we can say that at a 0.05 significance level the
variance of oil and gas firms is:
A. lower than 0.045.
B. higher than 0.045.
C. not higher than 0.045.
D. none of the above.
312.
An analyst wants to test whether the variance of return from pharmaceutical stocks different
from that of the overall market. For this purpose, he obtains the following data from a sample
of 21 pharmaceutical stocks and a sample of 41 stocks that are representative of the market.
Mean return from pharmaceutical stocks = 8%. Standard deviation of return from
pharmaceutical stocks = 9.2%. Mean return from the market stocks = 12%. Standard deviation
of return from the market stocks = 13%. What is the critical value of the statistic for this test at
a 0.05 level of significance?
A. 1.84.

http://64.106.152.240/inmarkets/passpro/index.cfm?event=questions....

2004-08-02

PASS PRO - Copyright 2004 Inmarkets Ltd - Licensed to Raymond chen

77/150

B. 1.99.
C. 2.07.
D. 2.29.
313.
Which of the following statements are NOT true? I. Type I error occurs when the alternate
hypothesis is wrongly accepted. II. Minimizing the probability of Type II error maximizes the
power of the test. III. Type II error occurs when the null hypothesis is rejected when it is
actually true. IV. Type I error occurs when the null hypothesis is not rejected when it is actually
false.
A. I and II.
B. I and III.
C. III and IV.
D. I, II and IV.
314.
An analyst collects a sample of 50 P/E ratios of stocks that are representative of the market.
The mean P/E of these stocks is 20 and the standard deviation is 8.5. What is the 95 percent
confidence interval for the mean P/E of stocks in this market?
A. 17.21 to 22.79.
B. 17.64 to 22.36.
C. 18.02 to 21.98.
D. 18.31 to 21.69.
315.
An analyst has calculated that the variance of returns of stock A based on 20 observations is
0.005 and the variance of returns of stock B based on 15 observations is 0.012. Using a 0.05
level of significance, the analyst:
A. can conclude that the variance of Stock A is the same as that of Stock B.
B. can conclude that the variance of Stock A is different from that of Stock B.
C. cannot conclude that the variance of Stock A is different from that of Stock B.
D. none of the above.
316.
Which of the following statements are NOT true? I. Type I error occurs when the null hypothesis
is not rejected when it is actually false. II. Type II error occurs when the null hypothesis is
rejected when it is actually true. III. Type I error occurs when the alternate hypothesis is
wrongly accepted. IV. Minimizing the probability of Type II error maximizes the power of the
test.
A. I and II.
B. I and III.
C. II and IV.

http://64.106.152.240/inmarkets/passpro/index.cfm?event=questions....

2004-08-02

PASS PRO - Copyright 2004 Inmarkets Ltd - Licensed to Raymond chen

78/150

D. I, II and IV.
317.
Using a sample size of 101 observations, an analyst determines that the standard deviation of
the returns from a stock is 10 percent. Using a 0.05 significance level, the analyst:
A. can conclude that the standard deviation of returns is lower than 18%.
B. cannot conclude that the standard deviation of returns is lower than 18%.
C. can conclude that the standard deviation of returns is not lower than 18%.
D. none of the above.
318.
An analyst believes that hedge funds have significantly (using a significance level of 0.05)
outperformed the S&P 500 over the past five years. So she picks a random group of 15 hedge
funds and finds that their mean return over this period is 85 percent and their standard
deviation is 45 percent. During the same period the S&P 500 has risen by 75 percent. The
critical value of the t-statistic for this study is:
A. 1.21.
B. 1.65.
C. 1.76.
D. 1.96.
319.
An analyst believes that a group of 16 active portfolio managers have achieved a significantly
higher performance (using a significance level of 0.05) than the average for all portfolio
managers over a certain period. Over the period of the study, the active portfolio managers
achieved a mean return of 15 percent. Over the same period the mean return for all portfolio
managers was 12 percent and their standard deviation was 8 percent. What is the null
hypothesis in this study?
A. The performance of active portfolio managers is higher than or equal to the average for all
portfolio managers.
B. The performance of active portfolio managers is higher than the average for all portfolio
managers.
C. The performance of active portfolio managers is lower than or equal to the average for all
portfolio managers.
D. The performance of active portfolio managers is lower than the average for all portfolio
managers.
320.
An analyst collects a sample of 40 P/E ratios of stocks that are representative of the market.
The mean P/E of these stocks is 18 and the standard error of this estimate 7.25. What is the 95
percent confidence interval for the mean P/E of stocks in this market?
A. 3.79 to 32.21.
B. 6.04 to 29.96.

http://64.106.152.240/inmarkets/passpro/index.cfm?event=questions....

2004-08-02

PASS PRO - Copyright 2004 Inmarkets Ltd - Licensed to Raymond chen

79/150

C. 1.18 to 34.82.
D. 7.78 to 28.22.
321.
An analyst wants to test whether the mean spending by tourists coming to a holiday resort is
greater than $2,000 with a 1 percent level of significance. He finds that the average spending
by 16 tourists is $2,200 and the standard deviation of the population is $400. This study is a:
A. one-tailed test.
B. two-tailed test.
C. four-tailed test.
D. alternate test.
322.
An analyst wants to test whether the return from transportation sector stocks is different from
that of the utility stocks. For this purpose, he obtains the following data from a sample of 21
transportation stocks and a sample of 41 utility stocks. Mean return from transportation stocks
= 9%. Standard deviation of return from transportation stocks = 13%. Mean return from utility
stocks = 12%. Standard deviation of return from utility stocks = 15%. Based on this
information and using a 0.05 significance level:
A. there is no evidence for a difference between the means.
B. there is sufficient evidence for a difference between the means.
C. there is insufficient evidence for a difference between the means.
D. none of the above.
323.
An analyst believes that a group of 16 active portfolio managers have achieved a significantly
higher performance (using a significance level of 0.05) than the average for all portfolio
managers over a certain period. Over the period of the study, the active portfolio managers
achieved a mean return of 15 percent. Over the same period the mean return for all portfolio
managers was 12 percent and their standard deviation was 8 percent. What is the null
hypothesis in this study?
A. The performance of active portfolio managers is higher than or equal to the average for all
portfolio managers.
B. The performance of active portfolio managers is higher than the average for all portfolio
managers.
C. The performance of active portfolio managers is lower than or equal to the average for all
portfolio managers.
D. The performance of active portfolio managers is lower than the average for all portfolio
managers.
324.
A fund manager is using a statistical package to
perform a linear regression between the number of
securities that she uses to replicate an index and
the 95 percent shortfall risk. The original data and

http://64.106.152.240/inmarkets/passpro/index.cfm?event=questions....

2004-08-02

PASS PRO - Copyright 2004 Inmarkets Ltd - Licensed to Raymond chen

80/150

intermediate statistics are shown on the right. The


value of coefficient of determination for this
regression is CLOSEST to:

A. 0.062.
B. 0.120.
C. 0.880.
D. 0.938.

325.
An analyst is using a statistical package to perform
a linear regression between the features offered in
a product and its sales volume at a constant price.
The original data and intermediate statistics are
shown on the right. The standard error of the
estimate from this regression is CLOSEST to:

A. 53.
B. 59.
C. 2,852.
D. 3,423.

326.
If the correlation coefficient of a linear regression is 0.6, the percentage of variation of the
dependent variable that is not explained by the independent variable is CLOSEST to:
A. 36%.
B. 40%.
C. 60%.

http://64.106.152.240/inmarkets/passpro/index.cfm?event=questions....

2004-08-02

PASS PRO - Copyright 2004 Inmarkets Ltd - Licensed to Raymond chen

81/150

D. 64%.
327.
If the correlation coefficient of a linear regression is 0.85, the percentage of variation of the
dependent variable that is not explained by the independent variable is CLOSEST to:
A. 15%.
B. 28%.
C. 72%.
D. 85%.
328.
An analyst is using a statistical package to perform a
linear regression between price of a new semiconductor cleaning product and its price. The original
data and intermediate statistics are shown on the
right. The value of the slope coefficient for this
regression is CLOSEST to:

A. -24.81.
B. -14.59.
C. 24.83.
D. 42.21.

329.
If the correlation coefficient of a linear regression is 0.33, the percentage of variation of the
dependent variable that is not explained by the independent variable is CLOSEST to:
A. 11%.
B. 33%.
C. 67%.
D. 89%.
330.
A fund manager is using a statistical package to
perform a linear regression between the number of
securities that she uses to replicate an index and
the 95 percent shortfall risk. The original data and
intermediate statistics are shown on the right. The
value of the slope coefficient for this regression is

http://64.106.152.240/inmarkets/passpro/index.cfm?event=questions....

2004-08-02

PASS PRO - Copyright 2004 Inmarkets Ltd - Licensed to Raymond chen

82/150

CLOSEST to:

A. -8.51.
B. -2.47.
C. 3.68.
D. 5.71.

331.
An analyst is using a statistical package to perform
a linear regression between the features offered in
a product and its sales volume at a constant price.
The original data and intermediate statistics are
shown on the right. The correlation between the
price and volume is CLOSEST to:

A. 0.079.
B. 0.151.
C. 0.849.
D. 0.921.

332.
A fund manager is using a statistical package to
perform a linear regression between the number of
securities that she uses to replicate an index and
the 95 percent shortfall risk. The original data and
intermediate statistics are shown on the right. The
value of coefficient of determination for this
regression is CLOSEST to:

http://64.106.152.240/inmarkets/passpro/index.cfm?event=questions....

2004-08-02

PASS PRO - Copyright 2004 Inmarkets Ltd - Licensed to Raymond chen

83/150

A. 0.062.
B. 0.120.
C. 0.880.
D. 0.938.

333.
An analyst is using a statistical package to perform
a linear regression between the features offered in
a product and its sales volume at a constant price.
The original data and intermediate statistics are
shown on the right. The value of coefficient of
determination for this regression is CLOSEST to:

A. 0.079.
B. 0.151.
C. 0.849.
D. 0.921.

334.
A fund manager is using a statistical package to
perform a linear regression between the number of
securities that she uses to replicate an index and
the 95 percent shortfall risk. The original data and
intermediate statistics are shown on the right. The
standard error of the estimate from this regression
is CLOSEST to:

http://64.106.152.240/inmarkets/passpro/index.cfm?event=questions....

2004-08-02

PASS PRO - Copyright 2004 Inmarkets Ltd - Licensed to Raymond chen

84/150

A. 4.
B. 17.
C. 21.
D. 26.

335.
A fund manager is using a statistical package to
perform a linear regression between the number of
securities that she uses to replicate an index and
the 95 percent shortfall risk. The original data and
intermediate statistics are shown on the right. The
standard error of the estimate from this regression
is CLOSEST to:

A. 4.
B. 17.
C. 21.
D. 26.

336.
An analyst is using a statistical package to perform a
linear regression between the risk and return from
securities in an emerging market country. The original
data and intermediate statistics are shown on the
right. The correlation between the price and volume is
CLOSEST to:

http://64.106.152.240/inmarkets/passpro/index.cfm?event=questions....

2004-08-02

PASS PRO - Copyright 2004 Inmarkets Ltd - Licensed to Raymond chen

85/150

A. 0.043.
B. 0.084.
C. 0.916.
D. 0.957.

337.
An analyst is using a statistical package to perform a
linear regression between price of a new semiconductor cleaning product and its price. The original
data and intermediate statistics are shown on the
right. The value of the intercept for this regression is
CLOSEST to:

A. 0.
B. 258.51.
C. 843.96.
D. 995.77.

338.
If the average return of 60 stocks in a certain industrial sector is 25 percent and the sample
standard deviation is 15 percent, standard error of the mean is CLOSEST to:
A. 1.76.
B. 1.82.
C. 1.88.

http://64.106.152.240/inmarkets/passpro/index.cfm?event=questions....

2004-08-02

PASS PRO - Copyright 2004 Inmarkets Ltd - Licensed to Raymond chen

86/150

D. 1.94.
339.
An analyst is using a statistical package to perform a
linear regression between the risk and return from
securities in an emerging market country. The original
data and intermediate statistics are shown on the
right. The value of coefficient of determination for this
regression is CLOSEST to:

A. 0.043.
B. 0.084.
C. 0.916.
D. 0.957.

340.
An analyst is using a statistical package to perform a
linear regression between price of a new semiconductor cleaning product and its price. The original
data and intermediate statistics are shown on the
right. The standard error of the estimate from this
regression is CLOSEST to:

A. 55.
B. 60.
C. 3,009.
D. 3,611.

341.
If the average return of 60 stocks in a certain industrial sector is 25 percent and the sample
standard deviation is 15 percent, standard error of the mean is CLOSEST to:
A. 1.76.
B. 1.82.

http://64.106.152.240/inmarkets/passpro/index.cfm?event=questions....

2004-08-02

PASS PRO - Copyright 2004 Inmarkets Ltd - Licensed to Raymond chen

87/150

C. 1.88.
D. 1.94.
342.
An analyst is using a statistical package to perform a
linear regression between the risk and return from
securities in an emerging market country. The original
data and intermediate statistics are shown on the
right. The standard error of the estimate from this
regression is CLOSEST to:

A. 0.225.
B. 0.247.
C. 0.051.
D. 0.061.

343.
If the correlation coefficient of a linear regression is 0.33, the percentage of variation of the
dependent variable that is not explained by the independent variable is CLOSEST to:
A. 11%.
B. 33%.
C. 67%.
D. 89%.
344.
An analyst is using a statistical package to perform a
linear regression between the risk and return from
securities in an emerging market country. The original
data and intermediate statistics are shown on the
right. The value of the slope coefficient for this
regression is CLOSEST to:

A. 0.37.
B. 1.24.

http://64.106.152.240/inmarkets/passpro/index.cfm?event=questions....

2004-08-02

PASS PRO - Copyright 2004 Inmarkets Ltd - Licensed to Raymond chen

88/150

C. 2.09.
D. 7.11.

345.
An analyst is using a statistical package to perform a
linear regression between price of a new semiconductor cleaning product and its price. The original
data and intermediate statistics are shown on the
right. The value of the intercept for this regression is
CLOSEST to:

A. 0.
B. 258.51.
C. 843.96.
D. 995.77.

346.
A fund manager is using a statistical package to
perform a linear regression between the number of
securities that she uses to replicate an index and
the 95 percent shortfall risk. The original data and
intermediate statistics are shown on the right. The
value of the slope coefficient for this regression is
CLOSEST to:

http://64.106.152.240/inmarkets/passpro/index.cfm?event=questions....

2004-08-02

PASS PRO - Copyright 2004 Inmarkets Ltd - Licensed to Raymond chen

89/150

A. -8.51.
B. -2.47.
C. 3.68.
D. 5.71.

347.
An analyst is using a statistical package to perform
a linear regression between the features offered in
a product and its sales volume at a constant price.
The original data and intermediate statistics are
shown on the right. The value of the intercept for
this regression is CLOSEST to:

A. 0.
B. 221.
C. 441.
D. 690.

348.
Which of the following test statistics is most appropriate for conducting the hypothesis test
given below?
H0: variance A = variance B; where the test is based on two random independent samples from
two normally distributed populations.
A. t-statistic.
B. z-statistic.

http://64.106.152.240/inmarkets/passpro/index.cfm?event=questions....

2004-08-02

PASS PRO - Copyright 2004 Inmarkets Ltd - Licensed to Raymond chen

90/150

C. F-statistic.
D. chi-square.
349.
Stock A has a standard deviation of 0.30 and Stock B has a standard deviation of 0.45. If the
covariance is 0.1013 what is the correlation coefficient?
A. 0.01.
B. 0.23.
C. 0.34.
D. 0.75.
350.
The variance of the returns from stock A is 0.018 and that of the market is 0.025. If the
covariance between the stock and the index is -0.002, their correlation coefficient is CLOSEST
to:
A. -0.23.
B. -0.11.
C. -0.09.
D. -0.08.
351.
Stock A has a standard deviation of 0.30 and Stock B has a standard deviation of 0.45. If their
correlation coefficient is 0.75 what is their covariance?
A. 0.1013.
B. 0.1350.
C. 0.2250.
D. 0.3375.
352.
A portfolio contains two perfectly negatively correlated investments with volatilities of 3 percent
and 10 percent. The proportion of these two securities that would lead to the lowest risk are:
A. 8% and 92%.
B. 92% and 8%.
C. 23% and 77%.
D. 77% and 23%.
353.
A portfolio contains two perfectly negatively correlated investments with volatilities of 3 percent
and 10 percent. The proportion of these two securities that would lead to the lowest risk are:

http://64.106.152.240/inmarkets/passpro/index.cfm?event=questions....

2004-08-02

PASS PRO - Copyright 2004 Inmarkets Ltd - Licensed to Raymond chen

91/150

A. 8% and 92%.
B. 92% and 8%.
C. 23% and 77%.
D. 77% and 23%.
354.
Stock A has a standard deviation of 0.30 and Stock B has a standard deviation of 0.45. If their
correlation coefficient is 0.75 what is their covariance?
A. 0.1013.
B. 0.1350.
C. 0.2250.
D. 0.3375.
355.
The lowest level of risk, measured by volatility, of a portfolio containing two perfectly negatively
correlated investments with volatilities of 5 percent and 7 percent is:
A. 0%.
B. 5%.
C. 7%.
D. 12%.
356.
If Security A and Security B are positively correlated, and the price of Security A increases, the
price of Security B:
A. will increase.
B. will decrease.
C. is most likely to increase than to decrease.
D. may decrease or remain unchanged, but will not increase.
357.
The lowest level of risk, measured by volatility, of a portfolio containing two perfectly negatively
correlated investments with volatilities of 3 percent and 10 percent respectively is:
A. 0%
B. 3%
C. 10%
D. 13%
358.
Stock A has a standard deviation of 0.30 and Stock B has a standard deviation of 0.45. If the

http://64.106.152.240/inmarkets/passpro/index.cfm?event=questions....

2004-08-02

PASS PRO - Copyright 2004 Inmarkets Ltd - Licensed to Raymond chen

92/150

covariance is 0.1013 what is the correlation coefficient?


A. 0.01.
B. 0.23.
C. 0.34.
D. 0.75.
359.
A positive covariance means that:
A. there is no relationship between asset returns.
B. the correlation coefficient is zero.
C. asset returns move in the opposite direction.
D. asset returns move in the same direction.
360.
Which of the following are no-arbitrage models for fixed income valuation?
I. Vasicek.
II. Black Scholes.
III. Hull and White.
IV. Heath, Jarrow and Morton.
A. I and II.
B. III and IV.
C. I, III and IV.
D. II, III and IV.
361.
Which of the following are no-arbitrage models for fixed income valuation?
I. Vasicek.
II. Black Scholes.
III. Hull and White.
IV. Heath, Jarrow and Morton.
A. I and II.
B. III and IV.
C. I, III and IV.
D. II, III and IV.
362.
Given two variables X and Y that are normally distributed, what of the following will also be
normally distributed?
A. X * Y.

http://64.106.152.240/inmarkets/passpro/index.cfm?event=questions....

2004-08-02

PASS PRO - Copyright 2004 Inmarkets Ltd - Licensed to Raymond chen

93/150

B. X + Y.
C. log(X) + log(Y).
D. exp(X) + exp(Y).
363.
Which of the following model incorporates both mean reversion and no-arbitrage?
A. Vasicek.
B. Black Scholes.
C. Cox, Ingersoll and Ross.
D. Heath, Jarrow and Morton.
364.
Given two variables X and Y that follow geometric Brownian motion, which of the following
variable will also follow geometric Brownian motion?
A. X * Y.
B. X + Y.
C. log(X) + log(Y).
D. exp(X) + exp(Y).
365.
Which of the following are no-arbitrage models for fixed income valuation?
I. Ho and Lee.
II. Hull and White.
III. Cox, Ingersoll and Ross.
IV. Heath, Jarrow and Morton.
A. I and II.
B. III and IV.
C. I, II and IV.
D. II, III and IV.
366.
Vasicek proposed the following model for interest rates:
dr = a * (b - r) * dt + s * dz
What is the long term mean of interest rates in this model?
A. a
B. b
C. r
D. s

http://64.106.152.240/inmarkets/passpro/index.cfm?event=questions....

2004-08-02

PASS PRO - Copyright 2004 Inmarkets Ltd - Licensed to Raymond chen

94/150

367.
Which of the following models for interest rates would allow for mean reversion?
A. dr = a * dt + s * dz
B. dr = a * dt - b * dt + s * dz
C. dr = a * (b - r) * dt + s * dz
D. dr = a * (r - b) * dt + s * dz
368.
Which of the following model incorporates both mean reversion and no-arbitrage?
A. Vasicek.
B. Ho and Lee.
C. Black Scholes.
D. Hull and White.
369.
For which of the following process can the mean change be not equal to zero?
I. Ito process.
II. Wiener process.
III. Generalized Wiener process.
A. I and II.
B. I and III.
C. II and III.
D. I, II and III.
370.
Given two variables X and Y that are lognormally distributed, what is the distribution of X * Y?
A. Normal.
B. Lognormal.
C. Exponential.
D. None of the above.

http://64.106.152.240/inmarkets/passpro/index.cfm?event=questions....

2004-08-02

PASS PRO - Copyright 2004 Inmarkets Ltd - Licensed to Raymond chen

95/150

PASS PRO Solutions


1. Correct answer: B
Continuously compounded rate = ln(FV/PV)/N = ln(10000 / 8455) / 2 = 8.39%.
Study Session: 1 - RA: 1
2. Correct answer: B
Using continuous compounding, PV = FV x exp(-Rate x Time period) = $700 x exp (- 9% x
9/12) = $654.
Study Session: 1 - RA: 1
3. Correct answer: D
Continuously compounded rate = ln(FV/PV)/N = ln(100 / 75) / 2 = 14.38%.
Study Session: 1 - RA: 1
4. Correct answer: D
Continuously compounded rate = ln(FV/PV)/N = ln(100 / 75) / 2 = 14.38%.
Study Session: 1 - RA: 1
5. Correct answer: D
Continuously compounded rate = ln(final value/initial value) / Number of periods = ln(70 /
60) / 4 = 3.85%.
Study Session: 1 - RA: 1
6. Correct answer: A
Continuously compounding return = ln(final value/initial value) / Number of periods = ln
(35/13) / 5 = 20%.
Study Session: 1 - RA: 1
7. Correct answer: C
Using continuous compounding, PV = FV x exp(-Rate x Time period) = $800 x exp (- 20% x
12/12) = $655.

http://64.106.152.240/inmarkets/passpro/index.cfm?event=questions....

2004-08-02

PASS PRO - Copyright 2004 Inmarkets Ltd - Licensed to Raymond chen

96/150

Study Session: 1 - RA: 1


8. Correct answer: D
Continuously compounded rate = ln(final value/initial value) / Number of periods = ln(70 /
60) / 4 = 3.85%.
Study Session: 1 - RA: 1
9. Correct answer: A
Continuously compounding return = ln(final value/initial value) / Number of periods = ln
(35/13) / 5 = 20%.
Study Session: 1 - RA: 1
10. Correct answer: B
Continuously compounded rate = ln(final value/initial value) / Number of periods = ln(45 /
30) / 5 = 8.11%.
Study Session: 1 - RA: 1
11. Correct answer: B
Using continuous compounding, PV = FV x exp(-Rate x Time period) = $4,500 x exp (- 9% x
6/12) = $4,302.
Study Session: 1 - RA: 1
12. Correct answer: C
Continuously compounded rate = ln(FV/PV)/N = ln(700 / 400) / 3 = 18.65%.
Study Session: 1 - RA: 1
13. Correct answer: C
Using continuous compounding, PV = FV x exp(-Rate x Time period) = $800 x exp (- 20% x
12/12) = $655.
Study Session: 1 - RA: 1
14. Correct answer: D
Continuously compounded rate = ln(final value/initial value) / Number of periods = ln(40 /
15) / 2 = 49.04%.
Study Session: 1 - RA: 1
15. Correct answer: C
Continuously compounded rate = ln(FV/PV)/N = ln(800 / 500) / 6 = 7.83%.
Study Session: 1 - RA: 1
16. Correct answer: B

http://64.106.152.240/inmarkets/passpro/index.cfm?event=questions....

2004-08-02

PASS PRO - Copyright 2004 Inmarkets Ltd - Licensed to Raymond chen

97/150

Using continuous compounding, PV = FV x exp(-Rate x Time period) = $700 x exp (- 9% x


9/12) = $654.
Study Session: 1 - RA: 1
17. Correct answer: A
This problem requires the use of binomial probability function, P(X=x) = fact(n) / fact(n - x) /
fact(x) * p^x * (1-p)^(n-x) = fact(8) / fact(8 - 7) / fact(7) * 0.4^7 * 0.6^1 = 0.8%.
Study Session: 1 - RA: 2
18. Correct answer: A
Given that the daily standard deviation is $4.27 million, the standard deviation over 7 days =
$4.27 million x (7/1)^0.5 = $11.29 million.
Using this standard deviation, the level of $40 million is 3.1 standard deviations [= (75 40)/11.29] from the mean value.
Given that the returns are normally distributed the probability of value falling more than 3.1
standard deviations from the mean value is 0.1% (since 100% of the probability falls between
3.1 standard deviations of the mean).
Study Session: 1 - RA: 2
19. Correct answer: C
The tail ends of a normal distribution stretch to infinity, although the area under these tails
becomes insignificant past 3 standard deviation from mean.
Study Session: 1 - RA: 2
20. Correct answer: D
Covariance = Correlation x Standard Deviation_A x Standard Deviation_B. Therefore Standard
Deviation_B
= Covariance / Correlation / Standard Deviation_A
= 4 / 0.5 / 16^0.5 = 2.
Therefore Variance_B = 2^2 = 4.
Study Session: 1 - RA: 2
21. Correct answer: D
The characteristics of a normal distribution are:
It is a continuous distribution.
It is bell shaped.
It is symmetrical about the mean.
It peaks at the mean expected value.
It extends theoretically from negative infinity to positive infinity (the probability asymptotically
approaches zero at plus and minus infinity).
It has a skewness of zero (i.e. it is symmetric).
It has a kurtosis (the level of peakedness) of three. Below three the distribution is platykurtic

http://64.106.152.240/inmarkets/passpro/index.cfm?event=questions....

2004-08-02

PASS PRO - Copyright 2004 Inmarkets Ltd - Licensed to Raymond chen

98/150

(too flat) and above three it is leptokurtic (too tall).


Study Session: 1 - RA: 2
22. Correct answer: C
Confidence interval = 1 - CND
(Quantile) = Area under the normal
distribution to the right of the given
Quantile point. Some of these points
are rather important due to their use
in VAR calculations.
Confidence Level 90% 95% 99%
Quantile (alpha) -1.282 -1.645 -2.326
Note: CND = Cumulative Normal
Distribution function.
Study Session: 1 - RA: 2
23. Correct answer: C
A return of 0 percent and 10 percent is between -2 standard deviation and -1 standard
deviation of the mean, i.e. a quartile of -2 to -1. The confidence levels corresponding to these
quartiles are 97.7% and 84.1%. Hence the probability that the return falls between these limits
= 97.7% - 84.1% = 13.6%
Study Session: 1 - RA: 2
24. Correct answer: A
Standard deviation = 100^0.5 = 10. Number of SD that 77 is away from mean = (77-100)/10
= -2.3. SD of 2.3 has a one-sided probability of 1.00%. Note: you can eliminate other
alternatives as being too high without solving the problem.
Study Session: 1 - RA: 2
25. Correct answer: C
The movement of S&P500 as described by the analyst is a binomial process, i.e. the S&P500
can go up with a probability of 0.375 or go down with a probability of 0.625. Carrying out this
process for 285 days the variance = n x p x (1 - p) = 285 x 0.375 x 0.625 = 66.797. Therefore,
the standard deviation = 66.797^0.5 = 8.173.
Study Session: 1 - RA: 2
26. Correct answer: A
Given that the daily standard deviation is $9 million, the standard deviation over 10 days = $9
million x (10/1)^0.5 = $28.46 million.
Given that the returns are normally distributed, we know that 99.9% of the outcomes will be
above 3.1 standard deviations below the mean, i.e. above $1.8 million.
Study Session: 1 - RA: 2

http://64.106.152.240/inmarkets/passpro/index.cfm?event=questions....

2004-08-02

PASS PRO - Copyright 2004 Inmarkets Ltd - Licensed to Raymond chen

99/150

27. Correct answer: D


Covariance = Correlation x Standard Deviation_A x Standard Deviation_B. Therefore Standard
Deviation_B
= Covariance / Correlation / Standard Deviation_A
= 4.2 / 0.6 / 5^0.5 = 3.1305.
Therefore, Variance_B = 3.1305^2 = 9.80.
Study Session: 1 - RA: 2
28. Correct answer: B
Confidence interval = 1 - CND
(Quantile) = Area under the normal
distribution to the right of the given
Quantile point. Some of these point
are rather important due to their use
in VAR calculations.
Confidence Level 90% 95% 99%
Quantile (alpha) 1.282 1.645 2.326
Note: CND = Cumulative Normal
Distribution function
Study Session: 1 - RA: 2
29. Correct answer: D
The characteristics of a normal distribution are:
It is a continuous distribution.
It is bell shaped.
It is symmetrical about the mean.
It peaks at the mean expected value.
It extends theoretically from negative infinity to positive infinity (the probability asymptotically
approaches zero at plus and minus infinity).
It has a skewness of zero (i.e. it is symmetric).
It has a kurtosis (the level of peakedness) of three. Below three the distribution is platykurtic
(too flat) and above three it is leptokurtic (too tall).
Study Session: 1 - RA: 2
30. Correct answer: B
The range of the 95 percent confidence interval = mean 1.96 x standard deviation / Number
of employees^0.5 = 35 1.96 x 15 / 80^0.5 = 35 3.3 = between 31.7 and 38.3
Study Session: 1 - RA: 2
31. Correct answer: D
Lognormal distribution is most appropriate for stock prices, since it does not allow for negative
values and implies that the returns are normally distributed.
Study Session: 1 - RA: 2

http://64.106.152.240/inmarkets/passpro/index.cfm?event=questions....

2004-08-02

PASS PRO - Copyright 2004 Inmarkets Ltd - Licensed to Raymond chen

100/150

32. Correct answer: B


The characteristics of a normal distribution are:
It is a continuous distribution.
It is bell shaped.
It is symmetrical about the mean.
It peaks at the mean expected value.
It extends theoretically from negative infinity to positive infinity (the probability asymptotically
approaches zero at plus and minus infinity).
It has a skewness of zero (i.e. it is symmetric).
It has a kurtosis (the level of peakedness) of three. Below three the distribution is platykurtic
(too flat) and above three it is leptokurtic (too tall).
Study Session: 1 - RA: 2
33. Correct answer: A
This problem requires the use of binomial probability function, P(X=x) = fact(n) / fact(n - x) /
fact(x) * p^x * (1-p)^(n-x) = fact(8) / fact(8 - 7) / fact(7) * 0.4^7 * 0.6^1 = 0.8%.
Study Session: 1 - RA: 2
34. Correct answer: D
Covariance = Correlation x Standard Deviation_A x Standard Deviation_B. Therefore Standard
Deviation_B
= Covariance / Correlation / Standard Deviation_A
= 4 / 0.5 / 16^0.5 = 2.
Therefore Variance_B = 2^2 = 4.
Study Session: 1 - RA: 2
35. Correct answer: D
Covariance = Correlation x Standard Deviation_A x Standard Deviation_B. Therefore Standard
Deviation_B
= Covariance / Correlation / Standard Deviation_A
= 15 / 0.5 / 25^0.5 = 6.
Therefore Variance_B = 6^2 = 36.
Study Session: 1 - RA: 2
36. Correct answer: A
The movement of S&P500 as described by the analyst is a binomial process, i.e. the S&P500
can go up with a probability of 0.714 or go down with a probability of 0.286.
Carrying out this process for 280 days the variance = n x p x (1 - p) = 280 x 0.714 x 0.286 =
57.177. Therefore the standard deviation = 57.177^0.5 = 7.562.
Study Session: 1 - RA: 2
37. Correct answer: C

http://64.106.152.240/inmarkets/passpro/index.cfm?event=questions....

2004-08-02

PASS PRO - Copyright 2004 Inmarkets Ltd - Licensed to Raymond chen

101/150

The tail ends of a normal distribution stretch to infinity, although the area under these tails
becomes insignificant past 3 standard deviation from mean.
Study Session: 1 - RA: 2
38. Correct answer: C
Given that the daily standard deviation is $1.5 million, the standard deviation over 250 days =
$1.5 million x (250/1)^0.5 = $23.72 million.
Given that the returns are normally distributed, we know that 95% of the outcomes will be
above 1.645 standard deviations below the mean, i.e. above $101 million.
Study Session: 1 - RA: 2
39. Correct answer: D
Lognormal distribution is most appropriate for stock prices, since it does not allow for negative
values and implies that the returns are normally distributed.
Study Session: 1 - RA: 2
40. Correct answer: C
Given that the daily standard deviation is $19 million, the standard deviation over 5 days = $19
million x (5/1)^0.5 = $42.49 million.
Given that the returns are normally distributed, we know that 95% of the outcomes will be
above 1.645 standard deviations below the mean, i.e. above $30.1 million.
Study Session: 1 - RA: 2
41. Correct answer: D
Covariance = Correlation x Standard Deviation_A x Standard Deviation_B. Therefore Standard
Deviation_B
= Covariance / Correlation / Standard Deviation_A
= 22 / 0.8 / 25^0.5 = 5.5.
Therefore Variance_B = 5.5^2 = 30.25.
Study Session: 1 - RA: 2
42. Correct answer: B
FV of this payment = $6,000 x (1 + 4%)^4 = $7,019.
Study Session: 1 - RA: 3
43. Correct answer: B
Using continuous compounding, PV = FV x exp(-Rate x Time period) = $8,400 x exp (- 8% x
6/12) = $8,071.
Study Session: 1 - RA: 3
44. Correct answer: A

http://64.106.152.240/inmarkets/passpro/index.cfm?event=questions....

2004-08-02

PASS PRO - Copyright 2004 Inmarkets Ltd - Licensed to Raymond chen

102/150

Continuously compounded rate = ln(FV/PV)/N = ln(21000 / 16000) / 2 = 13.6%.


Study Session: 1 - RA: 3
45. Correct answer: A
The value of this annuity at the start of the year 8 = Coupon x [1 - 1/(1 + i)^N]/i = 4,000 x [1
- 1/(1 + 5.25%)^10] / 5.25% = $30,515.
To find the value of this amount in today's money, we need to discount it back 7 years. Thus,
present value of this annuity = $30,515 / (1 + 5.25%)^7 = $21,328.
Study Session: 1 - RA: 3
46. Correct answer: C
The mean of this set is 6. Therefore, the variance is [(6-2)^2 + (6-4)^2 + (6-6)^2 + (6-8)^2
+ (6-10)^2]/4 = 10. Standard deviation = (variance)^0.5 = 3.2
Study Session: 1 - RA: 3
47. Correct answer: D
Effective rate = (1 + Stated rate / Periodicity)^Periodicity -1 = (1 + 9%/12)^12 -1 = 9.38%.
Study Session: 1 - RA: 3
48. Correct answer: B
Continuously compounded rate = ln(FV/PV)/N = ln(10,000 / 7,500) / 5 = 5.75%.
Study Session: 1 - RA: 3
49. Correct answer: D
Without any negative cash flow in the stream, the rate of return is infinite. So, the IRR of this
investment cannot be determined.
Study Session: 1 - RA: 3
50. Correct answer: C
FV of this payment = $4,000 x (1 + 7%)^10 = $7,869.
Study Session: 1 - RA: 3
51. Correct answer: A
The value of this annuity at the start of the year 2 = Coupon x [1 - 1/(1 + i)^N]/i = 900 x [1 1/(1 + 8%)^25] / 8% = $9,607.
To find the value of this amount in today's month we need to discount it back 1 years. Thus,
present value of this annuity = $9,607 / (1 + 8%)^1 = $8,895.
Study Session: 1 - RA: 3

http://64.106.152.240/inmarkets/passpro/index.cfm?event=questions....

2004-08-02

PASS PRO - Copyright 2004 Inmarkets Ltd - Licensed to Raymond chen

103/150

52. Correct answer: A


The value of this annuity at the start of the year 15 = Coupon x [1 - 1/(1 + i)^N]/i = 1500 x [1
- 1/(1 + 8%)^10] / 8% = $10,065.
To find the value of this amount in today's month, we need to discount it back 14 years. Thus,
present value of this annuity = $10,065 / (1 + 8%)^14 = $3,427.
Study Session: 1 - RA: 3
53. Correct answer: C
FV of this payment = $7,000 x (1 + 6%)^8 = $11,157.
Study Session: 1 - RA: 3
54. Correct answer: C
Continuously compounded rate = ln(FV/PV)/N = ln(8,000 / 7,000) / 4 = 3.34%.
Study Session: 1 - RA: 3
55. Correct answer: C
The mean of this set is 6. Therefore, the variance is [(6-2)^2 + (6-4)^2 + (6-6)^2 + (6-8)^2
+ (6-10)^2]/4 = 10. Standard deviation = (variance)^0.5 = 3.2
Study Session: 1 - RA: 3
56. Correct answer: A
Continuously compounded rate = ln(FV/PV)/N = ln(21000 / 16000) / 2 = 13.6%.
Study Session: 1 - RA: 3
57. Correct answer: A
The value of this annuity at the start of the year 2 = Coupon x [1 - 1/(1 + i)^N]/i = 2,000 x [1
- 1/(1 + 4%)^8] / 4% = $13,465.
To find the value of this amount in today's month, we need to discount it back one year. Thus,
present value of this annuity = $13,465 / (1 + 4%)^1 = $12,947.
Study Session: 1 - RA: 3
58. Correct answer: C
Effective rate = (1 + Stated rate / Periodicity)^Periodicity -1 = (1 + 7.79%/12)^12 -1 =
8.07%.
Study Session: 1 - RA: 3
59. Correct answer: C
Continuously compounded rate = ln(FV/PV)/N = ln(8,000 / 7,000) / 4 = 3.34%.

http://64.106.152.240/inmarkets/passpro/index.cfm?event=questions....

2004-08-02

PASS PRO - Copyright 2004 Inmarkets Ltd - Licensed to Raymond chen

104/150

Study Session: 1 - RA: 3


60. Correct answer: B
1,250 / r = 1,500 / (r + 1%)
or, 1,250 x (r + 1%) = 1,500 x r
or, r = 12.5 / (1,500 - 1,250) = 5%.
Study Session: 1 - RA: 3
61. Correct answer: C
FV of this payment = $7,000 x (1 + 6%)^8 = $11,157.
Study Session: 1 - RA: 3
62. Correct answer: B
FV of this payment = $6,000 x (1 + 4%)^4 = $7,019.
Study Session: 1 - RA: 3
63. Correct answer: B
Rate of inflation = (Rise in prices)^(1/Number of years) - 1 = (2)^(1/8) - 1 = 9.05%.
Study Session: 1 - RA: 3
64. Correct answer: B
The price of this bond is approximately equal to 100 / (1 + 0.05) ^ 2, which is equal to about
91.
Study Session: 1 - RA: 3
65. Correct answer: B
To achieve equivalence, 10,000 / (1 + r/2)^2 = 11,000 / (1 + r/2)^6 or,
(1 + r/2)^4 = 1.1, or
r = (1.1^(1/4) - 1) x 2 = 4.8%.
Study Session: 1 - RA: 3
66. Correct answer: C
Black Scholes framework uses continuously compounded rate. Given the semi-annual rate, this
can be calculated as = ln(1.025) x 2 = 4.9385%.
Study Session: 1 - RA: 3
67. Correct answer: A
Correlation coefficient is a measure of the linear relationship between two random variables. It
can be calculated by scaling the covariance between them and varies between -1 (perfect
negative correlation) to +1 (perfect positive correlation). If the variables are independent, they
will have a correlation coefficient of zero, but the reverse does not always hold true (i.e. a
correlation coefficient of zero does not necessarily mean that they are independent).

http://64.106.152.240/inmarkets/passpro/index.cfm?event=questions....

2004-08-02

PASS PRO - Copyright 2004 Inmarkets Ltd - Licensed to Raymond chen

105/150

Covariance between the variables = Correlation coefficient x Standard deviation of the first
variable x Standard deviation of the second variable. This calculation does not require the use
of mean at all.
Study Session: 1 - RA: 4
68. Correct answer: C
The long position on negatively correlated index X does not hedge the short S&P 500, but
rather the two add to it each other. In order to profit from the negative correlation, one needs
to go long on both assets.
Study Session: 1 - RA: 4
69. Correct answer: D
Covariance = Correlation x Standard Deviation_A x Standard Deviation_B. Therefore Standard
Deviation_B
= Covariance / Correlation / Standard Deviation_A
= 5 / 0.5 / 8^0.5 = 3.5355.
Therefore Variance_B = 3.5355^2 = 12.5.
Study Session: 1 - RA: 4
70. Correct answer: D
Mean = -2 x 60% + 10 x 20% + 20 x 20% = 4.8.
Study Session: 1 - RA: 4
71. Correct answer: D
First we calculate the Mean = -12 x 24% + 4 x 40% + 14 x 36% = 3.76
Then, Variance = (3.76 - -12)^2 x 24% + (3.76 - 4)^2 x 40% + (3.76 - 14)^2 x 36% = 97.38
Study Session: 1 - RA: 4
72. Correct answer: B
Correlation coefficient is a measure of the linear relationship between two random variables. It
can be calculated by scaling the covariance between them and varies between -1 (perfect
negative correlation) to +1 (perfect positive correlation). If the variables are independent, they
will have a correlation coefficient of zero, but the reverse does not always hold true (i.e. a
correlation coefficient of zero does not necessarily mean that they are independent).
Covariance between the variables = Correlation coefficient x Standard deviation of the first
variable x Standard deviation of the second variable. This calculation does not require the use
of mean at all.
Study Session: 1 - RA: 4
73. Correct answer: D
Covariance = Correlation x Standard Deviation_A x Standard Deviation_B. Therefore Standard
Deviation_B

http://64.106.152.240/inmarkets/passpro/index.cfm?event=questions....

2004-08-02

PASS PRO - Copyright 2004 Inmarkets Ltd - Licensed to Raymond chen

106/150

= Covariance / Correlation / Standard Deviation_A


= 5 / 0.5 / 8^0.5 = 3.5355.
Therefore Variance_B = 3.5355^2 = 12.5.
Study Session: 1 - RA: 4
74. Correct answer: B
Volatility scales as the square root of time. Therefore, the 22-day volatility = 5-day volatility x
(22 / 5)^0.5 = 0.89% x 2.0976 = 1.87%.
Study Session: 1 - RA: 4
75. Correct answer: C
Mean = -2 x 50% + 8 x 25% + 15 x 25% = 4.75.
Study Session: 1 - RA: 4
76. Correct answer: B
Firstly, we calculate the Mean = -2 x 60% + 10 x 20% + 20 x 20% = 4.8
Then, Variance = (4.8 - -2)^2 x 60% + (4.8 - 10)^2 x 20% + (4.8 - 20)^2 x 20% = 79.36
Finally, Standard deviation = 79.36^0.5 = 8.91.
Study Session: 1 - RA: 4
77. Correct answer: D
Mean = -10 x 30% + 5 x 40% + 25 x 30% = 6.5.
Study Session: 1 - RA: 4
78. Correct answer: B
Volatility scales as the square root of time. Therefore, the 23-day volatility = 10-day volatility x
(23 / 10)^0.5 = 1.4% x 1.5166 = 2.12%.
Study Session: 1 - RA: 4
79. Correct answer: B
Volatility scales as the square root of time. Therefore the 22-day volatility = 5-day volatility x
(22 / 5)^0.5 = 1.2% x 2.0976 = 2.52%.
Study Session: 1 - RA: 4
80. Correct answer: B
Firstly, we calculate the Mean = -15 x 30% + 5 x 40% + 25 x 30% = 5
Then, Variance = (5 - -15)^2 x 30% + (5 - 5)^2 x 40% + (5 - 25)^2 x 30% = 240

http://64.106.152.240/inmarkets/passpro/index.cfm?event=questions....

2004-08-02

PASS PRO - Copyright 2004 Inmarkets Ltd - Licensed to Raymond chen

107/150

Finally, Standard deviation = 240^0.5 = 15.49.


Study Session: 1 - RA: 4
81. Correct answer: B
Let x be the standard deviation of asset 1. Thus 13^2 = (0.5 x SD1)^2 + (0.5 x 19.5)^2 + 2 x
0.5 x 0.5 x 0.5 x SD1 x 19.5. Solving this problem requires trail and error. Always try the
middle number because even if it is not the answer, you can judge whether to try a higher
number or a lower number.
Study Session: 1 - RA: 4
82. Correct answer: B
Let x be the standard deviation of asset 1. Thus 13^2 = (0.5 x SD1)^2 + (0.5 x 19.5)^2 + 2 x
0.5 x 0.5 x 0.5 x SD1 x 19.5. Solving this problem requires trail and error. Always try the
middle number because even if it is not the answer, you can judge whether to try a higher
number or a lower number.
Study Session: 1 - RA: 4
83. Correct answer: C
Correlation coefficient is a measure of the linear relationship between two random variables. It
can be calculated by scaling the covariance between them and varies between -1 (perfect
negative correlation) to +1 (perfect positive correlation). If the variables are independent they
will have a correlation coefficient of zero, but the reverse does not always hold true (i.e. a
correlation coefficient of zero does not necessarily mean that they are independent).
Covariance between the variables = Correlation coefficient x Standard deviation of the first
variable x Standard deviation of the second variable. This calculation does not require the use
of mean at all.
Study Session: 1 - RA: 4
84. Correct answer: B
Mean = -4 x 20% + 6 x 40% + 12 x 40% = 6.4.
Study Session: 1 - RA: 4
85. Correct answer: C
Correlation coefficient is a measure of linear relationship between two random variables. It can
be calculated by scaling the covariance between them and varies between -1 (perfect negative
correlation) to +1 (perfect positive correlation). If the variables are independent they will have
a correlation coefficient of zero, but the reverse does not always hold true (i.e. a correlation
coefficient of zero does not necessarily mean that they are independent).
Covariance between the variables = Correlation coefficient x Standard deviation of the first
variable x Standard deviation of the second variable. This calculation does not require the use
of mean at all.
Study Session: 1 - RA: 4
86. Correct answer: B

http://64.106.152.240/inmarkets/passpro/index.cfm?event=questions....

2004-08-02

PASS PRO - Copyright 2004 Inmarkets Ltd - Licensed to Raymond chen

108/150

Volatility scales as the square root of time. Therefore, the 22-day volatility = 5-day volatility x
(22 / 5)^0.5 = 0.89% x 2.0976 = 1.87%.
Study Session: 1 - RA: 4
87. Correct answer: D
Covariance = Correlation x Standard Deviation_A x Standard Deviation_B. Therefore Standard
Deviation_B
= Covariance / Correlation / Standard Deviation_A
= 5 / 0.5 / 8^0.5 = 3.5355.
Therefore, Variance_B = 3.5355^2 = 12.5.
Study Session: 1 - RA: 4
88. Correct answer: D
Correlation coefficient is a measure of the linear relationship between two random variables. It
can be calculated by scaling the covariance between them and varies between -1 (perfect
negative correlation) to +1 (perfect positive correlation). If the variables are independent they
will have a correlation coefficient of zero, but the reverse does not always hold true (i.e. a
correlation coefficient of zero does not necessarily mean that they are independent).
Covariance between the variables = Correlation coefficient x Standard deviation of the first
variable x Standard deviation of the second variable. This calculation does not require the use
of mean at all.
Study Session: 1 - RA: 4
89. Correct answer: B
Volatility scales as the square root of time. Therefore the 250-day volatility = 5-day volatility x
(250 / 5)^0.5 = 1.5% x 7.0711 = 10.61%.
Study Session: 1 - RA: 4
90. Correct answer: B
Mean = -4 x 20% + 6 x 40% + 12 x 40% = 6.4.
Study Session: 1 - RA: 4
91. Correct answer: C
Mean = -2 x 50% + 8 x 25% + 15 x 25% = 4.75.
Study Session: 1 - RA: 4
92. Correct answer: D
Step 1. The returns for the three scenarios given are: 42.857% [= (50 - 35)/35], 20% [= (42 35) / 35], and -42.857% [= (20 - 35) / 35]
Step 2. Calculate expected price = 33% x 42.857% + 20% x 20% + 47% x -42.857% = -2%.

http://64.106.152.240/inmarkets/passpro/index.cfm?event=questions....

2004-08-02

PASS PRO - Copyright 2004 Inmarkets Ltd - Licensed to Raymond chen

109/150

Step 3. Calculate Variance = 33% x (-2% - 42.857%)^2 + 20% x (-2% - 20%)^2 + 47% x (2% + 42.857%)^2 = 0.154538
Step 4. Calculate volatility/standard deviation = 0.154538^0.5 = 39.31%.
Study Session: 1 - RA: 5
93. Correct answer: D
Coefficient of variation (CV) is a measure of relative risk and is calculated as: CV = (standard
deviation of returns) / (expected rate of return). The calculations for each investment are
shown below:
CV(A) = 0.008^0.5 / 0.15 = 0.596.
CV(B) = 0.005^0.5 / 0.12 = 0.589.
CV(C) = 0.03 / 0.08 = 0.375.
CV(D) = 0.02 / 0.03 = 0.667.
Study Session: 1 - RA: 5
94. Correct answer: D
The null hypothesis for this test is that the coefficient is equal to zero. Therefore the test
statistic = (Observed value - 0) / Standard error = (9% - 0) / 4% = 2.25.
Study Session: 1 - RA: 5
95. Correct answer: A
Coefficient of variation (CV) is a measure of relative risk and is calculated as: CV = (standard
deviation of returns) / (expected rate of return). The calculations for each investment are
shown below:
CV(A) = 0.005^0.5 / 0.19 = 0.372.
CV(B) = 0.002^0.5 / 0.15 = 0.298.
CV(C) = 0.03 / 0.1 = 0.3.
CV(D) = 0.01 / 0.03 = 0.333.
Study Session: 1 - RA: 5
96. Correct answer: D
Coefficient of variation (CV) is a measure of relative risk and is calculated as: CV = (standard
deviation of returns) / (expected rate of return). The calculations for each investment are
shown below:
CV(A) = 0.008^0.5 / 0.15 = 0.596.
CV(B) = 0.005^0.5 / 0.12 = 0.589.
CV(C) = 0.03 / 0.08 = 0.375.
CV(D) = 0.02 / 0.03 = 0.667.
Study Session: 1 - RA: 5
97. Correct answer: C
In this distribution, the lowest element is 1.5 percent below the mean whereas the highest
element is 2.5 percent above the mean. Thus the distribution is skewed towards the right.

http://64.106.152.240/inmarkets/passpro/index.cfm?event=questions....

2004-08-02

PASS PRO - Copyright 2004 Inmarkets Ltd - Licensed to Raymond chen

110/150

Study Session: 1 - RA: 5


98. Correct answer: B
The null hypothesis for this test is that the coefficient is equal to zero. Therefore, the test
statistic = (Observed value - 0) / Standard error = (12% - 0) / 4% = 3.
Study Session: 1 - RA: 5
99. Correct answer: D
The null hypothesis for this test is that the coefficient is equal to zero. Therefore the test
statistic = (Observed value - 0) / Standard error = (9% - 0) / 4% = 2.25.
Study Session: 1 - RA: 5
100. Correct answer: D
The null hypothesis for this test is that the coefficient is equal to zero. Therefore the test
statistic = (Observed value - 0) / Standard error = (11.8% - 0) / 3% = 3.93.
Study Session: 1 - RA: 5
101. Correct answer: C
Standard error of the mean = s / n^0.5 = 3.5 / 30^0.5 = 0.64.
Study Session: 1 - RA: 5
102. Correct answer: D
The null hypothesis for this test is that the coefficient is equal to zero. Therefore the test
statistic = (Observed value - 0) / Standard error = (12.25% - 0) / 8% = 1.53.
Study Session: 1 - RA: 5
103. Correct answer: D
Coefficient of variation (CV) is a measure of relative risk and is calculated as: CV = (standard
deviation of returns) / (expected rate of return). The calculations for each investment are
shown below:
CV(A) = 0.008^0.5 / 0.15 = 0.596.
CV(B) = 0.005^0.5 / 0.12 = 0.589.
CV(C) = 0.03 / 0.08 = 0.375.
CV(D) = 0.02 / 0.03 = 0.667.
Study Session: 1 - RA: 5
104. Correct answer: B
The null hypothesis for this test is that the coefficient is equal to zero. Therefore, the test
statistic = (Observed value - 0) / Standard error = (12% - 0) / 4% = 3.
Study Session: 1 - RA: 5
105. Correct answer: D

http://64.106.152.240/inmarkets/passpro/index.cfm?event=questions....

2004-08-02

PASS PRO - Copyright 2004 Inmarkets Ltd - Licensed to Raymond chen

111/150

Coefficient of variation (CV) is a measure of relative risk and is calculated as: CV = (standard
deviation of returns) / (expected rate of return). The calculations for each investment are
shown below:
CV(A) = 0.005^0.5 / 0.12 = 0.589.
CV(B) = 0.002^0.5 / 0.1 = 0.447.
CV(C) = 0.03 / 0.15 = 0.2.
CV(D) = 0.05 / 0.06 = 0.833.
Study Session: 1 - RA: 5
106. Correct answer: A
Coefficient of variation (CV) is a measure of relative risk and is calculated as: CV = (standard
deviation of returns) / (expected rate of return). The calculations for each investment are
shown below:
CV(A) = 0.002^0.5 / 0.1 = 0.447.
CV(B) = 0.003^0.5 / 0.15 = 0.365.
CV(C) = 0.03 / 0.08 = 0.375.
CV(D) = 0.01 / 0.05% = 0.2.
Study Session: 1 - RA: 5
107. Correct answer: C
Standard error of the mean = s /vn = 4/v40 = 0.63.
Study Session: 1 - RA: 5
108. Correct answer: D
Coefficient of variation (CV) is a measure of relative risk and is calculated as: CV = (standard
deviation of returns) / (expected rate of return). The calculations for each investment are
shown below:
CV(A) = 0.008^0.5 / 0.15 = 0.596.
CV(B) = 0.005^0.5 / 0.12 = 0.589.
CV(C) = 0.03 / 0.08 = 0.375.
CV(D) = 0.02 / 0.03 = 0.667.
Study Session: 1 - RA: 5
109. Correct answer: C
Standard error of the mean = s /n^0.5 = 6/90^0.5 = 0.63.
Study Session: 1 - RA: 5
110. Correct answer: C
Standard error of the mean = s /n^0.5 = 6/50^0.5 = 0.85.
Study Session: 1 - RA: 5
111. Correct answer: C

http://64.106.152.240/inmarkets/passpro/index.cfm?event=questions....

2004-08-02

PASS PRO - Copyright 2004 Inmarkets Ltd - Licensed to Raymond chen

112/150

Standard error of the mean = s /n^0.5 = 7/60^0.5 = 0.9.


Study Session: 1 - RA: 5
112. Correct answer: C
Standard error of the mean = s /n^0.5 = 6/90^0.5 = 0.63.
Study Session: 1 - RA: 5
113. Correct answer: D
The null hypothesis for this test is that the coefficient is equal to zero. Therefore the test
statistic = (Observed value - 0) / Standard error = (9% - 0) / 3.25% = 2.77.
Study Session: 1 - RA: 5
114. Correct answer: B
This problem requires a two-tailed F-test (with the null hypothesis Ho: variance of
pharmaceutical industry returns = variance of the market). From the given data the test
statistic = 0.13^2 / 0.092^2 = 2. The critical value of the F-statistic (for df1=40, df2=20 and
p=0.025) is 2.29. Since the test statistic is lower than the critical statistic, we cannot reject the
null hypothesis that the standard deviation of pharmaceutical stocks is same as that of the
market.
Study Session: 1 - RA: 6
115. Correct answer: C
Since this is a one-tailed test with a 0.01 significance level the critical Z value is 2.33.
Study Session: 1 - RA: 6
116. Correct answer: A
A t-test is used when the sample size is small and the variance of the population is unknown.
Study Session: 1 - RA: 6
117. Correct answer: D
Tests of the variance (or standard deviation) of a population require the chi-squared test.
Study Session: 1 - RA: 6
118. Correct answer: A
The required test for testing the variance is the chi-squared test. The test statistic = (n - 1) x
Sample variance / Hypothesized variance = 60 x 0.21^2 / 0.14^2 = 135. To test whether the
standard deviation is higher (Ho: standard deviation is lower than or equal to 14%), the critical
value of chi-squared will be 79.08 (using df = 60 and p= 0.05). Since the test statistic is higher
than the critical value, the analyst can reject the null hypothesis and conclude that the standard
deviation of returns is higher than 14%.
Study Session: 1 - RA: 6

http://64.106.152.240/inmarkets/passpro/index.cfm?event=questions....

2004-08-02

PASS PRO - Copyright 2004 Inmarkets Ltd - Licensed to Raymond chen

113/150

119. Correct answer: C


Tests of differences between variances (or standard deviation) of two populations require the Ftest. From the given data, the test statistic = 0.15^2 / 0.11^2 = 1.86. The critical value of the
F-statistic (for df1= 40, df2= 20 and p= 0.05) is 1.99. Since the test statistic is lower than the
critical statistic, we cannot reject the null hypothesis that the standard deviation of utility stocks
is equal to or higher than that of the market.
Study Session: 1 - RA: 6
120. Correct answer: B
The required test for testing the variance is the chi-squared test. The test statistic = (n - 1) x
Sample variance / Hypothesized variance = 19 x 0.12^2 / 0.18^2 = 8.44. To test whether the
standard deviation is different (Ho: standard deviation is equal to 18%), the critical values of
chi-squared will be 8.91 and 32.85(using df = 19, p-lower = 0.975 and p-higher = 0.025).
Since the test statistic falls outside the critical range, the analyst can reject the null hypothesis
and conclude that the standard deviation of returns is different from 18%.
Study Session: 1 - RA: 6
121. Correct answer: C
This is a two-tailed t-test for the difference between means of two independent populations.
Step 1. The common variance = [(n1 -1) x Variance1 + (n2 - 1) x Variance2] / (n1 + n2 - 2) =
[(21 -1) x 0.12^2 + (41 - 1) x 0.13^2] / (21 + 41 - 2) = 0.0161. Step 2. The value of the test
statistic = (Mean1 - Mean2) / (Common variance/n1 + Common variance/n2)^0.5 = (0.15 0.12) / (0.0161/21 + 0.0161/41)^0.5 = 0.88. Using degrees of freedom = n1 + n2 - 2 = 21 +
41 - 2 = 60 and level of significance = alpha / 2 = 0.025 the critical value = 2.0. Since the test
statistic is lower than the critical statistic, we cannot reject the null hypothesis that the mean
returns are same.
Study Session: 1 - RA: 6
122. Correct answer: C
The required test for testing the variance is the chi-squared test. The test statistic = (n - 1) x
Sample variance / Hypothesized variance = 14 x 0.1^2 / 0.15^2 = 6.22. To test whether the
standard deviation is different (Ho: standard deviation is equal to 15%), the critical values of
chi-squared will be 5.63 and 26.12 (using df=14, p-lower=0.975 and p-higher=0.025). Since
the test statistic falls inside the critical range, the analyst cannot reject the null hypothesis .
Study Session: 1 - RA: 6
123. Correct answer: A
This is a t-test for the difference between means of two independent populations. Step 1. The
common variance = [(n1 -1) x Variance1 + (n2 - 1) x Variance2] / (n1 + n2 - 2) = [(21 -1) x
0.13^2 + (41 - 1) x 0.15^2] / (21 + 41 - 2) = 0.0206. Step 2. The value of the test statistic =
(Mean1 - Mean2) / (Common variance/n1 + Common variance/n2)^0.5 = (0.12 - 0.09) /
(0.0206/21 + 0.0206/41)^0.5 = 0.78.
Study Session: 1 - RA: 6
124. Correct answer: A
Tests of the variance of a population require the chi-squared test. Since the analyst wants to
show that the standard deviation is less than 14%, this will be chosen as the alternative

http://64.106.152.240/inmarkets/passpro/index.cfm?event=questions....

2004-08-02

PASS PRO - Copyright 2004 Inmarkets Ltd - Licensed to Raymond chen

114/150

hypothesis and the null hypothesis will be that the standard deviation is higher than or equal to
14%. Thus, using a probability in the right tail of 95% and degrees of freedom of 24, the critical
value from the chi-squared table is 13.85.
Study Session: 1 - RA: 6
125. Correct answer: C
Tests of the variance of a population require the chi-squared test. For this data chi-squared =
(n - 1) x Sample variance / Hypothesized variance = 14 x 0.13^2 / 0.18^2 = 7.3.
Study Session: 1 - RA: 6
126. Correct answer: B
The null hypothesis in this study is that the performance of active portfolio managers <=
average for all portfolio managers. The value of test statistic for this, Z = (0.15 - 0.12)/(0.08 /
16^0.5) = 0.03 / (0.08 / 4) = 1.5. Since the analyst is using a 0.05 significance level the
critical value of Z is 1.65 (the rejection region is above 1.65). As the test statistic is lower than
the critical value the null hypothesis cannot be rejected, i.e. the performance of active portfolio
managers is not significantly higher than the average for all portfolio managers.
Study Session: 1 - RA: 6
127. Correct answer: D
Theoretically, the z-statistic can be used when the population variance is known, but it can also
be used for populations with unknown variances if the sample size is large. However, if the
population variance is unknown and the sample size is small then it is necessary to use tstatistic.
Study Session: 1 - RA: 6
128. Correct answer: C
The significance level does not depend on the data. Rather it is chosen separately based on how
much evidence is required before a null hypothesis will be rejected. The steps in the hypothesis
testing process are: 1) State a hypothesis. 2) Identify the test statistic and its probability
distribution. 3) Specify the significance level. 4) State the decision rule. 5) Collect data and
perform the calculations. 6) Make the statistical decision. 7) Make the investment/economic
decision based on the above.
Study Session: 1 - RA: 6
129. Correct answer: A
This problem requires a hypothesis test in which the null hypothesis, H0, is that Peter's
performance is not significantly higher than average. The z statistic for this test = (Peter's
performance - Average performance) / Standard deviation = (1,000,000 - 800,000) / 100,000
= 2. The critical value of the z statistic for a significance level of 0.05 is 1.645. Since the test
statistic is larger than this critical value, we reject the null hypothesis and accept the alternate
hypothesis that Peter's performance is significantly higher than average.
Study Session: 1 - RA: 6
130. Correct answer: B

http://64.106.152.240/inmarkets/passpro/index.cfm?event=questions....

2004-08-02

PASS PRO - Copyright 2004 Inmarkets Ltd - Licensed to Raymond chen

115/150

Tests of differences between variances (or standard deviation) of two populations require the Ftest. Looking up the F-table for 0.05 significance level, with 40 degrees of freedom in the
numerator and 20 in the denominator (note the population with the higher variance goes in the
numerator), the critical value of F is 1.99.
Study Session: 1 - RA: 6
131. Correct answer: D
Tests of the variance of a population require the chi-squared test. For this data chi-squared =
(n - 1) x Sample variance / Hypothesized variance = 59 x 0.22^2 / 0.045 = 63.46. Since the
analyst wants to show that the variance is more than 0.045, this will be chosen as the
alternative hypothesis and the null hypothesis will be that the variance is lower than or equal to
0.045. The critical value of the chi-squared statistic (for df = 59 and p= 5%) is 77.93. Since the
test statistic is lower than the critical statistic, we cannot reject the null hypothesis that the
variance is equal to or lower than 0.045.
Study Session: 1 - RA: 6
132. Correct answer: B
The required test for testing the variance is the chi-squared test. The test statistic = (n - 1) x
Sample variance / Hypothesized variance = 19 x 0.09^2 / 0.16^2 = 6.01. To test whether the
standard deviation is different (Ho: standard deviation is equal to 16%), the critical values of
chi-squared will be 8.91 and 32.85(using df = 19, p-lower = 0.975 and p-higher = 0.025).
Since the test statistic falls outside the critical range, the analyst can reject the null hypothesis
and conclude that the standard deviation of returns is different from 16%.
Study Session: 1 - RA: 6
133. Correct answer: D
The alternate hypothesis is the statement which will be accepted if the null hypothesis is proven
wrong. Therefore, we make whatever we are trying to test as the alternate hypothesis - in this
case that the mean price of luxury cars is greater than $80,000, and the null hypothesis as the
opposite (the mean price of luxury cars is less than or equal to $80,000). This problem is a
common example of how statisticians establish hypotheses by proving that the opposite (i.e.
the null hypothesis) is false.
Study Session: 1 - RA: 6
134. Correct answer: A
This is a paired comparison test because the two samples are not independent. The null
hypothesis: Ho: mean difference in betas before and after the dot-com bust = 0. The test
statistic = (Mean of differences - 0) / Standard error = (0.18 - 0) / (0.2 / 10^0.5) = 2.85. The
critical value of t-statistic for 9 degrees of freedom and p = 0.025 (this is a two sided test) is
2.26. Since the test statistic is higher than the critical value of t-statistic, the analyst can reject
the null hypothesis.
Study Session: 1 - RA: 6
135. Correct answer: C
Tests of the variance of a population require the chi-squared test. Since the analyst wants to
show that the variance is more than 0.04, this will be chosen as the alternative hypothesis and
the null hypothesis will be that the variance is lower than or equal to 0.04. Thus using a
probability in the right tail of 5% and degrees of freedom of 30, the critical value from the chi-

http://64.106.152.240/inmarkets/passpro/index.cfm?event=questions....

2004-08-02

PASS PRO - Copyright 2004 Inmarkets Ltd - Licensed to Raymond chen

116/150

squared table is 54.57.


Study Session: 1 - RA: 6
136. Correct answer: B
Tests of the variance of a population require the chi-squared test. For this data chi-squared =
(n - 1) x Sample variance / Hypothesized variance = 59 x 0.22^2 / 0.045 = 63.46.
Study Session: 1 - RA: 6
137. Correct answer: B
This is a paired comparison test because the two samples are not independent. The null
hypothesis: Ho: mean difference in betas before and after the 1990 deregulation = 0. The test
statistic = (Mean of differences - 0) / Standard error = (0.25 - 0) / (0.3 / 12^0.5) = 2.89. The
critical value of t-statistic for 11 degrees of freedom and p = 0.005 (this is a two sided test) is
3.11. Since the test statistic is lower than the critical value of t-statistic, the analyst cannot
reject the null hypothesis.
Study Session: 1 - RA: 6
138. Correct answer: A
In hypothesis testing we accept the alternate hypothesis if the null hypothesis has been
rejected. Type I error happens if the null hypothesis is rejected when it is actually true. Type II
error happens if the null hypothesis is accepted when it is actually false. The power of the test
is the probability of correctly rejecting the null hypothesis (when it is false), so minimizing Type
II errors would maximize the power of the test.
Study Session: 1 - RA: 6
139. Correct answer: C
Coefficient of determination = ESS / TSS = 3.312 / 3.617 = 0.916.
Study Session: 1 - RA: 7
140. Correct answer: A
This is a one-tailed test with 39 degrees of freedom and significance level of 0.1. Looking up the
Student's t-distribution for df = 39 and p = 0.1, we get the critical value of 1.304.
Study Session: 1 - RA: 7
141. Correct answer: D
Correlation = (Coefficient of determination)^0.5 = (ESS / TSS)^0.5 = (761.562 / 865.333)
^0.5 = 0.938.
Study Session: 1 - RA: 7
142. Correct answer: C
Firstly, we calculate the slope coefficient = Sum_XY / Sum_X^2 = 9.08 / 24.9 = 0.365.
Then we calculate the intercept = Avg_Y - Slope x Avg X = 4.343 - 0.365 x 3.5 = 3.07.

http://64.106.152.240/inmarkets/passpro/index.cfm?event=questions....

2004-08-02

PASS PRO - Copyright 2004 Inmarkets Ltd - Licensed to Raymond chen

117/150

Study Session: 1 - RA: 7


143. Correct answer: A
Firstly, we calculate the slope coefficient = Sum_XY / Sum_X^2 = 133.302 / 23.333 = 5.713.
Then we calculate the intercept = Avg_Y - Slope x Avg X = -15.667 - 5.713 x 6.333 = -51.85.
Study Session: 1 - RA: 7
144. Correct answer: D
Correlation = (Coefficient of determination)^0.5 = (ESS / TSS)^0.5 = (227,088.816 /
245,142.857)^0.5 = 0.962.
Study Session: 1 - RA: 7
145. Correct answer: D
The standard error = USS / (N-2) = (TSS - ESS) / (N-2) = (113,171.429 - 96,057.049) / (7-2)
= 3,422.876.
Study Session: 1 - RA: 7
146. Correct answer: A
Firstly, we calculate the slope coefficient = Sum_XY / Sum_X^2 = 133.302 / 23.333 = 5.713.
Then we calculate the intercept = Avg_Y - Slope x Avg X = -15.667 - 5.713 x 6.333 = -51.85.
Study Session: 1 - RA: 7
147. Correct answer: D
Correlation = (Coefficient of determination)^0.5 = (ESS / TSS)^0.5 = (3.312 / 3.617)^0.5 =
0.957.
Study Session: 1 - RA: 7
148. Correct answer: D
The percentage of explained variation = Correlation coefficient ^ 2 = 0.6 ^ 2 = 0.36 or 36%.
Therefore, the unexplained variation is 64%.
Study Session: 1 - RA: 7
149. Correct answer: D
Correlation = (Coefficient of determination)^0.5 = (ESS / TSS)^0.5 = (761.562 / 865.333)
^0.5 = 0.938.
Study Session: 1 - RA: 7
150. Correct answer: B
The percentage of explained variation = Correlation coefficient ^ 2 = 0.75 ^ 2 = 0.5625 or

http://64.106.152.240/inmarkets/passpro/index.cfm?event=questions....

2004-08-02

PASS PRO - Copyright 2004 Inmarkets Ltd - Licensed to Raymond chen

118/150

56.25%. Therefore, the unexplained variation is 43.75%.


Study Session: 1 - RA: 7
151. Correct answer: B
The percentage of explained variation = Correlation coefficient ^ 2 = 0.85 ^ 2 = 0.7225 or
72.25%. Therefore, the unexplained variation is 27.75%.
Study Session: 1 - RA: 7
152. Correct answer: D
The percentage of explained variation = Correlation coefficient ^ 2 = 0.25 ^ 2 = 0.0625 or
6.25%. Therefore the unexplained variation is 93.75%.
Study Session: 1 - RA: 7
153. Correct answer: C
Firstly, we calculate the slope coefficient = Sum_XY / Sum_X^2 = 9.08 / 24.9 = 0.365.
Then we calculate the intercept = Avg_Y - Slope x Avg X = 4.343 - 0.365 x 3.5 = 3.07.
Study Session: 1 - RA: 7
154. Correct answer: B
The percentage of explained variation = Correlation coefficient ^ 2 = 0.75 ^ 2 = 0.5625 or
56.25%. Therefore, the unexplained variation is 43.75%.
Study Session: 1 - RA: 7
155. Correct answer: C
The test to compare variances of two normally distributed populations is F-statistic.
Study Session: 1 - RA: 7
156. Correct answer: C
Coefficient of determination = ESS / TSS = 227,088.816 / 245,142.857 = 0.926.
Study Session: 1 - RA: 7
157. Correct answer: D
The standard error = USS / (N-2) = (TSS - ESS) / (N-2) = (3.617 - 3.312) / (7-2) = 0.061.
Study Session: 1 - RA: 7
158. Correct answer: D
The standard error = USS / (N-2) = (TSS - ESS) / (N-2) = (245,142.857 - 227,088.816) / (72) = 3,610.8082.

http://64.106.152.240/inmarkets/passpro/index.cfm?event=questions....

2004-08-02

PASS PRO - Copyright 2004 Inmarkets Ltd - Licensed to Raymond chen

119/150

Study Session: 1 - RA: 7


159. Correct answer: A
The slope coefficient = Sum_XY / Sum_X^2 = 9.08 / 24.9 = 0.365.
Study Session: 1 - RA: 7
160. Correct answer: D
The percentage of explained variation = Correlation coefficient ^ 2 = 0.25 ^ 2 = 0.0625 or
6.25%. Therefore the unexplained variation is 93.75%.
Study Session: 1 - RA: 7
161. Correct answer: C
The slope coefficient = Sum_XY / Sum_X^2 = 1,640 / 28 = 58.571.
Study Session: 1 - RA: 7
162. Correct answer: D
Correlation = (Coefficient of determination)^0.5 = (ESS / TSS)^0.5 = (227,088.816 /
245,142.857)^0.5 = 0.962.
Study Session: 1 - RA: 7
163. Correct answer: D
Correlation = (Coefficient of determination)^0.5 = (ESS / TSS)^0.5 = (96,057.049 /
113,171.429)^0.5 = 0.921.
Study Session: 1 - RA: 7
164. Correct answer: A
Since the investments are perfectly negatively correlated the volatility of the portfolio = (w1 x
v1 - w2 x v2). This can be reduced to zero by adjusting the amounts invested in the inverse of
the ratios of the volatilities (i.e. w1/w2 = v2/v1).
Study Session: 1 - RA: 8
165. Correct answer: C
The positive correlation only means that the prices will tend to move in the same direction, not
that they always will.
Study Session: 1 - RA: 8
166. Correct answer: C
Since the investments are perfectly negatively correlated the volatility of the portfolio = (w1 x
v1 - w2 x v2). This can be reduced to zero by adjusting the amounts invested in the inverse of
the ratios of the volatilities (i.e. w1/w2 = v2/v1)
Study Session: 1 - RA: 8

http://64.106.152.240/inmarkets/passpro/index.cfm?event=questions....

2004-08-02

PASS PRO - Copyright 2004 Inmarkets Ltd - Licensed to Raymond chen

120/150

167. Correct answer: C


Since the investments are perfectly negatively correlated the volatility of the portfolio = (w1 x
v1 - w2 x v2). This can be reduced to zero by adjusting the amounts invested in the inverse of
the ratios of the volatilities (i.e. w1/w2 = v2/v1)
Study Session: 1 - RA: 8
168. Correct answer: A
Since the investments are perfectly negatively correlated the volatility of the portfolio = (w1 x
v1 - w2 x v2). This can be reduced to zero by adjusting the amounts invested in the inverse of
the ratios of the volatilities (i.e. w1/w2 = v2/v1).
Study Session: 1 - RA: 8
169. Correct answer: C
The positive correlation only means that the prices will tend to move in the same direction, not
that they always will.
Study Session: 1 - RA: 8
170. Correct answer: A
Since the investments are perfectly negatively correlated the volatility of the portfolio = (w1 x
v1 - w2 x v2). This can be reduced to zero by adjusting the amounts invested in the inverse of
the ratios of the volatilities (i.e. w1/w2 = v2/v1)
Study Session: 1 - RA: 8
171. Correct answer: A
Since the investments are perfectly negatively correlated the volatility of the portfolio = (w1 x
v1 - w2 x v2). This can be reduced to zero by adjusting the amounts invested in the inverse of
the ratios of the volatilities (i.e. w1/w2 = v2/v1)
Study Session: 1 - RA: 8
172. Correct answer: C
The positive correlation only means that the prices will tend to move in the same direction, not
that they always will.
Study Session: 1 - RA: 8
173. Correct answer: C
Correlation coefficient = Covariance / (Standard deviation of stock A x Standard deviation of
market index) = -0.002 / (0.018^0.5 x 0.025^0.5) = -0.09.
Study Session: 1 - RA: 8
174. Correct answer: D
Covariance of returns is an absolute measure of movement. The magnitude depends on size of
the individual assets standard deviations and the relationship between co-movements.

http://64.106.152.240/inmarkets/passpro/index.cfm?event=questions....

2004-08-02

PASS PRO - Copyright 2004 Inmarkets Ltd - Licensed to Raymond chen

121/150

A positive covariance means that the asset returns move in the same direction, while a negative
covariance means that asset returns move in the opposite direction.
A zero covariance means that there is no relationship between asset returns.
Study Session: 1 - RA: 8
175. Correct answer: D
Of the models given here, only Heath, Jarrow and Morton is a no-arbitrage model that
incorporates mean reversion. Cox, Ingersoll and Ross model and Vasicek model incorporate
mean reversion but are not a no-arbitrage model. Black-Scholes does not meet either criteria.
Study Session: 1 - RA: 10
176. Correct answer: A
Since X and Y follow geometric Brownian motion, it follows that log(X) and log(Y) are normally
distributed, which means that log(X) + log(Y) is also normally distributed. log(X) + log(Y) is
equal to log(X * Y), so X * Y must follow geometric Brownian motion.
Study Session: 1 - RA: 10
177. Correct answer: C
Wiener process has a mean change of zero and variance proportional to the time interval.
Study Session: 1 - RA: 10
178. Correct answer: B
Wiener process has a mean change of zero and variance proportional to the time interval.
Generalized Wiener process and Ito process incorporate a trend variable a * dt.
Study Session: 1 - RA: 10
179. Correct answer: D
Only Hull and White is a no-arbitrage model that incorporates mean reversion. Ho and Lee is a
no-arbitrage model that does not incorporate mean reversion. Vasicek model incorporates mean
reversion but is not a no-arbitrage model. Black-Scholes does not meet either criteria.
Study Session: 1 - RA: 10
180. Correct answer: C
Wiener process has a mean change of zero and variance proportional to the time interval.
Study Session: 1 - RA: 10
181. Correct answer: B
If X and Y are lognormally distributed, log(X) and log(Y) will be normally distributed, which
means that log(X) + log(Y) = log(X * Y) is also normally distributed, implying that X * Y must
be lognormally distributed.

http://64.106.152.240/inmarkets/passpro/index.cfm?event=questions....

2004-08-02

PASS PRO - Copyright 2004 Inmarkets Ltd - Licensed to Raymond chen

122/150

Study Session: 1 - RA: 10


182. Correct answer: C
The Hull and White, the Ho and Lee, and the Heath, Jarrow and Morton are no-arbitrage
models.
Study Session: 1 - RA: 10
183. Correct answer: C
dr = a * (b - r) * dt + s * dz, is the only model given above that allows for the interest rate r to
mean revert to a level b.
Study Session: 1 - RA: 10
184. Correct answer: B
In the model: dr = a * (b - r) * dt + s * dz, the interest rate r will mean revert to a level b.
Study Session: 1 - RA: 10
185. Correct answer: B
If X and Y are normally distributed, X + Y will also be normally distributed.
Study Session: 1 - RA: 10
186. Correct answer: B
Using continuous compounding, PV = FV x exp(-Rate x Time period) = $8,500 x exp (- 6.5% x
5) = $8,008.
Study Session: 1 - RA: 1
187. Correct answer: A
Using continuous compounding, PV = FV x exp(-Rate x Time period) = $600 x exp (- 9% x 8) =
$292.
Study Session: 1 - RA: 1
188. Correct answer: A
Using continuous compounding, PV = FV x exp(-Rate x Time period) = $1,500 x exp (- 9% x 4)
= $1,047.
Study Session: 1 - RA: 1
189. Correct answer: C
Continuously compounded rate = ln(FV/PV)/N = ln(700 / 400) / 3 = 18.65%.
Study Session: 1 - RA: 1
190. Correct answer: D

http://64.106.152.240/inmarkets/passpro/index.cfm?event=questions....

2004-08-02

PASS PRO - Copyright 2004 Inmarkets Ltd - Licensed to Raymond chen

123/150

Continuously compounded rate = ln(final value/initial value) / Number of periods = ln(40 /


15) / 2 = 49.04%.
Study Session: 1 - RA: 1
191. Correct answer: B
Using continuous compounding, PV = FV x exp(-Rate x Time period) = $4,500 x exp (- 9% x
6/12) = $4,302.
Study Session: 1 - RA: 1
192. Correct answer: D
Continuously compounded rate = ln(final value/initial value) / Number of periods = ln(40 /
15) / 2 = 49.04%.
Study Session: 1 - RA: 1
193. Correct answer: A
Using continuous compounding, PV = FV x exp(-Rate x Time period) = $500 x exp (- 9% x
9/12) = $467.
Study Session: 1 - RA: 1
194. Correct answer: B
Continuously compounded rate = ln(FV/PV)/N = ln(10000 / 8455) / 2 = 8.39%.
Study Session: 1 - RA: 1
195. Correct answer: B
Continuously compounded rate = ln(FV/PV)/N = ln(10000 / 8455) / 2 = 8.39%.
Study Session: 1 - RA: 1
196. Correct answer: B
Using continuous compounding, PV = FV x exp(-Rate x Time period) = $4,500 x exp (- 9% x
6/12) = $4,302.
Study Session: 1 - RA: 1
197. Correct answer: D
Continuously compounded rate = ln(FV/PV)/N = ln(100 / 75) / 2 = 14.38%.
Study Session: 1 - RA: 1
198. Correct answer: C
Continuously compounded rate = ln(FV/PV)/N = ln(800 / 500) / 6 = 7.83%.
Study Session: 1 - RA: 1

http://64.106.152.240/inmarkets/passpro/index.cfm?event=questions....

2004-08-02

PASS PRO - Copyright 2004 Inmarkets Ltd - Licensed to Raymond chen

124/150

199. Correct answer: A


Using continuous compounding, PV = FV x exp(-Rate x Time period) = $400 x exp (- 9% x 8) =
$195.
Study Session: 1 - RA: 1
200. Correct answer: B
Using continuous compounding, PV = FV x exp(-Rate x Time period) = $8,500 x exp (- 6.5% x
5) = $8,008.
Study Session: 1 - RA: 1
201. Correct answer: B
Using continuous compounding, PV = FV x exp(-Rate x Time period) = $8,500 x exp (- 6.5% x
5) = $8,008.
Study Session: 1 - RA: 1
202. Correct answer: D
This problem requires the use of binomial probability function, P(X=x) = fact(n) / fact(n - x) /
fact(x) * p^x * (1-p)^(n-x) = fact(12) / fact(12 - 8) / fact(8) * 0.6^8 * 0.4^4 = 21.3%.
Study Session: 1 - RA: 2
203. Correct answer: A
Using the multiplication rule the number of packages = Number of golf sets x Number of ways
of choosing three golf balls out of the possible five types.
The second variable in the above equation can be calculated by using combinations formula =
n! / (n-r!) / r! = 5! / 2! / 3! = 10
Therefore, the number of packages = 2 x 10 = 20.
Study Session: 1 - RA: 2
204. Correct answer: B
The movement of S&P500 as described by the analyst is a binomial process, i.e. the S&P500
can go up with a probability of 0.467 or go down with a probability of 0.533. Carrying out this
process for 275 days the variance = n x p x (1 - p) = 275 x 0.467 x 0.533 = 68.451. Therefore
the standard deviation = 68.451^0.5 = 8.274.
Study Session: 1 - RA: 2
205. Correct answer: D
Lognormal distribution is most appropriate for stock prices since it does not allow for negative
values and implies that the returns are normally distributed.
Study Session: 1 - RA: 2
206. Correct answer: B

http://64.106.152.240/inmarkets/passpro/index.cfm?event=questions....

2004-08-02

PASS PRO - Copyright 2004 Inmarkets Ltd - Licensed to Raymond chen

125/150

A lognormal distribution cuts off at zero on the left-hand side and extends to infinity on the
right-hand side.
Study Session: 1 - RA: 2
207. Correct answer: B
A lognormal distribution cuts off at zero on the left-hand side and extends to infinity on the
right-hand side.
Study Session: 1 - RA: 2
208. Correct answer: A
The range of the 95 percent confidence interval = mean ? 1.96 x standard deviation /
Number of employees^0.5 = 32 ? 1.96 x 12 = 32 ? 23.5 = between 8.5 and 55.5
Study Session: 1 - RA: 2
209. Correct answer: B
Given that the daily standard deviation is $8 million, the standard deviation over five days = $8
million x (5/1)^0.5 = $17.89 million.
Given that the returns are normally distributed, we know that 99% of the outcomes will be
above 2.325 standard deviations below the mean, i.e. above $18.4 million.
Study Session: 1 - RA: 2
210. Correct answer: D
The characteristics of a normal distribution are:
It is a continuous distribution.
It is bell shaped.
It is symmetrical about the mean.
It peaks at the mean expected value.
It extends theoretically from negative infinity to positive infinity (the probability asymptotically
approaches zero at plus and minus infinity).
It has a skewness of zero (i.e. it is symmetric).
It has a kurtosis (the level of peakedness) of three. Below three the distribution is platykurtic
(too flat) and above three it is leptokurtic (too tall).
Study Session: 1 - RA: 2
211. Correct answer: C
Confidence interval = 1 - CND
(Quantile) = Area under the normal
distribution to the right of the given
Quantile point. Some of these points
are rather important due to their use
in VAR calculations.
Confidence Level 90% 95% 99%
Quantile (alpha) -1.282 -1.645 -2.326

http://64.106.152.240/inmarkets/passpro/index.cfm?event=questions....

2004-08-02

PASS PRO - Copyright 2004 Inmarkets Ltd - Licensed to Raymond chen

126/150

Note: CND = Cumulative Normal


Distribution function.
Study Session: 1 - RA: 2

212. Correct answer: D


Covariance = Correlation x Standard Deviation_A x Standard Deviation_B. Therefore Standard
Deviation_B
= Covariance / Correlation / Standard Deviation_A
= 16 / 0.5 / 8^0.5 = 11.3137.
Therefore Variance_B = 11.3137^2 = 128.
Study Session: 1 - RA: 2
213. Correct answer: B
The range of the 95 percent confidence interval = mean 1.96 x standard deviation / Number
of employees^0.5 = 35 1.96 x 15 / 80^0.5 = 35 3.3 = between 31.7 and 38.3
Study Session: 1 - RA: 2
214. Correct answer: D
Covariance = Correlation x Standard Deviation_A x Standard Deviation_B. Therefore Standard
Deviation_B
= Covariance / Correlation / Standard Deviation_A
= 16 / 0.5 / 8^0.5 = 11.3137.
Therefore Variance_B = 11.3137^2 = 128.
Study Session: 1 - RA: 2
215. Correct answer: A
Confidence interval = 1 - CND
(Quantile) = Area under the normal
distribution to the right of the given
Quantile point. Some of these point
are rather important due to their use
in VAR calculations.
Confidence Level 90% 95% 99%
Quantile (alpha) 1.282 1.645 2.326

http://64.106.152.240/inmarkets/passpro/index.cfm?event=questions....

2004-08-02

PASS PRO - Copyright 2004 Inmarkets Ltd - Licensed to Raymond chen

127/150

Note: CND = Cumulative Normal


Distribution function
Study Session: 1 - RA: 2

216. Correct answer: B


The characteristics of a normal distribution are:
It is a continuous distribution.
It is bell shaped.
It is symmetrical about the mean.
It peaks at the mean expected value.
It extends theoretically from negative infinity to positive infinity (the probability asymptotically
approaches zero at plus and minus infinity).
It has a skewness of zero (i.e. it is symmetric).
It has a kurtosis (the level of peakedness) of three. Below three the distribution is platykurtic
(too flat) and above three it is leptokurtic (too tall).
Study Session: 1 - RA: 2
217. Correct answer: C
A return of 0 percent and 10 percent is between -2 standard deviation and -1 standard
deviation of the mean, i.e. a quartile of -2 to -1. The confidence levels corresponding to these
quartiles are 97.7% and 84.1%. Hence the probability that the return falls between these limits
= 97.7% - 84.1% = 13.6%
Study Session: 1 - RA: 2
218. Correct answer: B
Given that the daily standard deviation is $9.62 million, the standard deviation over 5 days =
$9.62 million x (5/1)^0.5 = $21.51 million.
Using this standard deviation, the level of $20 million is 2.32 standard deviations [= (70 20)/21.51] from the mean value.
Given that the returns are normally distributed, the probability of the value falling more than
2.32 standard deviations from the mean value is 1% (since 98% of the probability falls between
2.32 standard deviations of the mean).
Study Session: 1 - RA: 2
219. Correct answer: D

http://64.106.152.240/inmarkets/passpro/index.cfm?event=questions....

2004-08-02

PASS PRO - Copyright 2004 Inmarkets Ltd - Licensed to Raymond chen

128/150

Covariance = Correlation x Standard Deviation_A x Standard Deviation_B. Therefore Standard


Deviation_B
= Covariance / Correlation / Standard Deviation_A
= 5 / (0.5 x 8^0.5) = 3.5355.
Therefore Variance_B = 3.5355^2 = 12.5.
Study Session: 1 - RA: 2
220. Correct answer: D
Increasing volatility will accenuate the right skew of the lognormal price distribution. Thus if the
mean is held constant the median must shift to the left.
Study Session: 1 - RA: 2
221. Correct answer: D
Covariance = Correlation x Standard Deviation_A x Standard Deviation_B. Therefore Standard
Deviation_B
= Covariance / Correlation / Standard Deviation_A
= 4.2 / 0.6 / 5^0.5 = 3.1305.
Therefore, Variance_B = 3.1305^2 = 9.80.
Study Session: 1 - RA: 2
222. Correct answer: D
Given that the daily standard deviation is $2.5 million, the standard deviation over 14 days =
$2.5 million x (14/1)^0.5 = $9.35 million.
Given that the returns are normally distributed, we know that 95% of the outcomes will be
above 1.645 standard deviations below the mean, i.e. above $84.6 million.
Study Session: 1 - RA: 2
223. Correct answer: A
Using the multiplication rule the number of packages = Number of golf sets x Number of ways
of choosing three golf balls out of the possible five types.
The second variable in the above equation can be calculated by using combinations formula =
n! / (n-r!) / r! = 5! / 2! / 3! = 10
Therefore the number of packages = 2 x 10 = 20.
Study Session: 1 - RA: 2
224. Correct answer: A
Using the multiplication rule the number of packages = Number of golf sets x Number of ways
of choosing two golf balls out of the possible three types.
The second variable in the above equation can be calculated by using combinations formula =
n! / (n-r!) / r! = 3! / 1! / 2! = 3
Therefore the number of packages = 3 x 3 = 9.

http://64.106.152.240/inmarkets/passpro/index.cfm?event=questions....

2004-08-02

PASS PRO - Copyright 2004 Inmarkets Ltd - Licensed to Raymond chen

129/150

Study Session: 1 - RA: 2


225. Correct answer: B
Step 1. Calculate the mean stock price = (33 + 43 + 45 + 48 + 46) / 5 = 43.
Step 2. Calculate the mean S&P level = (1150 + 1125 + 1140 + 1160 + 1170) / 5 = 1149.
Step 3. The covariance between the stock and S&P = [(33 - 43)(1150 - 1149) + (43 - 43)(1125
- 1149) + (45 - 43)(1140 - 1149) + (48 - 43)(1160 - 1149) + (46 - 43)(1170 - 1149)] / 5 =
18.
Study Session: 1 - RA: 2
226. Correct answer: A
Using the multiplication rule the number of packages = Number of golf sets x Number of ways
of choosing three golf balls out of the possible five types.
The second variable in the above equation can be calculated by using combinations formula =
n! / (n-r!) / r! = 5! / 2! / 3! = 10
Therefore the number of packages = 2 x 10 = 20.
Study Session: 1 - RA: 2
227. Correct answer: C
The rate offered to the investor is the stated rate in nominal terms. The periodic rate will be
14.49% / 12 = 1.21%, and the effective rate will be (1 + 14.49%/12)^12 - 1 = 15.49%.
Study Session: 1 - RA: 3
228. Correct answer: D
The relationship between NPV and discount rate depends on the nature of the cash flows, i.e.
whether they are initially negative and then positive or vice versa.
Study Session: 1 - RA: 3
229. Correct answer: A
The value of this annuity at the start of the year 5 = Coupon x [1 - 1/(1 + i)^N]/i = 6,000 x [1
- 1/(1 + 6%)^15] / 6% = $58,273.
To find the value of this amount in today's month, we need to discount it back four years. Thus,
present value of this annuity = $58,273 / (1 + 6%)^4 = $46,158.
Study Session: 1 - RA: 3
230. Correct answer: A
The value of this annuity at the start of the year 2 = Coupon x [1 - 1/(1 + i)^N]/i = 900 x [1 1/(1 + 8%)^25] / 8% = $9,607.
To find the value of this amount in today's month we need to discount it back 1 years. Thus,
present value of this annuity = $9,607 / (1 + 8%)^1 = $8,895.

http://64.106.152.240/inmarkets/passpro/index.cfm?event=questions....

2004-08-02

PASS PRO - Copyright 2004 Inmarkets Ltd - Licensed to Raymond chen

130/150

Study Session: 1 - RA: 3


231. Correct answer: B
Using continuous compounding, PV = FV x exp(-Rate x Time period) = $4,700 x exp (- 7% x
9/12) = $4,460.
Study Session: 1 - RA: 3
232. Correct answer: B
The price of this bond is approximately equal to 100 / (1 + 0.05) ^ 2, which is equal to about
91.
Study Session: 1 - RA: 3
233. Correct answer: A
Using continuous compounding, PV = FV x exp(-Rate x Time period) = $7,000 x exp (- 8% x
8/12) = $6,636.
Study Session: 1 - RA: 3
234. Correct answer: A
The value of this annuity at the start of the year 2 = Coupon x [1 - 1/(1 + i)^N]/i = 2,000 x [1
- 1/(1 + 4%)^8] / 4% = $13,465.
To find the value of this amount in today's month, we need to discount it back one year. Thus,
present value of this annuity = $13,465 / (1 + 4%)^1 = $12,947.
Study Session: 1 - RA: 3
235. Correct answer: B
Rate of inflation = (Rise in prices)^(1/Number of years) - 1 = (2)^(1/15) - 1 = 4.73%.
Study Session: 1 - RA: 3
236. Correct answer: D
Without any negative cash flow in the stream, the rate of return is infinite. So, the IRR of this
investment cannot be determined.
Study Session: 1 - RA: 3
237. Correct answer: A
The value of this annuity at the start of the year 5 = Coupon x [1 - 1/(1 + i)^N]/i = 6,000 x [1
- 1/(1 + 6%)^15] / 6% = $58,273.
To find the value of this amount in today's month, we need to discount it back four years. Thus,
present value of this annuity = $58,273 / (1 + 6%)^4 = $46,158.
Study Session: 1 - RA: 3
238. Correct answer: A

http://64.106.152.240/inmarkets/passpro/index.cfm?event=questions....

2004-08-02

PASS PRO - Copyright 2004 Inmarkets Ltd - Licensed to Raymond chen

131/150

Continuously compounded rate = ln(FV/PV)/N = ln(15,000 / 12,000) / 2 = 11.16%.


Study Session: 1 - RA: 3
239. Correct answer: D
$1 million is a relatively small amount and the liquidity risk is not high in most markets. All
other factors are crucial for the decision.
Study Session: 1 - RA: 3
240. Correct answer: A
The value of this annuity at the start of the year 15 = Coupon x [1 - 1/(1 + i)^N]/i = 1500 x [1
- 1/(1 + 8%)^10] / 8% = $10,065.
To find the value of this amount in today's month, we need to discount it back 14 years. Thus,
present value of this annuity = $10,065 / (1 + 8%)^14 = $3,427.
Study Session: 1 - RA: 3
241. Correct answer: D
$1 million is a relatively small amount and the liquidity risk is not high in most markets. All
other factors are crucial for the decision.
Study Session: 1 - RA: 3
242. Correct answer: D
Effective rate = (1 + Stated rate / Periodicity)^Periodicity -1 = (1 + 7.75%/12)^12 -1 =
8.03%.
Study Session: 1 - RA: 3
243. Correct answer: B
Continuously compounded rate = ln(FV/PV)/N = ln(10,000 / 7,500) / 5 = 5.75%.
Study Session: 1 - RA: 3
244. Correct answer: B
Step 1. Calculate w = 45 / 180 = 0.25.
Step 2. This bond has cash flows of 2.125, 2.125, 102.125 after 0.25, 1.25, 2.25 periods
respectively.
Discounting these cash flows at the yield of 3.975% / 2 = 1.9875%, we get the present values:
2.115, 2.073, 97.702. Adding these up, the full value of this bond is $101.89.
Step 3. The accrued coupon = coupon x (1 - w) = 100 x 4.25% / 2 x (1 - 0.25) = $1.594.
Step 4. Deducting this accrued coupon from the full price gives the clean price = $100.296.

http://64.106.152.240/inmarkets/passpro/index.cfm?event=questions....

2004-08-02

PASS PRO - Copyright 2004 Inmarkets Ltd - Licensed to Raymond chen

132/150

Study Session: 1 - RA: 3


245. Correct answer: D
The relationship between NPV and discount rate depends on the nature of the cash flows, i.e.
whether they are initially negative and then positive or vice versa.
Study Session: 1 - RA: 3
246. Correct answer: B
Using continuous compounding, PV = FV x exp(-Rate x Time period) = $8,400 x exp (- 8% x
6/12) = $8,071.
Study Session: 1 - RA: 3
247. Correct answer: D
Effective rate = (1 + Stated rate / Periodicity)^Periodicity -1 = (1 + 7.75%/12)^12 -1 =
8.03%.
Study Session: 1 - RA: 3
248. Correct answer: C
FV of this payment = $4,000 x (1 + 7%)^10 = $7,869.
Study Session: 1 - RA: 3
249. Correct answer: D
Effective rate = (1 + Stated rate / Periodicity)^Periodicity -1 = (1 + 9%/12)^12 -1 = 9.38%.
Study Session: 1 - RA: 3
250. Correct answer: A
Continuously compounded rate = ln(FV/PV)/N = ln(15,000 / 12,000) / 2 = 11.16%.
Study Session: 1 - RA: 3
251. Correct answer: B
Rate of inflation = (Rise in prices)^(1/Number of years) - 1 = (2)^(1/8) - 1 = 9.05%.
Study Session: 1 - RA: 3
252. Correct answer: B
Firstly, we calculate the Mean = -4 x 40% + 8 x 25% + 15 x 35% = 5.65
Then, Variance = (5.65 - -4)^2 x 40% + (5.65 - 8)^2 x 25% + (5.65 - 15)^2 x 35% = 69.23
Finally, Standard deviation = 69.23^0.5 = 8.32.
Study Session: 1 - RA: 4

http://64.106.152.240/inmarkets/passpro/index.cfm?event=questions....

2004-08-02

PASS PRO - Copyright 2004 Inmarkets Ltd - Licensed to Raymond chen

133/150

253. Correct answer: B


Volatility scales as the square root of time. Therefore the 250-day volatility = 10-day volatility x
(250 / 10)^0.5 = 1.8% x 5 = 9%.
Study Session: 1 - RA: 4
254. Correct answer: C
Since the correlation is 1, the return on Asset B should be the same proportion as the ratio of
volatility of B to A, i.e. Return on Asset B = (Volatility B / Volatility A) x Return on Asset =
(25% /15%) x 1.50% = 2.50%.
Study Session: 1 - RA: 4
255. Correct answer: C
Correlation coefficient is a measure of the linear relationship between two random variables. It
can be calculated by scaling the covariance between them and varies between -1 (perfect
negative correlation) to +1 (perfect positive correlation). If the variables are independent they
will have a correlation coefficient of zero, but the reverse does not always hold true (i.e. a
correlation coefficient of zero does not necessarily mean that they are independent).
Covariance between the variables = Correlation coefficient x Standard deviation of the first
variable x Standard deviation of the second variable. This calculation does not require the use
of mean at all.
Study Session: 1 - RA: 4
256. Correct answer: B
Firstly, we calculate the Mean = -6 x 15% + 6 x 40% + 12 x 45% = 6.9.
Then, Variance = (6.9 - -6)^2 x 15% + (6.9 - 6)^2 x 40% + (6.9 - 12)^2 x 45% = 36.98
Finally, Standard deviation = 36.98^0.5 = 6.08.
Study Session: 1 - RA: 4
257. Correct answer: D
Mean = -10 x 30% + 5 x 40% + 25 x 30% = 6.5.
Study Session: 1 - RA: 4
258. Correct answer: D
Correlation coefficient is a measure of the linear relationship between two random variables. It
can be calculated by scaling the covariance between them and varies between -1 (perfect
negative correlation) to +1 (perfect positive correlation). If the variables are independent they
will have a correlation coefficient of zero, but the reverse does not always hold true (i.e. a
correlation coefficient of zero does not necessarily mean that they are independent).
Covariance between the variables = Correlation coefficient x Standard deviation of the first
variable x Standard deviation of the second variable. This calculation does not require the use
of mean at all.

http://64.106.152.240/inmarkets/passpro/index.cfm?event=questions....

2004-08-02

PASS PRO - Copyright 2004 Inmarkets Ltd - Licensed to Raymond chen

134/150

Study Session: 1 - RA: 4


259. Correct answer: C
Mean = -8 x 25% + 12 x 40% + 24 x 35% = 11.2.
Study Session: 1 - RA: 4
260. Correct answer: D
Covariance = Correlation x Standard Deviation_A x Standard Deviation_B. Therefore Standard
Deviation_B
= Covariance / Correlation / Standard Deviation_A
= 5 / 0.5 / 8^0.5 = 3.5355.
Therefore, Variance_B = 3.5355^2 = 12.5.
Study Session: 1 - RA: 4
261. Correct answer: B
Firstly, we calculate the Mean = -8 x 25% + 12 x 40% + 24 x 35% = 11.2
Then, Variance = (11.2 - -8)^2 x 25% + (11.2 - 12)^2 x 40% + (11.2 - 24)^2 x 35% =
149.76
Finally, Standard deviation = 149.76^0.5 = 12.24.
Study Session: 1 - RA: 4
262. Correct answer: D
Correlation coefficient is a measure of the linear relationship between two random variables. It
can be calculated by scaling the covariance between them and varies between -1 (perfect
negative correlation) to +1 (perfect positive correlation). If the variables are independent they
will have a correlation coefficient of zero, but the reverse does not always hold true (a
correlation coefficient of zero does not necessarily mean that they are independent).
Covariance between the variables = Correlation coefficient x Standard deviation of the first
variable x Standard deviation of the second variable. This calculation does not require the use
of mean at all.
Study Session: 1 - RA: 4
263. Correct answer: B
First we calculate the Mean = -4 x 45% + 8 x 40% + 25 x 15% = 5.15
Then, Variance = (5.15 - -4)^2 x 45% + (5.15 - 8)^2 x 40% + (5.15 - 25)^2 x 15% = 100.03
Finally, Standard deviation = 100.03^0.5 = 10.
Study Session: 1 - RA: 4
264. Correct answer: C

http://64.106.152.240/inmarkets/passpro/index.cfm?event=questions....

2004-08-02

PASS PRO - Copyright 2004 Inmarkets Ltd - Licensed to Raymond chen

135/150

Correlation coefficient is a measure of linear relationship between two random variables. It can
be calculated by scaling the covariance between them and varies between -1 (perfect negative
correlation) to +1 (perfect positive correlation). If the variables are independent they will have
a correlation coefficient of zero, but the reverse does not always hold true (i.e. a correlation
coefficient of zero does not necessarily mean that they are independent).
Covariance between the variables = Correlation coefficient x Standard deviation of the first
variable x Standard deviation of the second variable. This calculation does not require the use
of mean at all.
Study Session: 1 - RA: 4
265. Correct answer: C
Mean = -8 x 25% + 12 x 40% + 24 x 35% = 11.2.
Study Session: 1 - RA: 4
266. Correct answer: B
Firstly, we calculate the Mean = -6 x 15% + 6 x 40% + 12 x 45% = 6.9.
Then, Variance = (6.9 - -6)^2 x 15% + (6.9 - 6)^2 x 40% + (6.9 - 12)^2 x 45% = 36.98
Finally, Standard deviation = 36.98^0.5 = 6.08.
Study Session: 1 - RA: 4
267. Correct answer: A
Correlation coefficient is a measure of the linear relationship between two random variables. It
can be calculated by scaling the covariance between them and varies between -1 (perfect
negative correlation) to +1 (perfect positive correlation). If the variables are independent they
will have a correlation coefficient of zero, but the reverse does not always hold true (i.e. a
correlation coefficient of zero does not necessarily mean that they are independent).
Covariance between the variables = Correlation coefficient x Standard deviation of the first
variable x Standard deviation of the second variable. This calculation does not require the use
of mean at all.
Study Session: 1 - RA: 4
268. Correct answer: B
Volatility scales as the square root of time. Therefore, the 23-day volatility = 10-day volatility x
(23 / 10)^0.5 = 1.4% x 1.5166 = 2.12%.
Study Session: 1 - RA: 4
269. Correct answer: A
Correlation coefficient is a measure of the linear relationship between two random variables. It
can be calculated by scaling the covariance between them and varies between -1 (perfect
negative correlation) to +1 (perfect positive correlation). If the variables are independent they
will have a correlation coefficient of zero, but the reverse does not always hold true (i.e. a
correlation coefficient of zero does not necessarily mean that they are independent).

http://64.106.152.240/inmarkets/passpro/index.cfm?event=questions....

2004-08-02

PASS PRO - Copyright 2004 Inmarkets Ltd - Licensed to Raymond chen

136/150

Covariance between the variables = Correlation coefficient x Standard deviation of the first
variable x Standard deviation of the second variable. This calculation does not require the use
of mean at all.
Study Session: 1 - RA: 4
270. Correct answer: C
Since the correlation is 1, the return on Asset B should be the same proportion as the ratio of
volatility of B to A, i.e. Return on Asset B = (Volatility B / Volatility A) x Return on Asset =
(25% /15%) x 1.50% = 2.50%.
Study Session: 1 - RA: 4
271. Correct answer: B
Volatility scales as the square root of time. Therefore, the 30-day volatility = 7-day volatility x
(30 / 7)^0.5 = 1.05% x 2.0702 = 2.17%.
Study Session: 1 - RA: 4
272. Correct answer: D
Mean = -2 x 60% + 10 x 20% + 20 x 20% = 4.8.
Study Session: 1 - RA: 4
273. Correct answer: B
Firstly, we calculate the Mean = -4 x 40% + 8 x 25% + 15 x 35% = 5.65
Then, Variance = (5.65 - -4)^2 x 40% + (5.65 - 8)^2 x 25% + (5.65 - 15)^2 x 35% = 69.23
Finally, Standard deviation = 69.23^0.5 = 8.32.
Study Session: 1 - RA: 4
274. Correct answer: B
Firstly, we calculate the Mean = -2 x 60% + 10 x 20% + 20 x 20% = 4.8
Then, Variance = (4.8 - -2)^2 x 60% + (4.8 - 10)^2 x 20% + (4.8 - 20)^2 x 20% = 79.36
Finally, Standard deviation = 79.36^0.5 = 8.91.
Study Session: 1 - RA: 4
275. Correct answer: B
Volatility scales as the square root of time. Therefore, the 30-day volatility = 7-day volatility x
(30 / 7)^0.5 = 1.05% x 2.0702 = 2.17%.
Study Session: 1 - RA: 4
276. Correct answer: B

http://64.106.152.240/inmarkets/passpro/index.cfm?event=questions....

2004-08-02

PASS PRO - Copyright 2004 Inmarkets Ltd - Licensed to Raymond chen

137/150

Firstly, we calculate the Mean = -15 x 30% + 5 x 40% + 25 x 30% = 5


Then, Variance = (5 - -15)^2 x 30% + (5 - 5)^2 x 40% + (5 - 25)^2 x 30% = 240
Finally, Standard deviation = 240^0.5 = 15.49.
Study Session: 1 - RA: 4
277. Correct answer: A
Coefficient of variation (CV) is a measure of relative risk and is calculated as: CV = (standard
deviation of returns) / (expected rate of return). The calculations for each investment are
shown below:
CV(A) = 0.004^0.5 / 0.18 = 0.351.
CV(B) = 0.005^0.5 / 0.25 = 0.283.
CV(C) = 0.04 / 0.12 = 0.333.
CV(D) = 0.025 / 0.08 = 0.313.
Study Session: 1 - RA: 5
278. Correct answer: D
Coefficient of variation (CV) is a measure of relative risk and is calculated as: CV = (standard
deviation of returns) / (expected rate of return). The calculations for each investment are
shown below:
CV(A) = 0.005^0.5 / 0.12 = 0.589.
CV(B) = 0.002^0.5 / 0.1 = 0.447.
CV(C) = 0.03 / 0.15 = 0.2.
CV(D) = 0.05 / 0.06 = 0.833.
Study Session: 1 - RA: 5
279. Correct answer: D
The null hypothesis for this test is that the coefficient is equal to zero. Therefore, the test
statistic = (Observed value - 0) / Standard error = (6% - 0) / 3% = 2.
Study Session: 1 - RA: 5
280. Correct answer: C
Coefficient of variation (CV) is a measure of relative risk and is calculated as: CV = (standard
deviation of returns) / (expected rate of return). The calculations for each investment are
shown below:
CV(A) = 0.005^0.5 / 0.24 = 0.295.
CV(B) = 0.002^0.5 / 0.18 = 0.248.
CV(C) = 0.05 / 0.15 = 0.333.
CV(D) = 0.08 / 0.25 = 0.32.
Study Session: 1 - RA: 5
281. Correct answer: C
Standard error of the mean = s /vn = 4/v40 = 0.63.

http://64.106.152.240/inmarkets/passpro/index.cfm?event=questions....

2004-08-02

PASS PRO - Copyright 2004 Inmarkets Ltd - Licensed to Raymond chen

138/150

Study Session: 1 - RA: 5


282. Correct answer: A
The null hypothesis for this test is that the coefficient is equal to zero. Therefore the test
statistic = (Observed value - 0) / Standard error = (15% - 0) / 9% = 1.67.
Study Session: 1 - RA: 5
283. Correct answer: A
Coefficient of variation (CV) is a measure of relative risk and is calculated as: CV = (standard
deviation of returns) / (expected rate of return). The calculations for each investment are
shown below:
CV(A) = 0.004^0.5 / 0.18 = 0.351.
CV(B) = 0.005^0.5 / 0.25 = 0.283.
CV(C) = 0.04 / 0.12 = 0.333.
CV(D) = 0.025 / 0.08 = 0.313.
Study Session: 1 - RA: 5
284. Correct answer: D
The null hypothesis for this test is that the coefficient is equal to zero. Therefore the test
statistic = (Observed value - 0) / Standard error = (12.25% - 0) / 8% = 1.53.
Study Session: 1 - RA: 5
285. Correct answer: C
Coefficient of variation (CV) is a measure of relative risk and is calculated as: CV = (standard
deviation of returns) / (expected rate of return). The calculations for each investment are
shown below:
CV(A) = 0.005^0.5 / 0.24 = 0.295.
CV(B) = 0.002^0.5 / 0.18 = 0.248.
CV(C) = 0.05 / 0.15 = 0.333.
CV(D) = 0.08 / 0.25 = 0.32.
Study Session: 1 - RA: 5
286. Correct answer: C
Standard error of the mean = s / n^0.5 = 3.5 / 30^0.5 = 0.64.
Study Session: 1 - RA: 5
287. Correct answer: A
The null hypothesis for this test is that the coefficient is equal to zero. Therefore the test
statistic = (Observed value - 0) / Standard error = (15% - 0) / 9% = 1.67.
Study Session: 1 - RA: 5
288. Correct answer: D

http://64.106.152.240/inmarkets/passpro/index.cfm?event=questions....

2004-08-02

PASS PRO - Copyright 2004 Inmarkets Ltd - Licensed to Raymond chen

139/150

The null hypothesis for this test is that the coefficient is equal to zero. Therefore the test
statistic = (Observed value - 0) / Standard error = (9% - 0) / 3.25% = 2.77.
Study Session: 1 - RA: 5
289. Correct answer: C
In this distribution, the lowest element is 1.5 percent below the mean whereas the highest
element is 2.5 percent above the mean. Thus the distribution is skewed towards the right.
Study Session: 1 - RA: 5
290. Correct answer: D
Step 1. The returns for the three scenarios given are: 42.857% [= (50 - 35)/35], 20% [= (42 35) / 35], and -42.857% [= (20 - 35) / 35]
Step 2. Calculate expected price = 33% x 42.857% + 20% x 20% + 47% x -42.857% = -2%.
Step 3. Calculate Variance = 33% x (-2% - 42.857%)^2 + 20% x (-2% - 20%)^2 + 47% x (2% + 42.857%)^2 = 0.154538
Step 4. Calculate volatility/standard deviation = 0.154538^0.5 = 39.31%.
Study Session: 1 - RA: 5
291. Correct answer: D
The null hypothesis for this test is that the coefficient is equal to zero. Therefore the test
statistic = (Observed value - 0) / Standard error = (11.8% - 0) / 3% = 3.93.
Study Session: 1 - RA: 5
292. Correct answer: C
Standard error of the mean = s /n^0.5 = 7/60^0.5 = 0.9.
Study Session: 1 - RA: 5
293. Correct answer: C
Standard error of the mean = s /n^0.5 = 6/50^0.5 = 0.85.
Study Session: 1 - RA: 5
294. Correct answer: D
The null hypothesis for this test is that the coefficient is equal to zero. Therefore, the test
statistic = (Observed value - 0) / Standard error = (6% - 0) / 3% = 2.
Study Session: 1 - RA: 5
295. Correct answer: A
Coefficient of variation (CV) is a measure of relative risk and is calculated as: CV = (standard
deviation of returns) / (expected rate of return). The calculations for each investment are

http://64.106.152.240/inmarkets/passpro/index.cfm?event=questions....

2004-08-02

PASS PRO - Copyright 2004 Inmarkets Ltd - Licensed to Raymond chen

140/150

shown below:
CV(A) = 0.005^0.5 / 0.19 = 0.372.
CV(B) = 0.002^0.5 / 0.15 = 0.298.
CV(C) = 0.03 / 0.1 = 0.3.
CV(D) = 0.01 / 0.03 = 0.333.
Study Session: 1 - RA: 5
296. Correct answer: B
The null hypothesis for this test is that the coefficient is equal to zero. Therefore the test
statistic = (Observed value - 0) / Standard error = (13.6% - 0) / 5% = 2.72.
Study Session: 1 - RA: 5
297. Correct answer: A
Coefficient of variation (CV) is a measure of relative risk and is calculated as: CV = (standard
deviation of returns) / (expected rate of return). The calculations for each investment are
shown below:
CV(A) = 0.002^0.5 / 0.1 = 0.447.
CV(B) = 0.003^0.5 / 0.15 = 0.365.
CV(C) = 0.03 / 0.08 = 0.375.
CV(D) = 0.01 / 0.05% = 0.2.
Study Session: 1 - RA: 5
298. Correct answer: B
The null hypothesis for this test is that the coefficient is equal to zero. Therefore the test
statistic = (Observed value - 0) / Standard error = (13.6% - 0) / 5% = 2.72.
Study Session: 1 - RA: 5
299. Correct answer: B
In hypothesis testing, Type I error is rejecting the null hypothesis when it is actually true. The
probability of Type I error is called the "level of significance", so choosing a lower level of
significance lowers the probability of Type I error. Type II error happens if the null hypothesis is
not rejected when it is actually false. The power of the test is the probability of correctly
rejecting the null hypothesis (when it is false), so minimizing Type II errors would maximize the
power of the test.
Study Session: 1 - RA: 6
300. Correct answer: C
Economic significance and statistical significance are not the same. Economic significance takes
into account statistical significance but also a number of other issues (externalities such as
transaction costs, regulations etc.) Note: the wording of the choices is a bit tricky. Choice A is
right in some cases, but in many situations statistical significance can suggest economic
significance, although it cannot confirm significance./p>
Study Session: 1 - RA: 6

http://64.106.152.240/inmarkets/passpro/index.cfm?event=questions....

2004-08-02

PASS PRO - Copyright 2004 Inmarkets Ltd - Licensed to Raymond chen

141/150

301. Correct answer: A


Tests of the variance of a population require the chi-squared test. For this data chi-squared =
(n - 1) x Sample variance / Hypothesized variance = 29 x 0.12^2 / 0.2^2 = 10.44. Since the
analyst wants to show that the standard deviation is less than 20%, this will be chosen as the
alternative hypothesis and the null hypothesis will be that the standard deviation is higher than
or equal to 20%. The critical value of the chi-squared statistic (for df = 29 and p= 95%) is
17.71. Since the test statistic is lower than the critical statistic, we can reject the null
hypothesis that the standard deviation is equal to or higher than 20%, and accept the
alternative hypothesis that the standard deviation is lower than 20%.
Study Session: 1 - RA: 6
302. Correct answer: D
In hypothesis testing we accept the alternate hypothesis if the null hypothesis has been
rejected. Type I error happens if the null hypothesis is rejected when it is actually true. The
probability of Type I error is called the level of significance, so choosing a lower level of
significance lowers the probability of Type I error. Type II error happens if the null hypothesis is
not rejected when it is actually false.
Study Session: 1 - RA: 6
303. Correct answer: A
This is a hypothesis test with the null hypothesis: performance of firms with a P/B ratio 25
percent below average is "less than or equal to" the market. If the calculated value of the test
statistic is higher than the critical value, we reject the null hypothesis and accept the alternative
hypothesis.
Study Session: 1 - RA: 6
304. Correct answer: C
Tests of differences between variances of two populations require the F-test. The value of the
test F-statistic = Higher variance / Lower variance = 0.13^2 / 0.1^2 = 1.69.
Study Session: 1 - RA: 6
305. Correct answer: B
Tests of the variance of a population require the chi-squared test. For this data, chi-squared =
(n - 1) x Sample variance / Hypothesized variance = 50 x 0.24^2 / 0.04 = 72. Since the
analyst wants to show that the variance is more than 0.04, this will be chosen as the alternative
hypothesis and the null hypothesis will be that the variance is lower than or equal to 0.04. The
critical value of the chi-squared statistic (for df=50 and p=0.05) is 67.505. Since the test
statistic is higher than the critical statistic, we can reject the null hypothesis (variance <=
0.04), and accept the alternative hypothesis (variance > 0.04).
Study Session: 1 - RA: 6
306. Correct answer: B
Population mean and probability distribution are usually unknown, which is why hypothesis
testing has to be undertaken. The steps in the hypothesis testing process are: 1) State a
hypothesis. 2) Identify the test statistic and its probability distribution. 3) Specify the
significance level. 4) State the decision rule. 5) Collect data and perform the calculations. 6)
Make the statistical decision. 7) Make the investment/economic decision based on the above.

http://64.106.152.240/inmarkets/passpro/index.cfm?event=questions....

2004-08-02

PASS PRO - Copyright 2004 Inmarkets Ltd - Licensed to Raymond chen

142/150

Study Session: 1 - RA: 6


307. Correct answer: A
Tests of the variance of a population require the chi-squared test. Since the analyst wants to
show that the standard deviation is less than 20%, this will be chosen as the alternative
hypothesis and the null hypothesis will be that the standard deviation is higher than or equal to
20%. Thus using a probability in the right tail of 95% and degrees of freedom of 29, the critical
value from the chi-squared table is 17.71.
Study Session: 1 - RA: 6
308. Correct answer: C
This is a two-tailed t-test for the difference between means of two independent populations.
Step 1. The common variance = [(n1 -1) x Variance1 + (n2 - 1) x Variance2] / (n1 + n2 - 2) =
[(21 -1) x 0.12^2 + (41 - 1) x 0.13^2] / (21 + 41 - 2) = 0.0161. Step 2. The value of the test
statistic = (Mean1 - Mean2) / (Common variance/n1 + Common variance/n2)^0.5 = (0.15 0.12) / (0.0161/21 + 0.0161/41)^0.5 = 0.88. Using degrees of freedom = n1 + n2 - 2 = 21 +
41 - 2 = 60 and level of significance = alpha / 2 = 0.025 the critical value = 2.0. Since the test
statistic is lower than the critical statistic, we cannot reject the null hypothesis that the mean
returns are same.
Study Session: 1 - RA: 6
309. Correct answer: A
Tests of the variance of a population require the chi-squared test. For this data chi-squared =
(n - 1) x Sample variance / Hypothesized variance = 39 x 0.11^2 / 0.2^2 = 11.8.
Study Session: 1 - RA: 6
310. Correct answer: C
The required test for testing the variance is the chi-squared test. The test statistic = (n - 1) x
Sample variance / Hypothesized variance = 14 x 0.1^2 / 0.15^2 = 6.22. To test whether the
standard deviation is different (Ho: standard deviation is equal to 15%), the critical values of
chi-squared will be 5.63 and 26.12 (using df=14, p-lower=0.975 and p-higher=0.025). Since
the test statistic falls inside the critical range, the analyst cannot reject the null hypothesis .
Study Session: 1 - RA: 6
311. Correct answer: D
Tests of the variance of a population require the chi-squared test. For this data chi-squared =
(n - 1) x Sample variance / Hypothesized variance = 60 x 0.22^2 / 0.045 = 64.53. Since the
analyst wants to show that the variance is more than 0.045, this will be chosen as the
alternative hypothesis and the null hypothesis will be that the variance is lower than or equal to
0.045. The critical value of the chi-squared statistic (for df = 60 and p = 5%) is 79.08. Since
the test statistic is lower than the critical statistic, we cannot reject the null hypothesis that the
variance is equal to or lower than 0.045.
Study Session: 1 - RA: 6
312. Correct answer: D
This problem requires a two-tailed F-test. Looking up the F-table for 0.025 significance level,

http://64.106.152.240/inmarkets/passpro/index.cfm?event=questions....

2004-08-02

PASS PRO - Copyright 2004 Inmarkets Ltd - Licensed to Raymond chen

143/150

with 40 degrees of freedom in the numerator and 20 in the denominator (note the population
with the higher variance goes in the numerator and that this is a two-sided test so p = 0.025),
the critical value of F is 2.29.
Study Session: 1 - RA: 6
313. Correct answer: C
In hypothesis testing, we accept the alternate hypothesis if the null hypothesis has been
rejected. Type I error happens if the null hypothesis is rejected when it is actually true. Type II
error happens if the null hypothesis is not rejected when it is actually false. The power of the
test is the probability of correctly rejecting the null hypothesis (when it is false), so minimizing
Type II errors would maximize the power of the test.
Study Session: 1 - RA: 6
314. Correct answer: B
The 95% confidence interval = Mean 1.96 x Standard error = Mean 1.96 x Standard
deviation / Sample size^0.5 = 20 1.96 x 8.5 / 50^0.5 = 17.64 to 22.36.
Study Session: 1 - RA: 6
315. Correct answer: C
This problem requires an F-test in which the null hypothesis Ho: variance of stock A = variance
of stock B. The test statistic = higher variance / lower variance = 0.012 / 0.005 = 2.4. The
critical F with 14 and 19 degrees of freedom and p=0.025 is 2.65. Since the test statistic is
lower, the analyst cannot reject the null hypothesis.
Study Session: 1 - RA: 6
316. Correct answer: A
In hypothesis testing we accept the alternate hypothesis if the null hypothesis has been
rejected. Type I error happens if the null hypothesis is rejected when it is actually true. Type II
error happens if the null hypothesis is accepted when it is actually false. The power of the test
is the probability of correctly rejecting the null hypothesis (when it is false), so minimizing Type
II errors would maximize the power of the test.
Study Session: 1 - RA: 6
317. Correct answer: A
The required test for testing the variance is the chi-squared test. The test statistic = (n - 1) x
Sample variance / Hypothesized variance = 100 x 0.1^2 / 0.18^2 = 30.86. To test whether the
standard deviation is lower (Ho: standard deviation is higher than or equal to 18%), the critical
value of chi-squared will be 77.93 (using df = 100 and p= 0.95). Since the test statistic is lower
than the critical value, the analyst can reject the null hypothesis and conclude that the standard
deviation of returns is lower than 18%.
Study Session: 1 - RA: 6
318. Correct answer: C
Since this is a one-tailed test with a 0.05 significance level and 14 degrees of freedom, the
critical value from the t-table is 1.76.

http://64.106.152.240/inmarkets/passpro/index.cfm?event=questions....

2004-08-02

PASS PRO - Copyright 2004 Inmarkets Ltd - Licensed to Raymond chen

144/150

Study Session: 1 - RA: 6


319. Correct answer: C
The null hypothesis is usually set to the opposite of what we are trying to proves, so that
discrediting the null hypothesis confirms our belief. In this case the null hypothesis H0:
performance of active portfolio managers < or = average for all portfolio managers.
Study Session: 1 - RA: 6
320. Correct answer: A
The 95% confidence interval = Mean 1.96 x Standard error = 18 1.96 x 7.25 = 3.79 to
32.21.
Study Session: 1 - RA: 6
321. Correct answer: A
The hypothesis test that the analyst will construct for this study is H0: Spending is equal to or
less than $2,000; and Ha = Spending is more than $2,000. Thus we have only one rejection
region (if the test statistic is higher than the critical statistic), i.e. this is a one-tailed test.
Study Session: 1 - RA: 6
322. Correct answer: C
This is a two-tailed t-test for the difference between means of two independent populations.
Step 1. The common variance = [(n1 -1) x Variance1 + (n2 - 1) x Variance2] / (n1 + n2 - 2) =
[(21 -1) x 0.13^2 + (41 - 1) x 0.15^2] / (21 + 41 - 2) = 0.0206. Step 2. The value of the test
statistic = (Mean1 - Mean2) / (Common variance/n1 + Common variance/n2)^0.5 = (0.12 0.09) / (0.0206/21 + 0.0206/41)^0.5 = 0.78. Using degrees of freedom = n1 + n2 - 2 = 21 +
41 - 2 = 60 and level of significance = alpha / 2 = 0.025 the critical value = 2.0. Since the test
statistic is lower than the critical statistic, we cannot reject the null hypothesis that the mean
returns are same.
Study Session: 1 - RA: 6
323. Correct answer: C
The null hypothesis is usually set to the opposite of what we are trying to proves, so that
discrediting the null hypothesis confirms our belief. In this case the null hypothesis H0:
performance of active portfolio managers < or = average for all portfolio managers.
Study Session: 1 - RA: 6
324. Correct answer: C
Coefficient of determination = ESS / TSS = 761.562 / 865.333 = 0.88.
Study Session: 1 - RA: 7
325. Correct answer: D
The standard error = USS / (N-2) = (TSS - ESS) / (N-2) = (113,171.429 - 96,057.049) / (7-2)
= 3,422.876.

http://64.106.152.240/inmarkets/passpro/index.cfm?event=questions....

2004-08-02

PASS PRO - Copyright 2004 Inmarkets Ltd - Licensed to Raymond chen

145/150

Study Session: 1 - RA: 7


326. Correct answer: D
The percentage of explained variation = Correlation coefficient ^ 2 = 0.6 ^ 2 = 0.36 or 36%.
Therefore, the unexplained variation is 64%.
Study Session: 1 - RA: 7
327. Correct answer: B
The percentage of explained variation = Correlation coefficient ^ 2 = 0.85 ^ 2 = 0.7225 or
72.25%. Therefore, the unexplained variation is 27.75%.
Study Session: 1 - RA: 7
328. Correct answer: A
The slope coefficient = Sum_XY / Sum_X^2 = -9,152.23 / 368.857 = -24.812.
Study Session: 1 - RA: 7
329. Correct answer: D
The percentage of explained variation = Correlation coefficient ^ 2 = 0.33 ^ 2 = 0.1089 or
10.89%. Therefore, the unexplained variation is 89.11%.
Study Session: 1 - RA: 7
330. Correct answer: D
The slope coefficient = Sum_XY / Sum_X^2 = 133.302 / 23.333 = 5.713.
Study Session: 1 - RA: 7
331. Correct answer: D
Correlation = (Coefficient of determination)^0.5 = (ESS / TSS)^0.5 = (96,057.049 /
113,171.429)^0.5 = 0.921.
Study Session: 1 - RA: 7
332. Correct answer: C
Coefficient of determination = ESS / TSS = 761.562 / 865.333 = 0.88.
Study Session: 1 - RA: 7
333. Correct answer: C
Coefficient of determination = ESS / TSS = 96,057.049 / 113,171.429 = 0.849.
Study Session: 1 - RA: 7
334. Correct answer: D

http://64.106.152.240/inmarkets/passpro/index.cfm?event=questions....

2004-08-02

PASS PRO - Copyright 2004 Inmarkets Ltd - Licensed to Raymond chen

146/150

The standard error = USS / (N-2) = (TSS - ESS) / (N-2) = (865.333 - 761.562) / (6-2) =
25.94.
Study Session: 1 - RA: 7
335. Correct answer: D
The standard error = USS / (N-2) = (TSS - ESS) / (N-2) = (865.333 - 761.562) / (6-2) =
25.94.
Study Session: 1 - RA: 7
336. Correct answer: D
Correlation = (Coefficient of determination)^0.5 = (ESS / TSS)^0.5 = (3.312 / 3.617)^0.5 =
0.957.
Study Session: 1 - RA: 7
337. Correct answer: D
First we calculate the slope coefficient = Sum_XY / Sum_X^2 = -9,152.23 / 368.857 = 24.812.
Then we calculate the intercept = Avg_Y - Slope x Avg X = 627.143 - -24.812 x 14.857 =
995.77.
Study Session: 1 - RA: 7
338. Correct answer: D
Standard error of the mean = Standard deviation / (Number of observations)^0.5 = 15 /
60^0.5 = 1.94.
Study Session: 1 - RA: 7
339. Correct answer: C
Coefficient of determination = ESS / TSS = 3.312 / 3.617 = 0.916.
Study Session: 1 - RA: 7
340. Correct answer: D
The standard error = USS / (N-2) = (TSS - ESS) / (N-2) = (245,142.857 - 227,088.816) / (72) = 3,610.8082.
Study Session: 1 - RA: 7
341. Correct answer: D
Standard error of the mean = Standard deviation / (Number of observations)^0.5 = 15 /
60^0.5 = 1.94.
Study Session: 1 - RA: 7
342. Correct answer: D

http://64.106.152.240/inmarkets/passpro/index.cfm?event=questions....

2004-08-02

PASS PRO - Copyright 2004 Inmarkets Ltd - Licensed to Raymond chen

147/150

The standard error = USS / (N-2) = (TSS - ESS) / (N-2) = (3.617 - 3.312) / (7-2) = 0.061.
Study Session: 1 - RA: 7
343. Correct answer: D
The percentage of explained variation = Correlation coefficient ^ 2 = 0.33 ^ 2 = 0.1089 or
10.89%. Therefore, the unexplained variation is 89.11%.
Study Session: 1 - RA: 7
344. Correct answer: A
The slope coefficient = Sum_XY / Sum_X^2 = 9.08 / 24.9 = 0.365.
Study Session: 1 - RA: 7
345. Correct answer: D
First we calculate the slope coefficient = Sum_XY / Sum_X^2 = -9,152.23 / 368.857 = 24.812.
Then we calculate the intercept = Avg_Y - Slope x Avg X = 627.143 - -24.812 x 14.857 =
995.77.
Study Session: 1 - RA: 7
346. Correct answer: D
The slope coefficient = Sum_XY / Sum_X^2 = 133.302 / 23.333 = 5.713.
Study Session: 1 - RA: 7
347. Correct answer: B
Firstly, we calculate the slope coefficient = Sum_XY / Sum_X^2 = 1,640 / 28 = 58.571.
Then, we calculate the intercept = Avg_Y - Slope x Avg X = 455.714 - 58.571 x 4 = 221.43.
Study Session: 1 - RA: 7
348. Correct answer: C
The test to compare variances of two normally distributed populations is F-statistic.
Study Session: 1 - RA: 7
349. Correct answer: D
Correlation coefficient = Covariance / (Standard deviation of stock A x Standard deviation of
stock B) = 0.1013 / (0.30 x 0.45) = 0.75.
Study Session: 1 - RA: 8
350. Correct answer: C

http://64.106.152.240/inmarkets/passpro/index.cfm?event=questions....

2004-08-02

PASS PRO - Copyright 2004 Inmarkets Ltd - Licensed to Raymond chen

148/150

Correlation coefficient = Covariance / (Standard deviation of stock A x Standard deviation of


market index) = -0.002 / (0.018^0.5 x 0.025^0.5) = -0.09.
Study Session: 1 - RA: 8
351. Correct answer: A
Covariance = Standard deviation of stock A x Standard deviation of stock B x Correlation
coefficient = 0.30 x 0.45 x 0.75 =0.1013.
Study Session: 1 - RA: 8
352. Correct answer: D
Since the investments are perfectly negatively correlated the volatility of the portfolio = (w1 x
v1 - w2 x v2). This can be reduced to zero by adjusting the amounts invested in the inverse of
the ratios of the volatilities (i.e. w1/w2 = v2/v1)
Study Session: 1 - RA: 8
353. Correct answer: D
Since the investments are perfectly negatively correlated the volatility of the portfolio = (w1 x
v1 - w2 x v2). This can be reduced to zero by adjusting the amounts invested in the inverse of
the ratios of the volatilities (i.e. w1/w2 = v2/v1)
Study Session: 1 - RA: 8
354. Correct answer: A
Covariance = Standard deviation of stock A x Standard deviation of stock B x Correlation
coefficient = 0.30 x 0.45 x 0.75 =0.1013.
Study Session: 1 - RA: 8
355. Correct answer: A
Since the investments are perfectly negatively correlated the volatility of the portfolio = (w1 x
v1 - w2 x v2). This can be reduced to zero by adjusting the amounts invested in the inverse of
the ratios of the volatilities (i.e. w1/w2 = v2/v1)
Study Session: 1 - RA: 8
356. Correct answer: C
The positive correlation only means that the prices will tend to move in the same direction, not
that they always will.
Study Session: 1 - RA: 8
357. Correct answer: A
Since the investments are perfectly negatively correlated the volatility of the portfolio = (w1 x
v1 - w2 x v2). This can be reduced to zero by adjusting the amounts invested in the inverse of
the ratios of the volatilities (i.e. w1/w2 = v2/v1)

http://64.106.152.240/inmarkets/passpro/index.cfm?event=questions....

2004-08-02

PASS PRO - Copyright 2004 Inmarkets Ltd - Licensed to Raymond chen

149/150

Study Session: 1 - RA: 8


358. Correct answer: D
Correlation coefficient = Covariance / (Standard deviation of stock A x Standard deviation of
stock B) = 0.1013 / (0.30 x 0.45) = 0.75.
Study Session: 1 - RA: 8
359. Correct answer: D
Covariance of returns is an absolute measure of movement. The magnitude depends on size of
the individual assets standard deviations and the relationship between co-movements.
A positive covariance means that the asset returns move in the same direction, while a negative
covariance means that asset returns move in the opposite direction.
A zero covariance means that there is no relationship between asset returns.
Study Session: 1 - RA: 8
360. Correct answer: B
Only the Hull and White and the Heath, Jarrow and Morton are no-arbitrage models.
Study Session: 1 - RA: 10
361. Correct answer: B
Only the Hull and White and the Heath, Jarrow and Morton are no-arbitrage models.
Study Session: 1 - RA: 10
362. Correct answer: B
If X and Y are normally distributed, X + Y will also be normally distributed.
Study Session: 1 - RA: 10
363. Correct answer: D
Of the models given here, only Heath, Jarrow and Morton is a no-arbitrage model that
incorporates mean reversion. Cox, Ingersoll and Ross model and Vasicek model incorporate
mean reversion but are not a no-arbitrage model. Black-Scholes does not meet either criteria.
Study Session: 1 - RA: 10
364. Correct answer: A
Since X and Y follow geometric Brownian motion, it follows that log(X) and log(Y) are normally
distributed, which means that log(X) + log(Y) is also normally distributed. log(X) + log(Y) is
equal to log(X * Y), so X * Y must follow geometric Brownian motion.
Study Session: 1 - RA: 10
365. Correct answer: C

http://64.106.152.240/inmarkets/passpro/index.cfm?event=questions....

2004-08-02

PASS PRO - Copyright 2004 Inmarkets Ltd - Licensed to Raymond chen

150/150

The Hull and White, the Ho and Lee, and the Heath, Jarrow and Morton are no-arbitrage
models.
Study Session: 1 - RA: 10
366. Correct answer: B
In the model: dr = a * (b - r) * dt + s * dz, the interest rate r will mean revert to a level b.
Study Session: 1 - RA: 10
367. Correct answer: C
dr = a * (b - r) * dt + s * dz, is the only model given above that allows for the interest rate r to
mean revert to a level b.
Study Session: 1 - RA: 10
368. Correct answer: D
Only Hull and White is a no-arbitrage model that incorporates mean reversion. Ho and Lee is a
no-arbitrage model that does not incorporate mean reversion. Vasicek model incorporates mean
reversion but is not a no-arbitrage model. Black-Scholes does not meet either criteria.
Study Session: 1 - RA: 10
369. Correct answer: B
Wiener process has a mean change of zero and variance proportional to the time interval.
Generalized Wiener process and Ito process incorporate a trend variable a * dt.
Study Session: 1 - RA: 10
370. Correct answer: B
If X and Y are lognormally distributed, log(X) and log(Y) will be normally distributed, which
means that log(X) + log(Y) = log(X * Y) is also normally distributed, implying that X * Y must
be lognormally distributed.
Study Session: 1 - RA: 10

PASS PRO , Expected Score , Adaptive Practice , Targeted Practice and Pick'N'Mix Test Creator
are trademarks of Inmarkets Ltd.

http://64.106.152.240/inmarkets/passpro/index.cfm?event=questions....

2004-08-02

You might also like